Какое число больше m или n если m 7 умножить на n: Свойства степеней, действия со степенями

Сумма и произведение цифр числа. Решение задачи на Python

Одной из часто используемых задач для начинающих изучать программирование является нахождение суммы и произведения цифр числа. Число может вводиться с клавиатуры или генерироваться случайно. Задача формулируется так:

Дано число. Найти сумму и произведение его цифр.


Например, сумма цифр числа 253 равна 10-ти, так как 2 + 5 + 3 = 10. Произведение цифр числа 253 равно 30-ти, так как 2 * 5 * 3 = 30.

В данном случае задача осложняется тем, что количество разрядов числа заранее (на момент написания программы) не известно. Это может быть и трехзначное число, как в примере выше, и восьмизначное, и однозначное.

Обычно предполагается, что данная задача должна быть решена арифметическим способом и с использованием цикла. То есть с заданным число должны последовательно выполняться определенные арифметические действия, позволяющие извлечь из него все цифры, затем сложить их и перемножить.

При этом используются операции деления нацело и нахождения остатка. Если число разделить нацело на 10, произойдет «потеря» последней цифры числа. Например, 253 ÷ 10 = 25 (остаток 3). С другой стороны, эта потерянная цифра есть остаток от деления. Получив эту цифру, мы можем добавить ее к сумме цифр и умножить на нее произведение цифр числа.

Пусть n – само число, suma – сумма его цифр, а mult – произведение. Тогда алгоритм нахождения суммы и произведения цифр можно словесно описать так:

  1. Переменной suma присвоить ноль.
  2. Переменной mult присвоить единицу. Присваивать 0 нельзя, так как при умножении на ноль результат будет нулевым.
  3. Пока значение переменной n больше нуля повторять следующие действия:
    1. Найти остаток от деления значения n на 10, то есть извлечь последнюю цифру числа.
    2. Добавить извлеченную цифру к сумме и увеличить на эту цифру произведение.
    3. Избавиться от последнего разряда числа n путем деления нацело на 10.

В языке Python операция нахождения остатка от деления обозначается знаком процента — %. Деление нацело — двумя слэшами — //.

Код программы на языке Python

n = int(input())
 
suma = 0
mult = 1
 
while n > 0:
    digit = n % 10
    suma = suma + digit
    mult = mult * digit
    n = n // 10
 
print("Сумма:", suma)
print("Произведение:", mult)

Пример выполнения:

253
Сумма: 10
Произведение: 30

Изменение значений переменных можно записать в сокращенном виде:

...
while n > 0:
    digit = n % 10
    suma += digit
    mult *= digit
    n //= 10
...

Приведенная выше программа подходит только для нахождения суммы и произведения цифр натуральных чисел, то есть целых чисел больше нуля. Если исходное число может быть любым целым, следует учесть обработку отрицательных чисел и нуля.

Если число отрицательное, это не влияет на сумму его цифр. В таком случае достаточно будет использовать встроенную в Python функции abc, которая возвращает абсолютное значение переданного ей аргумента. Она превратит отрицательное число в положительное, и цикл while с его условием n > 0 будет работать как и прежде.

Если число равно нулю, то по логике вещей сумма его цифр и их произведение должны иметь нулевые значения. Цикл срабатывать не будет. Поскольку исходное значение mult — это 1, следует добавить проверку на случай, если заданное число — это ноль.

Программа, обрабатывающая все целые числа, может начинаться так:

n = abs(int(input()))
 
suma = 0
mult = 1
if n == 0:
    mult = 0
...

Заметим, если в самом числе встречается цифра 0 (например, 503), то произведение всех цифр будет равно нулю. Усложним задачу:

Вводится натуральное число. Найти сумму и произведение цифр, из которых состоит это число. При этом если в числе встречается цифра 0, то ее не надо учитывать при нахождении произведения.

Для решения такой задачи в цикл добавляется проверка извлеченной цифры на ее неравенство нулю. Делать это надо до умножения на нее значения переменной-произведения.

n = int(input())
 
suma = 0
mult = 1
 
while n > 0:
    digit = n % 10
    if digit != 0:  
        suma += digit
        mult *= digit
    n = n // 10
 
print("Сумма:", suma)
print("Произведение:", mult)

Обратим внимание, что заголовок условного оператора if digit != 0: в Python можно сократить до просто if digit:. Потому что 0 — это False. Все остальные числа считаются истиной.

Приведенный выше математический алгоритм нахождения суммы и произведения цифр числа можно назвать классическим, или универсальным. Подобным способом задачу можно решить на всех императивных языках, независимо от богатства их инструментария. Однако средства языка программирования могут позволить решить задачу другим, зачастую более простым, путем. Например, в Python можно не преобразовывать введенную строку к числу, а извлекать из нее отдельные символы, которые преобразовывать к целочисленному типу int:

a = input()
 
suma = 0
mult = 1
 
for digit in a:
    suma += int(digit)
    mult *= int(digit)
 
print("Сумма:", suma)
print("Произведение:", mult)

Если добавить в код проверку, что извлеченный символ строки действительно является цифрой, то программа станет более универсальной. С ее помощью можно будет считать не только сумму и произведение цифр целых чисел, но и вещественных, а также цифр, извлекаемых из произвольной строки.

n = input()
 
suma = 0
mult = 1
 
for digit in n:
    if digit.isdigit():
        suma += int(digit)
        mult *= int(digit)
 
print("Сумма:", suma)
print("Произведение:", mult)

Пример выполнения:

это3 чи3с9ло!
Сумма: 15
Произведение: 81

Строковый метод isdigit проверяет, состоит ли строка только из цифр. В нашем случае роль строки играет одиночный, извлеченный на текущей итерации цикла, символ.

Глубокое знание языка Python позволяет решить задачу более экзотическими способами:

import functools
 
n = input()
n = [int(digit) for digit in n]
 
suma = sum(n)
mult = functools.reduce(lambda x, y: x*y, n)
 
print("Сумма:", suma)
print("Произведение:", mult)

Выражение [int(digit) for digit in n] представляет собой генератор списка. Если была введена строка "234", будет получен список чисел: [2, 3, 4].

Встроенная функция sum считает сумму элементов переданного ей аргумента.

Функция reduce модуля functools принимает два аргумента — лямбда-выражение и в данном случае список. Здесь в переменной x происходит накопление произведения, а y принимает каждое следующее значение списка.

Больше задач в PDF


Открытая Математика. Алгебра. Обыкновенные дроби

Обыкновенные дроби

Можно еще больше расширить числовое множество – так, чтобы операция деления над натуральными числами была выполнима всегда. Для этого введем понятие дроби.

Обыкновенной дробью называется число вида mn, где m и n – натуральные числа. Число m называется числителем этой дроби, а число n – её знаменателем.

Если n = 1, то дробь имеет вид m1, и её часто записывают просто m. Отсюда, в частности, следует, что любое натуральное число представимо в виде обыкновенной дроби со знаменателем 1.

Две дроби ab и cd называются равными, если ad=bc.

Например, 23=812, так как 2ċ12=3ċ8. Из этого определения следует, что дробь ab равна любой дроби вида ambm, где m – натуральное число. В самом деле, так как aċbm=bċam, то ab=ambm.   Итак, мы готовы сформулировать следующее правило.

Если числитель и знаменатель данной дроби умножить или разделить на одно и то же число, неравное нулю, то получится дробь, равная данной.

С помощью основного свойства дроби можно заменить данную дробь другой дробью, равной данной, но с меньшими числителем и знаменателем. Такая замена называется сокращением дроби. Например, 812=46=23 (здесь числитель и знаменатель разделили сначала на 2, а потом ещё на 2). Сокращение дроби можно провести тогда и только тогда, когда её числитель и знаменатель не являются взаимно простыми числами. Если же числитель и знаменатель данной дроби взаимно просты, то дробь сократить нельзя, например, 45 – несократимая дробь.

Сокращение обыкновенных дробей

 

Обыкновенная дробь mn называется правильной, если её числитель меньше её знаменателя, то есть m < n. Обыкновенная дробь называется неправильной, если её числитель больше её знаменателя, то есть m > n.

Справедливо следующее утверждение (его мы докажем ниже):

Всякую неправильную дробь можно представить в виде суммы натурального числа и правильной дроби.

 

Из двух дробей с одинаковыми знаменателями больше та дробь, числитель которой больше. Например, 37<57, 58<68. Из двух дробей с одинаковыми числителями больше та дробь, знаменатель которой меньше. Например, 12>15, 38<34. Чтобы сравнить две дроби с разными числителями и знаменателями, нужно преобразовать обе дроби так, чтобы их знаменатели стали одинаковыми. Такое преобразование называется приведением дробей к общему знаменателю. Сравнение обыкновенных дробей Пусть, например, даны две дроби 34  и  57. Умножим числитель и знаменатель первой дроби на 7, получим 34=2128. Умножим числитель и знаменатель второй дроби на 4, получим 57=2028. Итак, две дроби 34 и 57 приведены к общему знаменателю: 34=2128 и 57=2028.

Теперь знаменатели этих дробей одинаковы, значит, 2128>2028. Следовательно, 34>57. Ясно, что две дроби можно привести не к единственному общему знаменателю. Так, в нашем примере дроби 34  и 57 можно привести к знаменателю 56. В самом деле: 34=3ċ144ċ14=4256, 57=5ċ87ċ8=4056. Понятно, что эти две дроби можно привести к любому знаменателю, делящемуся одновременно на 4 и 7. Однако обычно стараются привести дроби к наименьшему общему знаменателю, который равен наименьшему общему кратному знаменателей двух данных дробей.

Привести дроби к наименьшему общему знаменателю: 1215  и 720.

Найдём сперва наименьшее общее кратное чисел 15 и 20. НОК (15, 20) = 60.

Так как 60 : 15 = 4, то числитель и знаменатель дроби 1215 нужно умножить на 4: 1215=12ċ415ċ4=4860. Поскольку 60 : 20 = 3, то числитель и знаменатель второй дроби нужно умножить на 3: 720=7ċ320ċ3=2160. Итак, дроби приведены к общему знаменателю: 1215=4860 и 720=2160.Ответ. 1215=4860, 720=2160.

В рассмотренном примере числа 4 и 3 называют дополнительными множителями для первой и второй дроби соответственно.

 

Теперь мы можем определить арифметические действия с дробями.

Сложение. Если знаменатели дробей одинаковы, то чтобы сложить эти дроби, нужно сложить их числители; знаменатель остаётся прежним, то есть ab+cb=a+cb. Если знаменатели данных дробей разные, то дроби нужно сначала привести к общему знаменателю, а потом поступить, как описано выше.

Вычитание. Если две дроби имеют одинаковые знаменатели, то ab-cb=a-cb. Если знаменатели данных дробей различны, то сперва приводят дроби к общему знаменателю, а потом вычитают их по вышеприведённой формуле.

Сложение и вычитание обыкновенных дробей

Умножение. Произведение двух дробей равно дроби, числитель которой равен произведению числителей данных дробей, а знаменатель равен произведению их знаменателей, то есть abċcd=acbd. Например, 23ċ47=2ċ43ċ7=821.

Деление. Деление дробей осуществляют следующим образом: ab:cd=adbc. Например, 35:27=3ċ75ċ2=2110.

В случае умножения и деления смешанных чисел всегда удобно переходить к неправильным дробям.

Умножение и деление обыкновенных дробей

Сложить две дроби 37 и 57. Ответ представить в виде неправильной дроби.

Имеем:
37+57=3+57=87.

Ответ. 87.

Сложить две дроби 1215 и 720. Ответ представить в виде неправильной дроби.

Имеем:
1215+720=4860+2160=48+2160=6960.

Ответ. 6960.

Теперь можно показать, что любую неправильную дробь можно представить в виде суммы натурального числа и правильной дроби (или в виде натурального числа, если дробь mn такова, что число m кратно n, например, 123=4).

Представить неправильную дробь в виде суммы натурального числа и правильной дроби: 1) 257;  2) 3712. 

Имеем:
1) 257=21+47=217+47=3+47.

2) 3712=36+112=3612+112=3+112.

Обычно сумму натурального числа и правильной дроби пишут без знака сложения, то есть вместо 3+47 пишут просто 347. Неправильная дробь, записанная в такой форме, называется смешанным числом. Говорят, что целая часть этого числа равна 3, а дробная – 47.

Ответ. 347, 3112.

Всякую неправильную дробь можно представить в виде смешанного числа (или в виде натурального числа). Понятно также, что верно и обратное: всякое смешанное число может быть представлено в виде неправильной дроби. Например, 435=4+35=205+35=20+35=235.

Выполнить действия. 1) 413-234; 2) 323ċ415; 3) 323:415.

Имеем:

  1. 413-234=4+13-(2+34)=4+13-2-34=(4-2)+13-34=2+4-912==2-512=1+1212-512=1+12-512=1+712=1712.
  2. 323ċ415=(3+23)ċ(4+15)=113ċ215=23115=225+615=15615.
  3. 323:415=113:215=113ċ521=5563.

Ответ. 1) 1712; 2) 15615; 3) 5563.





Купить входную дверь
masterdveri74.ru

 

Смотрите также: Математика, Английский язык, Химия, Биология, Физика, География, Астрономия. {30}$, чтобы увидеть, насколько они велики на самом деле. 9м$$

$\endgroup$

1

$\begingroup$

Это не ответ, мне просто нужно показать график. Я забыл пометить ось. Горизонтальный — $n$, а второй — $m$

Надеюсь, вам понравится

$\endgroup$

4

$\begingroup$

Еще одна попытка: 9н$.

$\endgroup$

3

Алгебраические выражения | ChiliMath

Математика, как и любой другой язык, имеет способ передачи идей. Алгебраическое выражение — это краткий способ описания математических объектов с помощью чисел, переменных (букв) и арифметических операций, таких как сложение, вычитание, умножение и деление.

Тремя основными компонентами алгебраических выражений являются числа , переменные и арифметические операции .

  • Числа или константы

Примеры: [латекс]1[/латекс], [латекс]6[/латекс], [латекс]8[/латекс], [латекс]27[/латекс], [латекс] 32[/latex] и т. д.

  • Переменные или буквы

Примеры: [latex]x[/latex], [latex]y[/latex], [latex]a[/latex], [latex]h [/latex], [latex]p[/latex] и т. д.

  • Арифметические операции

Примеры: [latex]+[/latex] (сложение),  [latex] – [/latex] (вычитание) , [латекс] \times[/латекс] (умножение) , [латекс]÷[/латекс] (деление)


Ниже приведены простые примеры, которые помогут вам ознакомиться с операциями сложения, вычитания, умножения и деления.

  • Сложение

сумма [латекс]х[/латекс] и [латекс]5[/латекс] → [латекс]х+5[/латекс]

  • Вычитание 90 020

разность [латекс]у[/латекс] и [латекс]3[/латекс] → [латекс]у-3[/латекс]

  • Умножение

произведение [латекс]n[/латекс] и [латекс]2[/латекс] → [латекс]2n[/латекс]

  • Деление

частное [латекс]k[/латекс] и [латекс]7[/латекс] → [латекс]\большой{{к \более 7}}[/латекс]


Пошаговые примеры написания алгебраических выражений

Давайте рассмотрим больше примеров.

Пример 1: Сумма удвоенного числа и [латекс]3[/латекс]

Ответ: Пусть переменная [латекс]х[/латекс] будет неизвестным числом. Таким образом, удвоение числа означает [латекс]2x[/латекс]. Сумма (используйте символ плюс) удвоенного числа и [латекс]3[/латекс] может быть записана как [латекс]2x+3[/латекс].


Пример 2: Разница тройного числа a и [latex]5[/latex]

Ответ:  Пусть переменная [latex]y[/latex] будет неизвестным числом. Таким образом, тройное число означает [латекс]3г[/латекс]. Разность (используйте символ минус) тройного числа и [латекс]5[/латекс] должна быть записана как [латекс]3у – 5[/латекс].


Пример 3:  Сумма частного [латекс]m[/латекс] и [латекс]2[/латекс] и произведение [латекс]4[/латекс] и [латекс]n[ /латекс].

Ответ:  В этом случае неизвестные номера уже указаны как [latex]m[/latex] и [latex]n[/latex]. Это одним поводом для беспокойства меньше.

Ключ в том, чтобы признать, что мы собираемся добавить частное и произведение.

  • частное [латекс]m[/латекс] и [латекс]2[/латекс] выражается как [латекс]\большой{{м \более 2}}[/латекс]
  • [латекс]4[/латекс] и [латекс]n[/латекс] выражается как [латекс]4n[/латекс]

Таким образом, сумма частного и произведения равна [латекс] {\ большой {{м \ более 2}}} + 4n[/латекс].


Пример 4: Разность произведения [латекс]7[/латекс] и [латекс]w[/латекс] и частного [латекс]2[/латекс] и [латекс]v[ /латекс].

Ответ: В этом случае неизвестным номерам были присвоены соответствующие переменные: [latex]w[/latex] и [latex]v[/latex].

 Ключ в том, чтобы признать, что мы собираемся вычесть произведение на частное некоторых выражений.

  • произведение [латекс]7[/латекс] и [латекс]в[/латекс] выражается как [латекс]7в[/латекс]
  • частное [латекс]2[/латекс] и [latex]v[/latex] выражается как [latex]\Large{{2 \over v}}[/latex]

Следовательно, разница между произведением и частным равна [latex]7w – {\Large{ {2 \over v}}}[/latex].


Общие слова или термины для обозначения сложения, вычитания, умножения и деления

Давайте рассмотрим некоторые распространенные слова или фразы, описывающие четыре арифметических операции. Очень важно, чтобы вы понимали эти слова или фразы, чтобы успешно написать или интерпретировать любое заданное алгебраическое выражение.


Преобразование математических фраз в алгебраические выражения

Ключом к обучению является изучение МНОГО примеров!

90 314 всего числа и 5 9031 4 x − 5
МАТЕМАТИЧЕСКИЕ ФРАЗЫ АЛГЕБРАИЧЕСКИЕ ВЫРАЖЕНИЯ
число плюс 9 у + 9
сумма числа и 10 м + 10
б + 5
число увеличить на 4 х + 4
ч отнять 2 ч − 2
2 убери по номеру 2 − ч
а число минус 11 k − 11
11 минус число 11 − k
число уменьшилось на 7 903 15 y − 7
разница n и 25 n − 25
разница 25 и n 25 − n
5 меньше числа
x меньше числа 5 5 − x
произведение r и 4 4r
7 раз число 7 шт.

Три простых правила относительно квадратного корня. Часть 3

GRE Mathematics уделяет особое внимание заданиям на квадратный корень. В двух предыдущих частях статьи, мы рассматривали, что делать, если все числа в задании положительные. Если же это не так, то следует применять ещё 2 правила GRE Maths.

Правило №2: если x2 = 9, то x = 3, x = -3

Эта ситуация отлична от описанных ранее . Мы больше не имеем знака квадратного корня, зато здесь есть показатель степени. Если 3 возвести в квадрат, то мы получим 9. Если мы возведем -3 в квадрат – мы также получим 9. Следовательно, оба числа являются возможным значением x, потому что оба делают равенство верным.

С математической точки зрения, мы бы сказали, что x = 3 или  x = -3. Если вы выполняете задание в разделе Quantitative Comparison, подумайте об этом следующим образом: если одно из них является возможным значением x, то оба варианта должны быть рассмотрены возможными значениями при сравнении Величины А и Величины В.

Правило №3: √(x)2 = 3, если x = 3, x = -3

Итак, вернемся к знаку квадратного корня, но теперь у нас есть и показатель степени! Что дальше? Указывать только положительное число, потому что мы имеем знак корня? Или указывать оба значения, потому что есть показатель степени?

Сначала вычислите значение x: возведите в степень оба значения √(x)2 = 3, чтобы получить x2 = 9. Вычислите квадратный корень, чтобы получить x = 3, x = -3 (как в правиле №2).

Подставьте оба числа в данное равенство,  √x2 = 3, и посмотрите, делают ли они равенство верным.  Если мы подставим 3 в равенство √x2 = 3, мы получим: √(3)2 = 3. Верно ли это? Да: √(3)2 = √9 и это действительно равняется 3.

Теперь подставьте в равенство -3: √(-3)2= 3. Под корнем у нас стоит отрицательное число, но также в скобках у нас есть квадратная степень. Следуйте установленному порядку действий: возведите число в квадрат, чтобы получить √9. Больше нет никаких отрицательных чисел под знаком корня! Заканчивая решение задачи, мы получаем √9, и снова это должно равняться 3, поэтому -3 тоже является возможным значением x. X может быть равен как 3, так и -3.

Запомните: в первом примере представлено либо действительное число, либо очевидная переменная (не возведение в степень!) под знаком квадратного корня. В обоих случаях мы должны получить решение с положительными значениями  корня, но не отрицательными.

Второй и третий примеры имеют квадратную степень. Во втором правиле нет знака квадратного корня – в этом случае  мы можем получить и положительный, и отрицательный ответ. В нашем третьем правиле есть и знак квадратного корня, и степень в квадрате. В этой ситуации мы должны произвести расчеты, как показано в примере. Сначала мы решаем оба варианта, а затем подставляем их в исходное равенство. Если эти варианты делают равенство верным, то это и есть правильный  ответ.

Подготовка к GRE Test включает в себя штудирование не только официальных учебников, но также изучение советов и подсказок, которые представлены здесь. Возможно, на самом тесте вам пригодятся именно они! Успехов!

Пример несложного задания на квадратные корни в тесте GRE:

 

По материалам сайта: www.manhattanprep.com

3-8 9 Оценить квадратный корень из 12 10 Оценить квадратный корень из 20 11 Оценить квадратный корень из 50 94 18 Оценить квадратный корень из 45 19 Оценить квадратный корень из 32 20 Оценить квадратный корень из 18 92

Квадратный корень из 3 — Как найти квадратный корень из 3?

LearnPracticeDownload

Квадратный корень из 3 выражается как √3 в радикальной форме и как (3) ½ или (3) 0,5 в экспоненциальной форме. Квадратный корень из 3, округленный до 7 знаков после запятой, равен 1,7320508. Это положительное решение уравнения x 2 = 3.

  • Корень квадратный из 3: 1,7320508075688772 909:20
  • Квадратный корень из 3 в экспоненциальной форме: (3) ½ или (3) 0,5
  • Квадратный корень из 3 в подкоренной форме: √3
1. Что такое квадратный корень из 3?
2. Является ли квадратный корень из 3 рациональным или иррациональным?
3. Как найти квадратный корень из 3?
4. Важные примечания 
5. Часто задаваемые вопросы о квадратном корне из 3
6. Сложные вопросы

Что такое квадратный корень из 3?

Квадратный корень из числа — это число, которое при умножении само на себя дает исходное число. Например, квадратный корень из 25 равен 5, так как 5 умножить на 5 дает 25. Однако у вас также могут быть квадратные корни некоторых чисел, которые не дают целых чисел, например 3. Мы можем выразить квадратный корень из 3 по-разному

  • Десятичная форма: 1,732.
  • Радикальная форма: √3
  • Форма экспонента: 3 1/2

Является ли квадратный корень из 3 рациональным или иррациональным?

  • Десятичная часть квадратного корня из 3 не является конечной. Это определение иррационального числа.
  • Глядя на десятичную форму корня 3, мы видим, что она бесконечна —
    √3 = 1,732050807…….
  • Следовательно, мы можем заключить, что Квадратный корень из 3 иррационален

Как найти квадратный корень из 3?

Поскольку мы пришли к выводу, что квадратный корень из 3 не является конечным, мы можем использовать только метод длинного деления для вычисления его значения.

  • Шаг 1: Для начала запишем 3 как 3.000000 и сгруппируем 0 после запятой в пары по 2 слева направо, как показано ниже. (для цифр слева от запятой соединяйте их справа налево) 909:20
  • Шаг 2: Задумайте число, которое при умножении само на себя меньше или равно 3. В этом случае это число будет 1.
  • Шаг 3: Разделив 3 на 1 с частным, равным 1, мы получим остаток 2. 
  • Шаг 4:  Перетащите пару нулей вниз и закрасьте ее рядом с 2 , чтобы получить делимое 200. 
  • Шаг 5: Делитель, который здесь равен 1, добавляется к самому себе и записывается ниже. Теперь у нас есть 2X в качестве нового делителя, и нам нужно найти значение X, которое делает произведение 2X × X меньше или равным 200. В этом случае 27 — это искомое значение 9.09:20
  • Шаг 6: Число 7 ставится в частном после запятой. Новый делитель для следующего деления будет 2X + X, что в данном случае равно 34.
    Действуя таким же образом и повторяя с шага 4, мы можем вычислить остальные десятичные дроби.

Изучение квадратных корней с помощью иллюстраций и интерактивных примеров

  • Квадратный корень из 4
  • Квадратный корень из 2
  • Квадратный корень из 5
  • Квадратный корень из 9
  • Квадратный корень из 15

Важные примечания

  • Действительные корни √3 равны ± 1,732.
  • Квадратный корень из полного квадрата — это всегда рациональное целое число, а корень других чисел всегда иррационален. Например, √16 = 4, а √17 = 4,1231…
  • .

Загадочные вопросы

  • Найдите значение √√3.
  • Какова длина стороны квадрата площадью 10? (Подсказка: используйте метод длинного деления)
  • Найдите квадратный корень из 33.

 

 

  1. Пример 1

    Джон интересовался, совпадает ли значение -√3 с √-3. Что вы думаете?

    Решение

    Отрицательные квадратные корни не могут быть действительными числами.
    -√3 — действительное число.
    Но √-3 — мнимое число.
    Следовательно, они не совпадают, а -√3 не совпадает с √-3.

  2.  

    Пример 2

    Майкл едет по шоссе со средней скоростью 50√3 км/ч ровно 1 час. Какое расстояние он преодолевает?

    Решение

    Нам нужно использовать формулу Расстояние = Скорость * Время
    Скорость = 50√3 = 86,603 км/ч
    Время = 1 час
    Используя формулу, Расстояние = 86,603 * 1 = 86,603

    Следовательно, Майкл преодолевает расстояние 86,603 км

  3. Пример: Если площадь круга равна 3π в 2 . Найдите радиус окружности.

    Решение:

    Пусть ‘r’ будет радиусом окружности.
    ⇒ Площадь круга = πr 2 = 3π в 2
    ⇒ г = ±√3 в
    Так как радиус не может быть отрицательным,
    ⇒ г = √3
    Квадратный корень из 3 равен 1,732.
    ⇒ г = 1,732 в

перейти к слайдуперейти к слайдуперейти к слайду

 

Хотите создать прочную основу для изучения математики?

Выйдите за рамки заучивания формул и поймите «почему», стоящее за ними. Испытайте Cuemath и приступайте к работе.

Забронируйте бесплатный пробный урок

Часто задаваемые вопросы о квадратном корне из 3

Каково значение квадратного корня из 3?

Квадратный корень из 3 равен 1,73205.

Почему квадратный корень из 3 является иррациональным числом?

Число 3 простое. Отсюда следует, что число 3 беспарное и не находится в степени двойки. Следовательно, квадратный корень из 3 иррационален.

Если квадратный корень из 3 равен 1,732. Найдите значение квадратного корня из 0,03.

Представим √0,03 в форме p/q, т.е. √(3/100) = 0,03/10 = 0,173. Следовательно, значение √0,03 = 0,173

Вычислить 14 плюс 16 квадратный корень 3

Данное выражение равно 14 + 16 √3.

Ом умножить на ампер: The page cannot be found

Перевести омы (Ω) в амперы (А): онлайн-калькулятор, формула

Sign in

Password recovery

Восстановите свой пароль

Ваш адрес электронной почты

Инструкция по использованию: Чтобы перевести омы (Ω) в амперы (А), введите сопротивление R в омах (Ω), напряжение U в вольтах (В) или мощность P в ваттах (Вт), затем нажмите кнопку “Рассчитать”. Таким образом будет получено значение силы тока I в амперах (А).

  • Калькулятор Ом в А (через вольты)
  • Калькулятор Ом в А (через ватты)

Калькулятор Ом в А (через вольты)

Формула для перевода Ом в А

Сила тока I в амперах (А) равняется напряжению U в вольтах (В), деленному на сопротивление R в омах (Ω).

Калькулятор Ом в А (через ватты)

Формула для перевода Ом в А

Сила тока I в амперах (А) равняется квадратному корню из результата деления мощности P в ваттах (Вт) на сопротивление R в омах (Ω).

ЧАЩЕ ВСЕГО ЗАПРАШИВАЮТ

Таблица знаков зодиака

Нахождение площади трапеции: формула и примеры

Нахождение длины окружности: формула и задачи

Римские цифры: таблицы

Таблица синусов

Тригонометрическая функция: Тангенс угла (tg)

Нахождение площади ромба: формула и примеры

Нахождение объема цилиндра: формула и задачи

Тригонометрическая функция: Синус угла (sin)

Геометрическая фигура: треугольник

Нахождение объема шара: формула и задачи

Тригонометрическая функция: Косинус угла (cos)

Нахождение объема конуса: формула и задачи

Таблица сложения чисел

Нахождение площади квадрата: формула и примеры

Что такое тетраэдр: определение, виды, формулы площади и объема

Нахождение объема пирамиды: формула и задачи

Признаки подобия треугольников

Нахождение периметра прямоугольника: формула и задачи

Формула Герона для треугольника

Что такое средняя линия треугольника

Нахождение площади треугольника: формула и примеры

Нахождение площади поверхности конуса: формула и задачи

Что такое прямоугольник: определение, свойства, признаки, формулы

Разность кубов: формула и примеры

Степени натуральных чисел

Нахождение площади правильного шестиугольника: формула и примеры

Тригонометрические значения углов: sin, cos, tg, ctg

Нахождение периметра квадрата: формула и задачи

Теорема Фалеса: формулировка и пример решения задачи

Сумма кубов: формула и примеры

Нахождение объема куба: формула и задачи

Куб разности: формула и примеры

Нахождение площади шарового сегмента

Что такое окружность: определение, свойства, формулы

Конвертер величин / Калькулятор единиц измерения

Изначальное значение:

Калькулятуру классических единиц измерения:

Категории измерений:Активность катализатораБайт / Битвес ткани (текстиль)ВремяВыбросы CO2Громкость звукаДавлениеДинамическая вязкостьДлина / РасстояниеЁмкостьИмпульсИндуктивностьИнтенсивность светаКинематическая вязкостьКоличество веществакоэффициент теплопередачи (U-value)Кулинария / РецептыМагнитный потокмагнитодвижущая силаМасса / ВесМассовый расходМолярная концентрацияМолярная массаМолярная теплоёмкостьМолярный объемМомент импульсаМомент силыМощностьМощностью эквивалентной дозыМузыкальный интервалНапряжённость магнитного поляНефтяной эквивалентОбъёмОбъёмная теплоёмкостьОбъёмный расход жидкостиОбъемный тепловой потокОсвещенностьПлоский уголПлотностьПлотность магнитного потокаПлощадьПоверхностное натяжениеПоглощённая дозаПриставки СИпроизведение дозы на длинупроизведения дозы на площадьПроизводительность компьютера (флопс)Производительность компьютера (IPS)РадиоактивностьРазмер шрифта (CSS)Световая энергияСветовой потокСилаСистемы исчисленияСкоростьСкорость вращенияСкорость передачи данныхСкорость утечкиСопротивление теплопередаче (значение R)Текстильные измеренияТелесный уголТемператураТепловой потокТеплоемкостьТеплопроводностьУдельная теплоёмкостьУскорениеЧастей в . ..ЧастотаЭквивалентная дозаЭкспозиционная дозаЭлектрическая эластичностьЭлектрический дипольный моментЭлектрический зарядЭлектрический токЭлектрическое напряжениеЭлектрическое сопротивлениеЭлектрической проводимостиЭнергияЯркостьFuel consumption   

Изначальное значение:

Изначальная единица измерения:Ангстрем [Å]Астрономическая единица [AU]аттометр [ам]гектометр [гм]Гигаметр [Гм]декаметр [дам]дециметр [дм]Дюйм [in]Икс-единица — СигбанКабельтовКвартеркилометр [км]ЛинкЛокоть (британский)Мегаметр [Мм]Метр [м]Метрическая милямикрометр [мкм]миллиметр [мм]Миль — тыcячМиля (международная) [mi]Миля (США)Морская миляМорская саженьнанометр [нм]Парсек [pc]Перчпикометр [пм]Планковская длинаПольРимская миляРодсантиметр [см]Световые годыСветовые дниСветовые минутыСветовые секундыСветовые часыСтатутная миляТвипфемтометр [фм]ФурлонгФут [ft]Чейн [ch]Ярд

Требуемая единица измерения:Ангстрем [Å]Астрономическая единица [AU]аттометр [ам]гектометр [гм]Гигаметр [Гм]декаметр [дам]дециметр [дм]Дюйм [in]Икс-единица — СигбанКабельтовКвартеркилометр [км]ЛинкЛокоть (британский)Мегаметр [Мм]Метр [м]Метрическая милямикрометр [мкм]миллиметр [мм]Миль — тыcячМиля (международная) [mi]Миля (США)Морская миляМорская саженьнанометр [нм]Парсек [pc]Перчпикометр [пм]Планковская длинаПольРимская миляРодсантиметр [см]Световые годыСветовые дниСветовые минутыСветовые секундыСветовые часыСтатутная миляТвипфемтометр [фм]ФурлонгФут [ft]Чейн [ch]Ярд

Перевод единиц измерения никак нельзя назвать банальной задачей: Миллиметр, сантиметр, дециметр, метр, километр, миля, морская миля, фут, ярд, дюйм, локоть, парсек и световой год. С помощью этих измерений могут быть рассчитаны расстояния. И это далеко не все возможные измерения, а лишь наиболее распространенные из них. В случаях измерений площади (квадратный метр, квадратный километр, ар, гектар, морган, акр и другие), температуры (в градусах по Цельсию, по Кельвину, по Фаренгейту), скорости (м/с, км/час, миль/ч, узлы, мах), веса (центнер, килограмм, метрическая тонна, американская тонна, стандартная тонна, фунт и другие) и объема (кубический метр, гектолитр, английский галлон жидкости, американский жидкий галлон, американский сухой галлон, баррель и другие) ситуация не намного лучше. А если всего этого вам показалось мало — большинство из этих единиц также имеют подразделения и высшие единицы (например, милли-, санти-, деци-). Короче говоря, хаос, в котором так трудно разобраться без помощи справочника или других средств. Данный калькулятор единиц измерения идеально подходит для перевода данных единиц.

Калькулятор-конвертор для единиц измерения. Способен преобразовать огромное количество единиц измерения.

Что такое закон Ома? | Fluke

Закон Ома — это формула, используемая для расчета соотношения между напряжением, током и сопротивлением в электрической цепи.

Для студентов, изучающих электронику, закон Ома (E = IR) так же важен, как уравнение относительности Эйнштейна (E = mc²) для физиков.

E = I x R

При расшифровке это означает напряжение = ток x сопротивление , или вольт = ампер x ом , или В = А x Ом .

Названный в честь немецкого физика Георга Ома (1789-1854), закон Ома касается ключевых величин, действующих в цепях:
(аббревиатура) Роль в схемы Если вам интересно: Напряжение E Вольт (В) Давление, запускающее поток электронов E = электро движущая сила (термин старой школы) Ток I Ампер, ампер (А) Скорость потока электронов I = интенсивность 90 039 Сопротивление R Ом (Ом) Блокировка потока Ω = греческая буква омега

Если два из этих значений известны, технические специалисты могут изменить закон Ома для расчета третьего. Просто измените пирамиду следующим образом:

Если вы знаете напряжение (E) и силу тока (I) и хотите знать сопротивление (R), уменьшите X R в пирамиде и рассчитайте оставшееся уравнение (см. первое или последнее уравнение). слева, пирамида вверху).

Примечание: Сопротивление нельзя измерить в работающей цепи, поэтому закон Ома особенно полезен, когда его необходимо рассчитать. Вместо того, чтобы отключать цепь для измерения сопротивления, технический специалист может определить R, используя приведенный выше вариант закона Ома.

Теперь, если вы знаете напряжение (E) и сопротивление (R) и хотите узнать ток (I), вычеркните X из I и вычислите оставшиеся два символа (см. среднюю пирамиду выше).

А если вы знаете ток (I) и сопротивление (R) и хотите знать напряжение (E), умножьте нижние половины пирамиды (см. третью, или крайнюю правую, пирамиду вверху).

Попробуйте выполнить несколько расчетов на основе простой последовательной цепи, включающей только один источник напряжения (батарея) и сопротивление (свет). В каждом примере известны два значения. Используйте закон Ома, чтобы вычислить третий.

Пример 1: Напряжение (E) и сопротивление (R) известны.

Какой ток в цепи?

I = E/R = 12 В/6 Ом = 2 А

Пример 2: Напряжение (E) и ток (I) известны.

Какое сопротивление создает лампа?

R = E/I = 24 В/6 А = 4 Ом

Пример 3: Ток (I) и сопротивление (R) известны. Какое напряжение?

Какое напряжение в цепи?

E = I x R = (5A)(8Ω) = 40 В

Когда Ом опубликовал свою формулу в 1827 году, его ключевой вывод заключался в том, что количество электрического тока, протекающего через проводник, равно прямо пропорционально приложенному к нему напряжению. Другими словами, требуется один вольт давления, чтобы протолкнуть один ампер тока через сопротивление в один ом.

Что проверять с помощью закона Ома

Закон Ома можно использовать для проверки статических значений компонентов схемы, уровней тока, источников напряжения и падения напряжения. Если, например, контрольно-измерительный прибор обнаруживает измеренный ток выше нормального, это может означать, что сопротивление уменьшилось или напряжение увеличилось, что привело к возникновению ситуации с высоким напряжением. Это может указывать на проблему с питанием или цепью.

В цепях постоянного тока (постоянного тока) измерение тока ниже нормального может означать, что напряжение уменьшилось или сопротивление цепи увеличилось. Возможными причинами повышенного сопротивления являются плохие или ослабленные соединения, коррозия и/или поврежденные компоненты.

Нагрузки в цепи потребляют электрический ток. Нагрузками могут быть любые компоненты: небольшие электрические устройства, компьютеры, бытовая техника или большой двигатель. К большинству этих компонентов (нагрузок) прикреплена заводская табличка или информационная наклейка. Эти паспортные таблички содержат сертификаты безопасности и несколько идентификационных номеров.

Технические специалисты обращаются к шильдикам компонентов, чтобы узнать стандартные значения напряжения и силы тока. Если во время тестирования техники обнаруживают, что обычные значения не регистрируются на их цифровых мультиметрах или токоизмерительных клещах, они могут использовать закон Ома, чтобы определить, какая часть цепи дает сбой, и исходя из этого определить, в чем может заключаться проблема.

Основы науки о цепях

Цепи, как и вся материя, состоят из атомов. Атомы состоят из субатомных частиц:

  • Протоны (с положительным электрическим зарядом)
  • Нейтроны (бесзарядные)
  • Электроны (отрицательно заряженные)

Атомы остаются связанными силами притяжения между ядром атома и электронами в его внешней оболочке. Под влиянием напряжения атомы в цепи начинают реформироваться, и их компоненты проявляют потенциал притяжения, известный как разность потенциалов. Взаимно притягивающиеся свободные электроны движутся навстречу протонам, создавая поток электронов (ток). Любой материал в цепи, который ограничивает этот поток, считается сопротивлением.

Ссылка: Принципы цифрового мультиметра Глена А. Мазура, American Technical Publishers.

Связанные статьи

  • Устранение неполадок неисправных двигателей с помощью проверки сопротивления изоляции
  • Безопасность электрических испытаний – подготовка к проверке без напряжения
Электрический блок

Вт (Вт)

  • Определение мощности
  • Калькулятор преобразования ватт в мВт, кВт, МВт, ГВт, дБм, дБВт
  • Таблица префиксов единиц измерения ватт
  • Как перевести ватты в киловатты
  • Как преобразовать ватты в милливатт
  • Как преобразовать ватты в дБм
  • Как преобразовать ватты в ампер
  • Как преобразовать ватты в вольты
  • Как преобразовать ватты в омы
  • Как перевести ватты в БТЕ
  • Как перевести ватты в джоули
  • Как преобразовать ватты в лошадиные силы
  • Как перевести ватты в кВА
  • Как преобразовать ватты в ВА
  • Таблица потребляемой мощности

Определение мощности

Ватт — единица мощности (обозначение: Вт).

Единица ватт названа в честь Джеймса Уатта, изобретателя паровой машины.

Один ватт определяется как скорость потребления энергии в один джоуль в секунду.

1 Вт = 1 Дж / 1 с

Один ватт также определяется как ток в один ампер при напряжении в один вольт.

1 Вт = 1 В × 1 А

Калькулятор преобразования ватт в мВт, кВт, МВт, ГВт, дБм, дБВт

Преобразование ватт в милливатт, киловатт, мегаватт, гигаватт, дБм, дБВт.

Введите мощность в одно из текстовых полей и нажмите кнопку Convert :

Введите милливатт: мВт
Введите мощность: Вт
Введите киловатты: кВт
Введите мегаватты: МВт
Введите гигаватт: ГВт
Введите дБм: дБм
Введите дБВт: дБВт
     

Таблица префиксов единиц измерения ватт

наименование символ преобразование пример
пиковатт пВт 1 пВт = 10 -12 Вт P = 10 пВт
нановатт нВт 1 нВт = 10 -9 Вт P = 10 нВт
микроватт мкВт 1 мкВт = 10 -6 Вт P = 10 мкВт
милливатт мВт 1 мВт = 10 -3 Вт P = 10 мВт
Вт Вт P = 10 Вт
киловатт кВт 1 кВт = 10 3 Вт P = 2 кВт
мегаватт МВт 1 МВт = 10 6 Вт P = 5 МВт
гигаватт ГВт 1 ГВт = 10 9 Вт P = 5 ГВт

Как преобразовать ватты в киловатты

Мощность P в киловаттах (кВт) равна мощности P в ваттах (Вт), деленной на 1000:

P (кВт) = P (Вт) / 1000

Как перевести ватты в милливатт

Мощность P в милливаттах (мВт) равна к мощности P в ваттах (Вт) умножить на 1000:

P (мВт) = P (Вт) ⋅ 1000

Как преобразовать ватты в дБм

Мощность P в децибел-милливаттах (дБм) равно 10-кратному основанию 10 логарифм мощности P в милливаттах (мВт), деленный на 1 милливатт:

P (дБм) = 10 ⋅ log 10 ( P (мВт) / 1 мВт)

Как преобразовать ватты в ампер s

Ток I в амперах (А) равен мощность P в ваттах (Вт), деленная на напряжение V в вольтах (В):

I (А) = P (Вт) / В (В) 9 0005

Как для преобразования ватт в вольты

Напряжение V в вольтах (В) равно мощности P в ваттах (Вт), деленной на силу тока I в амперах (А):

В (В) = P (Вт) / I (А)

Как перевести ватты в омы 90 409

R (Ом) = P (Вт) / I (А) 2

R (Ом) = В (В) 2 / П (Ш)

Как преобразовать ватты в БТЕ/ч

P (БТЕ/ч) = 3,412142 ⋅ P (Вт)

Как перевести ватты в джоули

E (Дж) = P 902 16 (Ш) т (с)

Как перевести ватты в лошадиные силы

P (л. с.) = P (Вт) / 746

Как перевести ватты в кВА

Реальная мощность P in ватт (Вт) равно 1000 умножить на полную мощность S в киловольт-амперах (кВА) умножить на коэффициент мощности (PF) или косинус фазового угла φ:

P (Вт) = 1000 ⋅ S (кВА) PF = 1000 ⋅ S (кВА) 9041 6 ⋅ cos φ

Как преобразовать ватты в ВА

реальная мощность P в ваттах (Вт) равна полной мощности S в вольт-амперах (ВА), умноженной на коэффициент мощности (PF) или косинус фазового угла φ:

P (Вт) = S (ВА) PF = S (ВА) ⋅ cos φ

Потребляемая мощность некоторых электрических компонентов

Сколько ватт потребляет дом? Сколько ватт потребляет телевизор? Сколько ватт потребляет холодильник?

Электрический компонент Типичная потребляемая мощность в ваттах
ЖК-телевизор 30.

Углы и стороны треугольника: Стороны и углы треугольника

Углы и стороны в треугольниках

85. Построим равнобедренный ∆ABC (чер. 92), у которого AB = BC. Тогда мы знаем, что его углы при основании равны, т. е. ∠A = ∠C. Пронумеруем эти углы, – тогда ∠1 = ∠2. Станем теперь строить новые треугольники ABC’, ABC» и т. д. так, чтобы сторона AB и ∠B оставались неизменными, но сторона BC увеличивалась. Тогда угол A должен увеличиваться (что очевидно), а угол C станет уменьшаться: мы видим, что ∠3 < ∠2, ∠4 < ∠3 и т. д., потому что ∠2 есть внешний угол для ∆ACC’ и, следов., ∠2 > ∠3 или ∠3 < ∠2, также ∠3 есть внешний угол ∆AC’C» и, след., ∠3 > ∠4 или ∠4 < ∠3 и т. д. (уменьшение угла C видно еще из того, что сумма углов треугольника всегда равна 2d: угол B не изменяется, угол A увеличивается, – след., ∠C должен уменьшаться).

Из этих построений мы вправе сделать заключения:

1) Если в треугольнике две стороны равны, то против них лежат равные углы.

2) Если в треугольнике две стороны не равны, то против большей из них лежит и больший угол.

86. Теперь, наоборот, построим: 1) треугольник с двумя равными углами и 2) треугольник с двумя неравными углами и сравним стороны, противолежащие этим углам. Для решения вопросов, здесь возникающих, воспользуемся способом рассуждения, часто употребляемым в математике.

1) Пусть ∆ABC (чер. 93) постоен так, что ∠A = ∠C. Сравнить стороны BC и BA.

Пока, не зная ничего про стороны AB и BC, мы можем сделать об них 3 предположения: 1) AB = BC, 2) AB > BC и 3) AB < BC – иных предположений быть не может. Рассматривая эти предположения, мы можем заметить, что два из них не годятся, так как противоречат предыдущему. В самом деле, 2-е предположение, что AB > BC должно, на основании предыдущего п., повлечь за собою следствие, что ∠C > ∠A, а у нас построено: ∠C = ∠A. Следовательно, это предположение должно быть вычеркнуто. Также из 3-го предположения, что AB < BC следует, что ∠A > ∠C, что также противоречит нашему построению. Следовательно, и это предположение должно быть вычеркнуто. Остается поэтому лишь одно предположение, что AB = BC, которое и должно быть верно, так как иных сделать нельзя. Поэтому имеем:

Если в треугольнике два угла равны, то против них лежат равные стороны.

2) Пусть ∆ABC (чер. 93) постоен так, что ∠A > ∠C. Сравнить стороны BA и BC.
Опять мы можем сделать 3 предположения: 1) AB = BC, 2) AB > BC и 3) AB < BC. Теперь видим, что первое предположение не годится, так как на основании п. 85 из него вытекало бы, что ∠C = ∠A, что противоречит нашему построению; также найдем, что 2-е предположение, что AB > BC не годится, так как из него вытекало бы, что ∠C > ∠A, что противоречит построению. Остается лишь 3-е предположение, что AB < BC, которое и должно быть верно. Поэтому имеем:

Если два угла в треугольнике неравны, то против большего из них лежит большая сторона.

Теперь легко решаются вопросы: 1) какая из сторон прямоугольного треугольника самая большая? 2) Какая из сторон тупоугольного треугольника самая большая?

87. В двух предыдущих пп. мы имели дело с двумя положениями: 1) против большей стороны лежит больший угол и 2) против большего угла лежит большая сторона. Мы нашли, что эти мысли справедливы для одного треугольника. Возникает вопрос, справедливы ли они для двух треугольников? Несомненно справедливы для двух равных треугольников, так как равные треугольники можно наложением слить в один треугольник. Но, вообще говоря, к двум различным (не равным) треугольникам эти положения не могут быть применимы: мы можем построить два таких треугольника ∆ABC и ∆A’B’C’ (чер. 94), чтобы ∠B был > ∠B’, но AC была бы < A’C’. Чертеж пояснений не требует. Но есть один случай, когда указанные мысли применимы и к двум различным треугольникам).

Этот случай легко уясняется наглядно. Возьмем две палочки AB и BC (чер. 95 bis) и сложим их концами (в точке B). Если вращать палочку BC около точки B по стрелке, то ∠B станет увеличиваться: сторона BC будет менять свое положение (пусть одно из них есть BC’), но все время BC останется равным самому себе; не изменяется также и отрезок AB. Обратим внимание, что точками A и C определяется еще отрезок AC, на чертеже не изображенный. При вышеуказанном вращении точка C меняет свое место и нам ясно, что этот отрезок AC, не изображенный на чертеже, должен увеличиваться (напр. AC’ > AC), т. е., если 2 стороны треугольника не изменяются, а угол между ними увеличивается, то третья сторона так же увеличивается. В тексте этот случай выяснен без помощи такого наглядного представления.

Построим два таких треугольника, чтобы у них было по две равных стороны, но чтобы углы между ними не были равны. Пусть в ∆ABC и в ∆A’B’C’ (чер. 95) имеем AB = A’B’, BC = B’C’, но ∠B > ∠B’. Сравним стороны AC и A’C’, лежащие против неравных углов. Для этого наложим ∆A’B’C’ на ∆ABC так, чтобы сторона A’B’ совпала с равною ей стороною AB. Тогда сторона B’C’ должна пойти внутри ∠B, потому что ∠B’ < ∠B, но где кончится эта сторона, т. е., где расположится точка C’, неизвестно. Может быть, она расположится как раз на стороне AC, может быть, расположится вне ∆ABC, а может быть внутри этого треугольника. Разберем эти три случая отдельно.

1) Пусть ∆A’B’C’ расположится так, что займет положение ∆ABD (чер. 96), так что точка C’ попадет в D, на сторону AC; тогда, очевидно, AD < AC или A’C’ < AC (AD есть та же сторона A’C’, только перенесенная в другое место).

2) Пусть ∆A’B’C’ при наложении займет положение ∆ABD (чер. 97), т. е. точка C’ расположится в точке D, вне ∆ABC. Тогда, соединив точки C и D, получим ∆BCD, у которого BC = BD, так как, по построению, B’C’ = BC, а BD есть та же сторона B’C’, лишь перенесенная в другое место. Поэтому ∆BCD – равнобедренный, и ∠BCD = ∠BDC. Рассмотрим теперь ∆ACD; про два его угла, а именно про ∠C (или ∠ACD) и про ∠D (или ∠ADC) легко сообразить, пользуясь отмеченными равными углами равнобедренного треугольника, какой из них больше другого. В самом деле, мы видим, что ∠ACD < отмеченного угла BCD при основании равнобедренного треугольника, а ∠ADC > отмеченного угла BDC при основании равнобедренного треугольника. Но ∠BCD = ∠BDC, следовательно, ∠ADC > ∠ACD. Поэтому на основании п. 86 (применяя его к ∆ACD) имеем AC > AD, но AD есть сторона A’C’, перенесенная в другое место, – следовательно, AC > A’C’.

3) Пусть ∆A’B’C’ при наложении займет положение ∆ABD (чер. 98), т. е. точка C’ расположится внутри ∆ABC. Тогда, соединив точки C и D, получим равнобедренный ∆CBD (BD = BC, ибо BD есть сторона B’C’, перенесенная в другое положение, а B’C’ = BC по построению) и, следовательно, ∠BCD = ∠BDC. Если продолжить стороны BD и BC по направлениям DM и CN, то получим два внешних угла этого равнобедренного треугольника ∠MDC и ∠NCD, но ∠MDC = ∠NCD, следовательно, ∠ADC > ∠ACD, а поэтому, на основании п. 86, имеем AC > AD, или AC > A’C’ (AD есть сторона A’C’, перенаправленная в другое положение).

Итак, во всех трех случаях оказалось, что

AC > A’C’,

т. е., если две стороны одного треугольника соответственно равны двум сторонам другого треугольника, но углы между ними не равны, то против большого угла лежит большая сторона.

88. Разберем обратный вопрос. Пусть построены ∆ABC и ∆A’B’C’ (чер. 95) так, что AB = A’B’, BC = B’C’, но AC > A’C’, т. е. два треугольника имеют по две равных стороны, но третьи стороны их не равны. Сравнить ∠B и ∠B’.
Воспользуемся тем же способом рассуждения, как в п. 86.

Пока мы можем об углах B и B’ сделать три предположения: 1) ∠B = ∠B’, 2) ∠B > ∠B’ и 3) ∠B < ∠B’.

Первое предположение не годится, так как тогда наши треугольники, имея по построению по две равных стороны и равные углы между ними, были бы равны, и, следовательно, AC’ = A’C’, а это противоречит построению. Третье предположение, что ∠B < ∠B’ также не годится, так как тогда к этим треугольникам был бы применим результат, найденный в предыдущем п., на основании которого имели бы AC < A’C’, что также противоречит построению. Остается второе предположение, что ∠B > ∠B’, которое и должно быть верно. Итак:

Если две стороны одного треугольника равны соответственно двум сторонам другого, но третьи стороны этих треугольников не равны, то против большей стороны лежит и больший угол.

Подготовка школьников к ЕГЭ и ОГЭ (Справочник по математике — Планиметрия

Поиск по сайту:

Справочник по математикеГеометрия (Планиметрия)Треугольники
ФигураРисунокФормулировка
Треугольник

Рассматриваются три точки, не лежащие на одной прямой, и три отрезка, соединяющие эти точки.

Треугольником называют часть плоскости, ограниченную этими отрезками, отрезки называют сторонами треугольника, а концы отрезков – вершинами треугольника.

Большая сторона треугольникаПротив большей стороны треугольника лежит больший угол
Больший угол треугольникаПротив большего угла треугольника лежит большая сторона
Меньшая сторона треугольникаПротив меньшей стороны треугольника лежит меньший угол
Меньший угол треугольникаПротив меньшего угла треугольника лежит меньшая сторона
Длины сторон треугольника

Длины сторон треугольника удовлетворяют неравенству треугольника: длина любой стороны треугольника меньше суммы длин двух других сторон.

a < b + c

Длины сторон треугольника удовлетворяют неравенству треугольника: длина любой стороны треугольника больше модуля разности длин двух других сторон.

a > |b – c|

Углы треугольника

Сумма углов треугольника равна 180°

Посмотреть доказательство

Внешний угол треугольника

Внешний угол треугольника равен сумме двух внутренних углов треугольника, не смежных с ним.

δ = α + β .

Посмотреть доказательство

Больший угол треугольника

Величина большего угла треугольника не может быть меньшей, чем 60°.

,

где α – больший угол треугольника.

Меньший угол треугольника

Величина меньшего угла треугольника не может быть большей, чем 60°.

,

где β – меньший угол треугольника.

Теорема косинусов

a2 = b 2 + c 2 – 2bc cos α ,

Посмотреть доказательство

Теорема синусов

,

где R – радиус описанной окружности.

Посмотреть доказательство

Треугольник

Рассматриваются три точки, не лежащие на одной прямой, и три отрезка, соединяющие эти точки.

Определение. Треугольником называют часть плоскости, ограниченную этими отрезками, отрезки называют сторонами треугольника, а концы отрезков – вершинами треугольника.

Большая сторона треугольника
Против большей стороны треугольника лежит больший угол
Больший угол треугольника
Против большего угла треугольника лежит большая сторона
Меньшая сторона треугольника
Против меньшей стороны треугольника лежит меньший угол
Меньший угол треугольника
Против меньшего угла треугольника лежит меньшая сторона
Длины сторон треугольника

Длины сторон треугольника удовлетворяют неравенству треугольника: длина любой стороны треугольника меньше суммы длин двух других сторон.

a < b + c

Длины сторон треугольника удовлетворяют неравенству треугольника: длина любой стороны треугольника больше модуля разности длин двух других сторон.

a > |b – c|

Углы треугольника

Сумма углов треугольника равна 180°

Посмотреть доказательство

Внешний угол треугольника

Внешний угол треугольника равен сумме двух внутренних углов треугольника, не смежных с ним.

δ = α + β .

Посмотреть доказательство

Больший угол треугольника

Величина большего угла треугольника не может быть меньшей, чем 60°.

,

где α – больший угол треугольника.

Меньший угол треугольника

Величина меньшего угла треугольника не может быть большей, чем 60°.

,

где β – меньший угол треугольника.

Теорема косинусов

a2 = b 2 + c 2 – 2bc cos α ,

Посмотреть доказательство

Теорема синусов

,

где R – радиус описанной окружности.

Посмотреть доказательство

Треугольник

Рассматриваются три точки, не лежащие на одной прямой, и три отрезка, соединяющие эти точки.

Определение. Треугольником называют часть плоскости, ограниченную этими отрезками, отрезки называют сторонами треугольника, а концы отрезков – вершинами треугольника.

Большая сторона треугольника

Свойство большей стороны треугольника:

Против большей стороны треугольника лежит больший угол

Больший угол треугольника

Свойство большего угла треугольника:

Против большего угла треугольника лежит большая сторона

Меньшая сторона треугольника

Свойство меньшей стороны треугольника:

Против меньшей стороны треугольника лежит меньший угол

Меньший угол треугольника

Свойство меньшего угла треугольника:

Против меньшего угла треугольника лежит меньшая сторона

Длины сторон треугольника

Неравенства трегольника:

Длины сторон треугольника удовлетворяют неравенству треугольника: длина любой стороны треугольника меньше суммы длин двух других сторон.

a < b + c

Длины сторон треугольника удовлетворяют неравенству треугольника: длина любой стороны треугольника больше модуля разности длин двух других сторон.

a > |b – c|

Углы треугольника

Свойство углов треугольника:

Сумма углов треугольника равна 180°

Посмотреть доказательство

Внешний угол треугольника

Свойство внешнего угла треугольника:

Внешний угол треугольника равен сумме двух внутренних углов треугольника, не смежных с ним.

δ = α + β .

Посмотреть доказательство

Больший угол треугольника

Свойство большего угла треугольника:

Величина большего угла треугольника не может быть меньшей, чем 60°.

,

где α – больший угол треугольника.

Меньший угол треугольника

Свойство меньшего угла треугольника:

Величина меньшего угла треугольника не может быть большей, чем 60°.

,

где β – меньший угол треугольника.

Теорема косинусов

Теорема косинусов:

a2 = b 2 + c 2 – 2bc cos α ,

Посмотреть доказательство

Теорема синусов

Свойство меньшего угла треугольника:

,

где R – радиус описанной окружности.

Посмотреть доказательство

      На сайте можно также ознакомиться с нашими учебными материалами для подготовки к ЕГЭ и ОГЭ по математике.

До ЕГЭ по математике осталось
днейчасовминутсекунд


НАШИ ПАРТНЕРЫ
  • «НПО Астек»
  • «Fastvideo»
  • Бюро переводов «Медтран»
  • Независимый бизнес-консультант Е. Самаров

       

Правила треугольника — стороны, углы, внешние углы, градусы и другие свойства

Калькулятор треугольника (Этот бесплатный онлайн-инструмент позволяет вычислить все измерения сторон и углов на основе введенных вами данных и нарисовать бесплатно загружаемое изображение вашего треугольника!)

Рабочие листы треугольника 90 $$


Правило 2: Стороны треугольника — Теорема о неравенстве треугольника: Эта теорема утверждает, что сумма длин любых двух сторон треугольника должна быть больше третьей стороны. )


Правило 3: Соотношение между измерением сторон и углов в треугольнике: Наибольший внутренний угол и сторона лежат друг против друга. Это же правило относится и к наименьшему размеру. угол и сторона, а средний угол и сторона.


Правило 4 Удаленные внешние углы. Эта теорема утверждает, что мера внешнего угла $$ \angle A$$ равна сумме измерений удаленных внутренних углов. еще)

Чем отличаются внутренние и внешние углы треугольника?

На этот вопрос отвечает картинка ниже. Вы создаете внешний угол, продолжая любую сторону треугольника.

9{\circ} $$.

Чтобы проверить истинность этого правила, попробуйте интерактивный треугольник Math Warehouse, который позволяет вам перетаскивать разные стороны треугольника и исследовать взаимосвязь между углами и стороны. Независимо от того, как вы расположите три стороны треугольника, сумма градусов всех внутренние углы (три угла внутри треугольника) всегда равны 180°.

Это свойство внутренних углов треугольника является просто частным примером общее правило для внутренних углов любого многоугольника.

Внутренние углы треугольника (рабочий лист)

Интерактивная демонстрация внутренних углов

∠ А
∠ Б
∠ С
Итого 180

Перетащите точки треугольника, чтобы начать демонстрацию

Практические задачи (правило внутренних углов)

Проблема 1

Чему равен m$$\angle$$LNM в приведенном ниже треугольнике?

$$ \угол $$ LMN = 34°
$$ \угол $$ MLN = 29°

Используйте правило для внутренних углов треугольника:

м$$ \угол $$ LNM +m$$ \угол $$ LMN +m$$ \угол $$ MLN =180°
м$$ \угол $$ LNM +34° + 29° =180°
м$ $ \угол $$ LNM +63° =180°
м$$ \угол $$ LNM = 180° — 63° = 117°

Проблема 2

Внутренние углы треугольника равны $$ \angle $$ HOP, $$ \angle $$ HPO и $$ \angle $$ PHO. $$ \angle $$ HOP равен 64°, а m$$ \angle $$ HPO равен 26°.
Что такое m$$ \angle $$ PHO?

Использовать внутренние углы правила треугольника:

м$$ \угол $$ PHO = 180° — 26° -64° = 90°

Взаимосвязь — Измерения сторон и углов

В любом треугольнике

  • наибольший внутренний угол равен против наибольшей стороны
  • наименьший внутренний угол против наименьшая сторона
  • внутренний угол среднего размера равен напротив средняя сторона

Чтобы проверить истинность утверждений, вы можете использовать интерактивный треугольник Math Warehouse, который позволяет вам перетаскивать разные стороны треугольника и исследовать отношения между мерами углов и стороны. Независимо от того, как вы расположите три стороны треугольника, вы обнаружите, что утверждения в параграфе выше верны.

(Хорошо, равнобедренный и равносторонний треугольник являются исключениями из-за того, что у них нет ни одного наименьшего сторона или, в случае равностороннего треугольника, даже наибольшая сторона. Тем не менее, принцип, изложенный выше, остается в силе. истинный. !)

Рабочий лист по взаимосвязи между длинами сторон и измерениями углов треугольника

Калькулятор треугольника (Этот бесплатный онлайн-инструмент позволяет вычислить все измерения сторон и углов на основе введенных вами данных и нарисовать бесплатно загружаемое изображение вашего треугольника!)

Рабочие листы треугольника

Удаленные, внешние и внутренние углы треугольника

Удаленный, внешний вид, внутренний рабочий лист Бесплатный PDF

Что такое удаленный и внутренний углы?
Это все о расширении стороны треугольника

Внешний угол треугольника или любой многоугольник, образованный расширением одной из сторон.

В треугольнике каждый внешний угол имеет два удаленные внутренние углы. Удаленные внутренние углы — это всего лишь два угла, которые находятся внутри треугольник и противоположный от внешнего угла.

Формула

Как видно из рисунка выше, формула для дальнего и внутреннего углов гласит, что мера внешнего угла $$ \angle A $$ равна сумме удаленных друг от друга внутренних углов.

Перефразируя, угол «вне треугольника» (внешний угол A) равен D + C (сумма удаленных внутренних углов).

Интерактивная демонстрация удаленных и внешних углов

Интерактивная программа ниже позволяет вам перетаскивать точки треугольник вокруг. Обратите внимание, что сумма удаленных внутренние углы (C и D) равны внешнему углу A.

∠ С =

∠ Д =

∠ А =

Выберите удаленные углы C и DC и BB и D

Перетащите точки треугольника, чтобы начать демонстрацию

Практические задачи

Проблема 1

Используйте формулы, описанные на этой странице, для расчета значений x (дальнего внутреннего угла) и Y.

2 в степени х равно 1: кубических, тригонометрических, логарифмических и др. уравнений · Калькулятор Онлайн для чайников 🫖🤓

2
Функция — Квадрат x
ctg(x)
Функция — Котангенс от x
arcctg(x)
Функция — Арккотангенс от x
arcctgh(x)
Функция — Гиперболический арккотангенс от x
tg(x)
Функция — Тангенс от x
tgh(x)
Функция — Тангенс гиперболический от x
cbrt(x)
Функция — кубический корень из x
gamma(x)
Гамма-функция
LambertW(x)
Функция Ламберта
x! или factorial(x)
Факториал от x
DiracDelta(x)
Дельта-функция Дирака
Heaviside(x)
Функция Хевисайда

Интегральные функции:

Si(x)
Интегральный синус от x
Ci(x)
Интегральный косинус от x
Shi(x)
Интегральный гиперболический синус от x
Chi(x)
Интегральный гиперболический косинус от x

В выражениях можно применять следующие операции:

Действительные числа
вводить в виде 7. 3
— возведение в степень
x + 7
— сложение
x — 6
— вычитание
15/7
— дробь

Другие функции:

asec(x)
Функция — арксеканс от x
acsc(x)
Функция — арккосеканс от x
sec(x)
Функция — секанс от x
csc(x)
Функция — косеканс от x
floor(x)
Функция — округление x в меньшую сторону (пример floor(4.5)==4.0)
ceiling(x)
Функция — округление x в большую сторону (пример ceiling(4.5)==5.0)
sign(x)
Функция — Знак x
erf(x)
Функция ошибок (или интеграл вероятности)
laplace(x)
Функция Лапласа
asech(x)
Функция — гиперболический арксеканс от x
csch(x)
Функция — гиперболический косеканс от x
sech(x)
Функция — гиперболический секанс от x
acsch(x)
Функция — гиперболический арккосеканс от x

Постоянные:

pi
Число «Пи», которое примерно равно ~3. 2
3-8 9 Оценить квадратный корень из 12 10 Оценить квадратный корень из 20 11 Оценить квадратный корень из 50 94 18 Оценить квадратный корень из 45 19 Оценить квадратный корень из 32 20 Оценить квадратный корень из 18 92

Видео с вопросами: Нахождение значений выражений с использованием функций и показателей

Стенограмма видео

Учитывая, что 𝑓 меньше одного из 𝑥 равно восьми в степени 𝑥 и 𝑓 меньше двух из 𝑥 равно одному больше восьми в степени 𝑥, определить значение 𝑓 меньше одного из девяти больше двух минус 𝑓 меньше двух из отрицательных четырех больше 𝑓 меньше одного из пяти больше двух минус 𝑓 меньше двух из отрицательных двух.

Итак, наш первый член будет восемь в степени девять на два. Мы получаем это, потому что у нас есть 𝑓 sub one из девяти над двумя. Таким образом, мы заменим девять вместо двух на 𝑥 в нашей функции, которая является 𝑓 sub one. Тогда мы получим минус один на восемь в степени минус четыре, потому что мы подставили это вместо нашего 𝑥. И тогда у нас есть восемь в степени пять на два, потому что мы заменили пять на два вместо 𝑥, а затем минус один на восемь в степени минус два. Хорошо, отлично! Итак, что нам теперь делать?

Что ж, теперь нам нужно начать использовать некоторые из наших индексных законов. Ну, во-первых, если у нас есть 𝑥 в отрицательной степени, это будет равно единице больше 𝑥. Таким образом, мы можем переписать то, что у нас есть, как восемь в степени девять над двумя минус восемь в степени минус один все в степени минус четыре разделить на восемь в степени пять над двумя минус восемь в степени отрицательной единицы в степени отрицательной двойки.

Итак, что мы можем сделать, так это применить еще одно из наших правил индекса или экспоненты. И это если у нас есть 𝑥 в степени 𝑎 в степени 𝑏, это равно 𝑥 в степени 𝑎𝑏. Итак, мы умножаем наши показатели. Итак, мы получим восемь в степени девять на два минус восемь в степени четыре. И это потому, что у нас есть отрицательная единица, умноженная на отрицательную четыре, что дает нам четыре, затем все делится на восемь в степени пять на два минус восемь в квадрате.

Так что в этот момент нам может быть интересно, что мы собираемся делать. Что ж, именно здесь мы должны кое-что заметить. И это то, что мы можем факторизовать некоторые термины, которые у нас есть в нашем выражении. Мы видим, что на самом деле у нас есть множитель, который совпадает в числителе и знаменателе. А это один минус восемь в степени минус половина. Ну, как мы это получим?

Ну, если вы посмотрите на числитель, мы получим восемь в степени девять на два, умноженное на один минус восемь в степени минус половина. Что ж, если мы собираемся умножить восемь в степени девятки на два и восемь в степени минус половина, то мы добавим показатели степени. Что ж, девять больше двух плюс минус один больше двух дает нам восемь больше двух, что равно четырем, что у нас было в числителе ранее.

Итак, если мы посмотрим на знаменатель, мы получим восемь в степени пять на два, умноженное на один минус восемь в степени минус половина. Итак, опять же, если бы у нас были экспоненты, у нас было бы пять больше двух плюс отрицательная половина, что дает нам четыре больше двух или двух, опять же, что у нас было изначально. Хорошо, отлично! Итак, каков наш следующий шаг?

Что ж, один минус восемь в степени минус половина сокращаются в числителе и знаменателе, потому что мы можем разделить на это. Итак, у нас получится восемь в степени девять на два, деленное на восемь в степени пять на два. Итак, что мы можем сделать, так это использовать еще один из наших индексных законов. И это то, что 𝑥 в степени 𝑎 разделить на 𝑥 в степени 𝑏 равно 𝑥 в степени 𝑎 минус 𝑏. Итак, мы получим восемь в степени девять на два минус пять на два. Ну, девять на два это то же самое, что четыре с половиной.

5 примеров микроэкономики: Ошибка 403 — доступ запрещён

что изучают, в чем отличие, примеры

Содержание:

  • Определение экономики
  • Макроэкономика и микроэкономика — что собой представляют
    • Макроэкономика
    • Микроэкономика
  • Особенности исследований, как взаимодействуют между собой
  • Как связаны микроэкономика и макроэкономика, примеры
    • Пример взаимосвязи микро- и макроэкономики
  • Чем отличается микроэкономика от макроэкономики

Содержание

  • Определение экономики
  • Макроэкономика и микроэкономика — что собой представляют
    • Макроэкономика
    • Микроэкономика
  • Особенности исследований, как взаимодействуют между собой
  • Как связаны микроэкономика и макроэкономика, примеры
    • Пример взаимосвязи микро- и макроэкономики
  • Чем отличается микроэкономика от макроэкономики

Определение экономики

Понятие «Экономика» произошло от греческого слова oikonomos (ойкономос), что означает «домоводство» или «ведение домашнего хозяйства. Впервые данное определение было употреблено Ксенофонтом (430-355 гг. до н.э.).

 Экономика имеет различные толкования:

  1. Экономика — система хозяйства. То есть совокупность материальных и нематериальных условий производства и благ, которые люди используют для преобразования природного вещества для удовлетворения собственных потребностей и обеспечения условий жизни.
  2. Вся деятельность, которая осуществляется ежедневно и связана с производством и товарообменом.
  3. Это комплекс экономических отношений, возникающих между людьми в процессе их жизнедеятельности.
  4. Это знание, перерастающее в науку. Она изучает хозяйственную жизнь, ее закономерности, связанную с ними систему отношений между людьми. Экономическая теория ищет способы наиболее эффективного применения ресурсов в условиях их ограниченности для удовлетворения общественных и личных потребностей.

Макроэкономика и микроэкономика — что собой представляют

Макроэкономика

Определение

Макроэкономика — особая часть экономической теории, изучающая государственную и мировую экономики в целом и их функционирование.

Осторожно! Если преподаватель обнаружит плагиат в работе, не избежать крупных проблем (вплоть до отчисления). Если нет возможности написать самому, закажите тут.

Цели макроэкономики:

  • стабильный рост национального объема производства;
  • обеспечение полной занятости ресурсов;
  • поддержание стабильного уровня цен;
  • поддержание относительного равновесия между импортом и экспортом, то есть поддержание равновесного внешнеторгового баланса, складывающегося на основе свободной продажи товаров;
  • минимизация уровня инфляции;
  • поддержание высокого уровня занятости и минимизация безработицы;
  • поддержание и улучшение качества жизни населения;
  • поддержание стабильного курса национальной валюты.

Основным показателем результатов макроэкономической деятельности является валовый национальный продукт (ВНП).

Определение

ВНП — денежный показатель, который включает в себя всю стоимость произведенных в экономике за определенный промежуток времени конечных товаров и услуг. ВНП обычно рассчитывают за год.

Для расчета ВНП считают всю добавленную стоимость проданной или непроданной текущей продукции, но исключают из данного показателя продажу подержанных товаров, непроизводственные сделки и промежуточные товары.

Определение

Добавленная стоимость — рыночная цена, установленная для произведенной фирмой продукции, не учитывающая стоимость приобретенных у поставщиков потребленных материалов и сырья.

Дополнительными параметрами макроэкономики являются:

Валовый внутренний продукт — показатель, учитывающий продукцию, произведенную на территории страны всеми субъектами экономических отношений, за исключением заграничных отделений и филиалов фирм. Отличается от ВНП на величину разницы между экспортом и импортом.

Чистый национальный продукт (ЧНП) — это валовый национальный продукт (ВНП), не учитывающий амортизационные отчисления на потребление капитала. Это та сумма конечной продукции и услуг, которая осталась для потребления после замены умершего капитала.

Национальный доход (НД) — совокупный доход, заработанный поставщиками ресурсов. НД рассчитывается путем вычитания косвенных налогов на бизнес из ЧНП.

Личный доход (ЛД) — выплаченный суммарный доход семей и индивидов до уплаты налогов. ЛД — фактически полученный доход, который получают путем прибавления суммы трансфертных платежей и вычитания взносов на социальное страхование, налогов на прибыль корпораций, нераспределенной прибыли корпораций из НД.

Располагаемый личный доход (РЛД) — ЛД после вычитания индивидуальных налогов граждан.

Микроэкономика

Микроэкономический анализ ориентирован на индивидуальные компоненты экономики.

Индивидуальный экономический компонент или субъект — простой элемент хозяйственной системы, который самостоятельно реализует собственные экономические функции и является цельным, то есть рассматривается в рамках этой системы единым целым.

Микроэкономический анализ изучает поведение фирм, домохозяйств, инвесторов, предприятий, собственников земель и т. д., то есть отдельных экономических субъектов:

  • на рынках экономических ресурсов в целом: рынок капитала, труда, земли и т.д.;
  • на рынках отдельных товаров и услуг: рынок напитков или иных пищевых продуктов, зерна, книг, компьютеров, массажных услуг, нефти, транспорта и т.д.;
  • на различных типах рыночных структур: олигополия и монополия, совершенная и несовершенная конкуренции.

Микроэкономика рассматривает, как принимает экономические решения отдельная фирма (производитель) и домохозяйство (потребитель или собственник экономических ресурсов).

Принятие экономических решений, то есть процесс выбора определенного варианта развития экономики, — основа жизнедеятельности всех экономических субъектов и всей экономики в целом.

Экономический субъект выбирает определенный вариант своей деятельности и считает его наилучшим в сравнении с другими вариантами (относительно своих возможных выгод). При этом теряется возможность реализации альтернатив.

Работая над реализацией выбранного варианта, субъект уже не сможет свои затраченные ресурсы направить на альтернативный вариант. Поэтому все возможные выгоды другого варианта будут недостижимы для экономического субъекта, то есть он терпит альтернативные издержки.

Альтернативные издержки — издержки упущенных возможностей, оценочная выгода от второго наилучшего варианта экономического действия.

Учет альтернативных издержек считается важнейшим принципом микроэкономического анализа. Например, принцип альтернативности положен в основу модели производственных возможностей, позволяющих проанализировать экономическое развитие с позиции ответа на вопросы «как, что и для кого» производить.

Субъекты микроэкономики могут выступать во многих качествах, например, фирма может быть одновременно и производителем, и потребителем, и инвестором, и собственником и т.д. Но микроэкономический анализ рассматривает фирму, как и другие экономические единицы, в каждом из ее качеств отдельно.

Показателями микроэкономики являются индивидуальные величины:

  • объем выпуска товаров и услуг отдельной фирмы;
  • рентабельность активов предприятия;
  • прибыль отдельной фирмы;
  • рыночный спрос и предложение отдельного товара или экономического ресурса;
  • цена отдельного товара или услуги;
  • заработная плата работника;
  • объем нереализованных товаров и услуг;
  • доход отдельного потребителя;
  • дополнительные издержки на ремонт, транспортировку товаров и т.д.

Особенности исследований, как взаимодействуют между собой

Происходящие экономические процессы имеют свои закономерности, которые исследуют с точки зрения разных теорий.

Позитивная экономическая теория изучает экономику на базе обобщенных и систематизированных данных, полученных на основе наблюдений, реального опыта, информационных данных о реальном положении процессов и объектов.

Нормативная экономическая теория строится на основе идеального образа функционирования объектов и процессов. Используется для построения теоретической системы.

Оба метода важны, и для создания эффективной экономической теории необходимо их применять в совокупности.

Экономическая теория включает в себя два раздела: макроэкономический и микроэкономический анализы.

Предметом изучения макроэкономики как раздела экономической теории являются закономерности общественного воспроизводства, а также условия установления общего экономического равновесия.

Определение

Общее экономическое равновесие — состояние экономики, при котором пропорции обмена сложились таким образом, что на всех рынках одновременно достигнуто равенство спроса и предложения, и при этом ни один из субъектов не может улучшить свое благосостояние в сложившихся условиях.

Макроэкономическая теория изучает народное хозяйство в целом, как единую интегрированную систему, то есть все проблемы, связанные с совместной деятельностью огромного числа экономических объектов в рамках национальной экономики или экономики мира.

В микроэкономике предметом является отдельно взятый структурный элемент экономики. Он представляет собой относительно обособленную, самостоятельную экономическую единицу в экономической системе. В рамках общего макроэкономического взгляда на эту систему данный элемент не рассматривается отдельно и не несет важного значения, но он является системообразующим и основным ее звеном.

В рамках микроэкономического анализа рассматривают деятельность индивидуальных экономических субъектов: достижение ими собственных экономических задач через разработку, принятие и реализацию решений.

Микроэкономика, изучая отдельную единицу, также учитывает ее взаимодействие с другими единицами и рассматривает их особенную среду обитания — отрасли производства, рынки ресурсов и сбыта, организацию конкуренции. Она также исследует тот суммарный результат, который получит все общество в целом вследствие всеобщего взаимодействия экономических единиц.

Здесь микроэкономика неизбежно выходит из проблемы макроэкономики, следовательно, границы между этими двумя разделами экономической теории условны. В основе понимания макропроцессов лежит анализ поведения отдельных хозяйственных единиц.

Справочные материалы по макроэкономике включают в себя разделы микроэкономических основ исследования и методологические обоснования макроэкономических зависимостей.

И, наоборот, микроэкономические справочные материалы включают в себя темы из макроэкономики: роль государства в экономике отраслей, национальная экономика, спрос и предложение, эффективность рыночных систем, регулирование экономики и т.д.

Таким образом, макроэкономика и микроэкономика тесно взаимосвязаны друг с другом и составляют единую науку. Оба раздела экономической теории для анализа экономических процессов применяют одинаковые методологические принципы.

Как связаны микроэкономика и макроэкономика, примеры

Макро- и микроэкономические процессы тесно взаимодействуют друг с другом и подчинены единой задаче — удовлетворение общественных потребностей путем исследования факторов и закономерностей экономического роста и применения их на практике.

Миллионы индивидуальных решений определяют общеэкономические тенденции. То есть люди стремятся получить максимальный эффект от взаимодействия друг с другом с целью улучшения благосостояния. Это взаимодействие является результатом макроэкономических событий.

С другой стороны, индивидуальные субъекты микроэкономики анализируют и учитывают макроэкономическую ситуацию для принятия собственных экономических решений.

Пример взаимосвязи микро- и макроэкономики

Экономическое развитие фирм напрямую зависит от элементов макроэкономических решений, например, от сбережений, инвестиций, потребительских расходов и т.д.

В свою очередь макроэкономический анализ имеет в своей основе простейшую модель из микроэкономики: кругооборот продуктов и доходов между фирмами, домашними хозяйствами.

Фирмы производят товары и услуги, пользуясь ресурсами, которые им предоставили домохозяйства: труд, предпринимательские способности, земля, капитал. Домохозяйства предоставляют ресурсы фирмам с целью удовлетворения собственных потребностей.

Взаимосвязи, возникающие между фирмами и домохозяйствами, постоянно повторяются и осуществляются посредством натурально-вещественной и денежной форм.

Равенство суммы доходов и продаж фирм домохозяйств является основным принципом модели данного кругооборота. Следовательно, заработанные в кругообороте доходы пропорциональны выпущенной продукции и валовому национальному продукту.

Чем отличается микроэкономика от макроэкономики

Уровень анализа экономических процессов является основным отличием микроэкономики от макроэкономики.

К примеру, данные отдельной фирмы о количестве устроенных или уволенных сотрудников, или о количестве произведенных продуктов, не позволят сделать выводы о положении дел общего объема выпуска в экономике страны или об уровне безработицы.

И, наоборот, изменение совокупного спроса не отражают действительности в изменении спроса отдельного домохозяйства на определенный товар или услугу.

Отличия микро- и макроэкономики:

  1. Объектом в макроэкономике является экономика в целом, а в микроэкономике — индивидуальные компоненты.
  2. В макроэкономике агентами являются совокупные экономические агенты или секторы экономики, а в микроэкономике — фирмы и домохозяйства.
  3. Макроэкономический анализ в отличие от микроэкономического абстрагируется от различий между отдельными рынками и отраслями. Он поддерживает макроэкономическое равновесие через выяснение механизма функционирования экономической системы в общем.
  4. В рамках макроэкономики рассматривают экономические проблемы, воздействующие на всю экономику, а не на отдельные ее отрасли.
  5. Макроэкономика оперирует совокупными величинами, такие как ВНП, ВВП, совокупный спрос и т.д., а микроэкономика — индивидуальными величинами, такие как заработная плата сотрудника, прибыль отдельной фирмы, цена отдельного товара и т.д.
  6. В микроэкономике важны изменения относительных цен экономических ресурсов (соотношение цен одних товаров к другим), возникающих в отношениях между различными отраслями и видами производств. Для макроэкономики важны изменения абсолютных цен — то есть изменения общего уровня цен, что позволяет проанализировать проблему инфляции.
  7. Макроэкономические системы, в отличие от микроэкономических, могут восстановить свое равновесие посредством изменения количества, например, изменение количества совокупных расходов или увеличение выпуска товаров. Микроэкономические системы ориентируются на изменение уровня цен, чтобы восстановить равновесие.

Насколько полезной была для вас статья?

Рейтинг: 5.00 (Голосов: 1)

Макроэкономика и микроэкономика – что изучают, различия и примеры в обществознании кратко 

4.3

Средняя оценка: 4.3

Всего получено оценок: 153.

Обновлено 10 Июня, 2022

4.3

Средняя оценка: 4.3

Всего получено оценок: 153.

Обновлено 10 Июня, 2022

Экономическая сфера жизни общества решает проблему удовлетворения потребностей людей в условиях ограниченности имеющихся ресурсов. От того, насколько хорошо организована хозяйственная жизнь станы, зависят остальные области человеческой деятельности. Узнаем, что такое макроэкономика и микроэкономика, чем они отличаются.

Микроэкономика и макроэкономика в системе хозяйственных отношений

Макро- и микроэкономика являются составными частями экономики в целом. Для того, чтобы лучше понять их суть, познакомимся с определениями этих понятий.

  • Макроэкономика — это раздел экономики, объектом изучения которой являются экономические системы, особенности их функционирования, эффективность их составных частей. Так, например, она исследует хозяйственную деятельность государств мира.
  • Микроэкономика — это раздел, изучающий деятельность производителей и предприятий, небольших отраслей хозяйственной жизни, выявляющий особенности отдельных организаций. Например, объектом исследования может стать ценовая политика в отдельной области, товарооборот фирмы.

Таким образом, основное различие между понятиями заключается в степени охвата исследуемой области.

Впервые термин «макроэкономика» был употреблён в 1933 году учёным из Норвегии Рагнаром Фришем. Спустя три года его теорию более подробно разработал в своих трудах Джон Мейнард Кейнс, экономист из Великобритании.

Рис. 1. Рагнар Фриш.

Особенности макроэкономики

Исследователи макроэкономики выявляют проблемы мирового или национального масштаба, анализируют эффективность механизмов, регулирующих крупные сектора хозяйственной деятельности. Узнаем, что изучает макроэкономика.

  • Определение темпов экономического развития, выявление факторов, влияющих на него.
  • Анализ причин подъёма и спада экономики, разработка алгоритма выхода из кризиса.
  • Выявление уровня безработицы в стране, исследование её причин, определение эффективных способов борьбы с ней.
  • Изучение механизмов ценообразования, борьба с инфляцией.
  • Разработка государственного бюджета, анализ приоритетных сфер деятельности.
  • Составление плана торговой политики, контроль над объёмами экспорта и импорта.
  • Анализ показателей ВВП, национального дохода.
Рис. 2. Рост ВВП.

Таким образом, макроэкономика рассматривает хозяйственную деятельность как систему, выявляет закономерности её развития, исследует наиболее эффективные методы её организации.

Сфера деятельности микроэкономики

В обществознании под микроэкономикой понимается отрасль экономики, объектом изучения которой являются отдельные предприятия, товары и услуги, рынки.

  • Микроэкономика уделяет внимание спросу и предложению, занимается выявлением потребительских интересов, разрабатывает механизмы маркетинга.
  • Исследует проблемы отдельных производителей.
  • Регулирует рыночное равновесие в ситуации дефицита или профицита.
  • Выявляет количество населения, заинтересованного в приобретении товара, сохраняет баланс в системе покупатель-продавец.
  • Занимается составлением бизнес-планов отдельных фирм, даёт рекомендации по ведению бюджета семьи.
  • Анализирует показатели, связанные с производительностью труда (затраты на производство, количество выпускаемой продукции).
  • Отслеживает влияние автоматизации производства на развитие предприятий, даёт рекомендации по эффективному использованию времени, рабочих сил, ресурсов.
Рис. 3. Таблица «Макроэкономика и микроэкономика».

Микро- и макроэкономика не существуют изолированно друг от друга. Тесную связь этих областей можно подтвердить многочисленными примерами. Так, регулирование темпов инфляции сказывается на ценовой политике отдельных фирм. Коммерческие организации предоставляют рабочие места, что помогает регулировать уровень безработицы в государстве.

Что мы узнали?

Макроэкономика — раздел хозяйственной деятельности, исследующий экономику мирового или государственного уровня, выявляющий особенности развития крупных секторов. Микроэкономика изучает деятельность отдельных предприятий, разрабатывает механизмы эффективных экономических отношений в рамках небольшой отрасли.

Тест по теме

Доска почёта

Чтобы попасть сюда — пройдите тест.

    Пока никого нет. Будьте первым!

Оценка доклада

4.3

Средняя оценка: 4.3

Всего получено оценок: 153.


А какая ваша оценка?

Примеры микроэкономики (Микроэкономика и ее примеры)

Примеры микроэкономики

Содержание

В этой статье будет определен термин «микроэкономика ». Мы кратко объясним примеров микроэкономики и что этот термин означает по отношению к экономике. Наконец, мы приведем несколько примеров микроэкономики.

Согласно Investopedia, микроэкономика — это социальная наука, изучающая последствия стимулов и решений. В частности, речь идет о том, как они влияют на использование и распределение ресурсов.

Микроэкономика показывает, как и почему разные товары имеют разную ценность. Примеры микроэкономики включают в себя то, как люди и предприятия ведут себя и получают выгоду от эффективного производства и обмена, а также то, как люди лучше всего взаимодействуют и работают друг с другом. Как правило, микроэкономика обеспечивает более точное и детальное понимание, чем макроэкономика.

Что такое микроэкономика и ее примеры?

Микроэкономика занимается ценами и производством на отдельных рынках и взаимодействием между различными рынками. Мы уже обсуждали различия между микро- и макроэкономикой, из которых можно заметить сходство между микро- и макроэкономикой. Но помните, что микроэкономика рассматривает решения отдельных лиц и организаций о распределении ресурсов производства, обмена и потребления.

Вот несколько примеров микроэкономики

Лучший пример микроэкономической темы включает;

Спрос

Как спрос на товары влияет на доход, предпочтения, цены и такие факторы, как ожидания. Спрос относится к готовности потребителя покупать товары и услуги, а желание платить за них является примером микроэкономики.

Предложение

Как производители решают получить доступ к рынкам, масштабировать производство и в конечном итоге уйти с рынков? Предложение относится к экономическому термину, который влечет за собой количество определенного продукта или услуги, которые поставщики желают и могут предложить потребителям по определенной цене в течение определенного оговоренного периода времени в соответствии с примерами микроэкономики.

Цены

Как отдельные лица, домохозяйства и компании реагируют на цену товаров и как они в конечном итоге влияют на нее своим спросом и предложением.

Эластичность

Так спрос и предложение реагируют на изменения. Например, когда семья требует меньше товара, цена которого растет, и выбирает альтернативный заменитель. Скажем, семья, которая любит ирландский картофель на завтрак, может заменить сладкий картофель в случае, если цена первого вырастет.

Альтернативная стоимость

Это когда отдельные лица и фирмы, исходя из наличия ограниченных ресурсов, выбирают один вариант вместо другого. Это потому, что они не могут делать и то, и другое. Например, человек решает заняться бизнесом, а не поступить в колледж, потому что у него нет ни денег, ни времени на то и другое.

Конкурентное преимущество

Конкурентное преимущество, согласно Investopedia, «относится к факторам, которые позволяют компании производить товары или услуги лучше или дешевле, чем ее конкуренты. Эти факторы позволяют производственному предприятию генерировать больше продаж или более высокую прибыль по сравнению с его рыночными конкурентами». Таким образом, конкурентное преимущество, пример микроэкономики, представляет собой бизнес-атрибут, который дает хозяйствующему субъекту свободу действий, чтобы превзойти своих конкурентов.

Конкурентное преимущество включает доступ к природным ресурсам, которые доступны не всем конкурентам, доступ к новым технологиям, высококвалифицированным человеческим ресурсам или рабочей силе, способность производить продукцию по более низкой цене, чем другие участники бизнеса (Chaharbaghi ​​and Lynch, 1999 ).

Потребительский выбор

Здесь потребности, восприятие и информация формируют потребительский выбор. Потребительский выбор влечет за собой обдуманные решения, которые принимают потребители различных продуктов и услуг. Это относится к тому, как потребители решают, какие продукты они будут покупать и потреблять в течение определенного периода времени (Сальваторе, 2008).

Производительность

Производительность относится к показателю экономической эффективности, который сравнивает количество произведенных товаров и услуг с количеством ресурсов, использованных при производстве тех же товаров и услуг (www.bls.gov). «Экономическая производительность — это стоимость продукции, полученной на единицу затрат». (Economic Web Institute) Например, если в час отдельный рабочий производит 2 единицы продукции, стоимость которых составляет 20 долларов, то производительность рассматриваемого рабочего составляет 40 долларов.

Экономика благосостояния

Экономика благосостояния, согласно Britannica, — это «раздел экономики, который стремится оценивать экономическую политику с точки зрения ее влияния на благосостояние общества». Он рассматривает два типа благосостояния. Индивидуальное благополучие включает в себя счастье (например, оценку своего счастья), сравнение стран и коллективное благополучие. Он также рассматривает эффективное распределение ресурсов. Например, фонд развития сообщества (CDF) Замбии предназначен для оказания помощи почти всем жителям избирательного округа.

Информационная экономика

Информационная экономика влечет за собой влияние информации и информационных систем на экономические и впоследствии экономические решения. Обычно это услуги, используемые для создания и обработки информации, которая имеет решающее значение для того, чтобы люди могли принимать обоснованные экономические решения.

Доверие потребителей

Британская энциклопедия определяет доверие потребителей как «экономический показатель, который измеряет степень оптимизма потребителей в отношении общего состояния экономики страны и их собственного финансового положения». Доверие потребителей определяется состоянием экономики и тем, что освещается и сообщается в СМИ. Некоторыми из факторов, влияющих на доверие потребителей, являются изменения ставок на жилье, уровень безработицы и инфляция.

В заключение, примеры микроэкономики показывают, что микроэкономика является неотъемлемым атрибутом экономики. Это помогает специалистам в области экономики (экономистам) изучать и понимать динамику экономики, тем самым помогая им принимать обоснованные экономические решения.

Какой лучший пример микроэкономической темы?

Например, микроэкономика используется для объяснения почему спрос на определенный продукт увеличивается, когда у большего числа людей есть деньги. При низком доходе немногие люди будут иметь покупательную способность, чтобы купить товар, поэтому спрос уменьшится. Микроэкономика имеет дело с отдельными агрегатами экономики, такими как спрос, предложение, полезность и т.д.

ПОХОЖИЕ СТАТЬИ!
  • Сходства между микро- и макроэкономикой
  • Различия между микро- и макроэкономикой
  • Стоит ли иметь диплом бухгалтера?
  • Предложение в экономике
  • Типы государственных финансов
  • Четыре области государственных финансов
  • Бухгалтерский учет и финансы (Разница)
  • Различные виды аудита
  • Типы долгового финансирования
  • Тип s управленческой экономики
  • Четыре стадии бюджетного процесса

Микроэкономика Определение, использование и концепции

К

Команда Инвестопедии

Полная биография

Авторы Investopedia имеют разный опыт работы, и за 24 года работы тысячи опытных писателей и редакторов внесли свой вклад.

Узнайте о нашем редакционная политика

Обновлено 01 мая 2022 г.

Рассмотрено

Питер Вестфолл

Рассмотрено Питер Вестфолл

Полная биография

Питер Вестфолл — выдающийся профессор информационных систем и количественных наук Техасского технологического университета.

Узнайте о нашем Совет финансового контроля

Факт проверен

Пит Рэтберн

Факт проверен Пит Рэтберн

Полная биография

Пит Рэтберн — редактор текстов и специалист по проверке фактов с опытом работы в области экономики и личных финансов, а также более чем двадцатилетним опытом преподавания.

Узнайте о нашем редакционная политика

Инвестопедия / Тара Ананд

Что такое микроэкономика?

Микроэкономика — это социальная наука, изучающая последствия стимулов и решений, особенно то, как они влияют на использование и распределение ресурсов. Микроэкономика показывает, как и почему разные товары имеют разную стоимость, как люди и предприятия ведут себя и извлекают выгоду из эффективного производства и обмена, а также как люди лучше всего координируют свои действия и сотрудничают друг с другом. Вообще говоря, микроэкономика дает более полное и детальное понимание, чем макроэкономика.

Что такое микроэкономика?

Ключевые выводы

  • Микроэкономика изучает решения отдельных лиц и фирм о распределении ресурсов производства, обмена и потребления.
  • Микроэкономика занимается ценами и производством на отдельных рынках и взаимодействием между различными рынками, но оставляет изучение агрегатов в масштабах всей экономики макроэкономике.
  • Микроэкономисты формулируют различные типы моделей на основе логики и наблюдаемого человеческого поведения и проверяют модели на реальных наблюдениях.

Понимание микроэкономики

Микроэкономика изучает то, что может произойти (тенденции), когда люди делают выбор в ответ на изменения в стимулах, ценах, ресурсах и/или методах производства. Отдельные участники часто группируются в микроэкономические подгруппы, такие как покупатели, продавцы и владельцы бизнеса. Эти группы создают спрос и предложение на ресурсы, используя деньги и процентные ставки в качестве механизма ценообразования для координации.

Использование микроэкономики

Микроэкономика может применяться в положительном или нормативном смысле. Позитивная микроэкономика описывает экономическое поведение и объясняет, чего ожидать, если изменятся определенные условия. Если производитель поднимает цены на автомобили, позитивная микроэкономика говорит, что потребители склонны покупать меньше, чем раньше. Если крупный медный рудник в Южной Америке рухнет, цена на медь будет расти, потому что предложение ограничено. Позитивная микроэкономика может помочь инвестору понять, почему цены на акции Apple Inc. могут упасть, если потребители будут покупать меньше iPhone. Микроэкономика также может объяснить, почему более высокая минимальная заработная плата может вынудить The Wendy’s Company нанимать меньше рабочих.

Эти объяснения, выводы и прогнозы позитивной микроэкономики затем можно также применять нормативно, чтобы предписывать, что люди, предприятия и правительства должны делать, чтобы достичь наиболее ценных или выгодных моделей производства, обмена и потребления среди участников рынка. Это расширение выводов микроэкономики от того, что есть , до того, что должно быть или что люди должны делать , также требует, по крайней мере, неявного применения какой-либо этической или моральной теории или принципов, что обычно означает некоторую форму. утилитаризма.

Метод микроэкономики

Исторически микроэкономические исследования проводились в соответствии с теорией общего равновесия, разработанной Леоном Вальрасом в Elements of Pure Economics (1874), и теорией частичного равновесия, представленной Альфредом Маршаллом в Принципах экономики (1890). Методы Маршалла и Вальраса подпадают под более широкое понятие неоклассической микроэкономики. Неоклассическая экономика фокусируется на том, как потребители и производители делают рациональный выбор, чтобы максимизировать свое экономическое благосостояние, с учетом ограничений, связанных с тем, какой доход и ресурсы у них есть. Экономисты-неоклассики делают упрощающие предположения о рынках, такие как совершенное знание, бесконечное число покупателей и продавцов, однородные товары или статические переменные отношения, чтобы построить математические модели экономического поведения.

Эти методы пытаются представить человеческое поведение на языке функциональной математики, что позволяет экономистам разрабатывать математически проверяемые модели отдельных рынков. Неоклассики верят в построение измеримых гипотез об экономических событиях, а затем используют эмпирические данные, чтобы увидеть, какие гипотезы работают лучше всего. Таким образом, они следуют в «логическом позитивизме» или «логическом эмпиризме» ветви философии. Микроэкономика применяет ряд методов исследования в зависимости от изучаемого вопроса и вовлеченного поведения.

Основные понятия микроэкономики

Изучение микроэкономики включает в себя несколько ключевых концепций, в том числе (но не ограничиваясь ими):

  • Стимулы и поведение : Как люди, как отдельные лица, так и фирмы, реагируют на ситуации, с которыми они сталкиваются.
  • Теория полезности : Потребители предпочтут покупать и потреблять комбинацию товаров, которая максимизирует их счастье или «полезность», с учетом ограничения дохода, который они могут потратить.
  • Теория производства : Это изучение производства или процесса преобразования ресурсов в продукты. Производители стремятся выбрать такое сочетание ресурсов и методов их комбинирования, которое сведет к минимуму затраты, чтобы максимизировать свою прибыль.
  • Теория цен : Теория полезности и производства взаимодействуют, чтобы создать теорию спроса и предложения, которая определяет цены на конкурентном рынке.

Уравнение реакции c o2: CO + O2 = ? уравнение реакции

Методом электронного баланса уравнять окислительно – восстановительные реакции а) CO+O2→CO2 б) HI+SO2→I2+S+h3O — вопрос №2350443 — Учеба и наука

Ответы

01. 03.17

Михаил Александров

Читать ответы

Ольга

Читать ответы

Владимир

Читать ответы

Посмотреть всех экспертов из раздела Учеба и наука > Химия

Похожие вопросы

Помогите написать уравнения реакций с помощью которых можно осуществить следующие превращения: SiO2-Si-Ca2Si-Sih5-SiO2-Si; б). ..

CaC2->C2h3->C6H6->C6H6-NO2->C6H6-Nh3

В каком году была сформулирована теория — предшественница кислородной теории горения.

пропуская водород над нагретым Fe2O3,можно…

Решено

FeSO4+KClO3+h3SO4=Fe2(SO4)3+KCl+h3O

Пользуйтесь нашим приложением

Химические уравнения — как расставлять коэффициенты?

Поможем понять и полюбить химию

Начать учиться

129.6K

Из этой статьи вы узнаете, что такое химические уравнения, зачем они нужны и как их составлять, а также вспомните классификацию химических реакций — все это, конечно, с примерами уравнений.

Химическое уравнение — это условная запись химического превращения с помощью химических формул и математических знаков

При составлении химических уравнений используют математические знаки «+», «−», «=», а также числа — они выступают в качестве коэффициентов и индексов.

Коэффициенты показывают число частиц (атомов или молекул), а индексы — число атомов, которые входят в состав молекулы.

Химическую реакцию можно изобразить в виде схемы:

На схеме протекание реакции представлено нагляднее, но сложные химические процессы изобразить таким способом сложно. Поэтому их записывают в виде химического уравнения.

Вещества, которые вступают в реакцию, называют исходными веществами, или реагентами. Вещества, которые образуются в результате, называют продуктами реакции.

Давайте разберем этот пример химического уравнения. Здесь видно, что из двух молекул водорода и одной молекулы кислорода образуются две молекулы воды. Реагенты в данном случае — водород и кислород, продукт реакции — вода.

Новые вещества образуются вследствие перегруппировки исходных атомов. В результате химической реакции атомы химических элементов никуда не исчезают и не возникают новые, их число остается неизменным — это следует из закона сохранения массы веществ.

Закон сохранения массы веществ

Масса веществ, вступивших в реакцию, равна массе веществ, образовавшихся в ходе этой реакции.

Закон сохранения массы веществ лежит в основе химии и используется при составлении уравнений химических реакций.

Алгоритм составления уравнения химической реакции

Рассмотрим, как составлять уравнения химических реакций, на примере взаимодействия магния и кислорода с образованием оксида магния.

  1. Записываем химические формулы исходных веществ в левой части уравнения. Напоминаем: молекулы H2, N2, O2, F2, Cl2, Br2, I2 двухатомны. Между исходными веществами ставим «+», а затем знак «=».

  2. После знака равенства записываем химическую формулу продукта. Химическую формулу необходимо составить с учетом валентностей химических элементов.

  3. Согласно закону сохранения массы веществ, число атомов каждого химического элемента до и после реакции должно быть одинаковым. Давайте посмотрим, как расставлять коэффициенты в химических уравнениях, чтобы закон выполнялся.

    Из составленной химической реакции видно, что количество атомов магния слева и справа от знака равенства одинаково, но атомов кислорода слева два, а справа один.

    Чтобы уравнять число атомов в химическом уравнении, находим наименьшее общее кратное (НОК), в нашем случае — 2. А затем делим НОК на количество атомов кислорода в реагентах и полученное число записываем в виде коэффициента.

    Это важно

    Коэффициент 1 в уравнении химической реакции не указывается, но при подсчете суммы коэффициентов в уравнении его необходимо учитывать.

  4. Проверим количество атомов магния до и после знака «=». Если перед химической формулой уже стоит коэффициент, то для подсчета количества атомов необходимо умножить коэффициент на индекс, который относится к этому химическому элементу.

  5. Чтобы уравнять количество атомов магния в химической реакции, посчитаем НОК и разделим его на количество атомов с каждой стороны от знака «=». Результат деления и будет являться коэффициентом (повторяем расстановку коэффициентов в химическом уравнении из 3-го пункта).

  6. Получаем уравнение химической реакции, в котором в исходных веществах и продуктах реакции по 2 атома магния и кислорода.

Сумма коэффициентов в этом химическом уравнении равна 5 (2 + 1 + 2 = 5).

Коэффициенты, которые стоят в химическом уравнении перед веществами, указывают на мольное соотношение исходных веществ и продуктов реакции, по которому и производятся расчеты.

Твоя пятёрка по английскому.

С подробными решениями домашки от Skysmart

Типы химических реакций

Химические реакции можно классифицировать по различным признакам:

  1. По числу и составу исходных веществ и продуктов реакции.

  2. По изменению степени окисления.

  3. По тепловому эффекту.

  4. По агрегатному состоянию.

  5. По наличию или отсутствию катализатора.

  6. По обратимости.

По числу и составу исходных веществ и продуктов реакции

По этому признаку выделяют 4 типа реакций: реакции соединения, реакции разложения, реакции замещения и реакции обмена.

Реакции соединения — это реакции, в результате которых из нескольких более простых веществ образуется одно более сложное.

Например, простые вещества барий и кислород взаимодействуют с образованием сложного вещества оксида бария:

2Ba + O2 = 2BaO.

Также к реакциям соединения относится взаимодействие оксида натрия с водой с образованием более сложного вещества гидроксида натрия. Оно более сложное, так как состоит уже из трех атомов химических элементов, в отличие от веществ-реагентов, которые состоят из двух атомов:

Na2O + H2O = 2NaOH.

Реакции разложения — это реакции, в результате которых из одного более сложного вещества образуется несколько более простых веществ. Является процессом, обратным реакции соединения.

Пример такой реакции — разложение нитрата серебра на несколько более простых веществ: серебро, оксид азота (IV) и кислород.

2AgNO3 = 2Ag + 2NO2↑ + O2↑.

Что это за стрелочка? 🤔

Стрелка вверх означает, что получившееся вещество является газом, который покидает место проведения реакции и больше не участвует в ней.

Реакции замещения — это такие реакции, в результате которых атомы простого вещества замещают атомы одного химического элемента в сложном веществе. Также возможно замещение функциональных групп в сложном веществе.

Например, замещение атомов водорода в молекуле соляной кислоты на атомы цинка:

Zn + 2HCl = ZnCl2 + H2↑.

Реакции обмена — это реакции между двумя сложными веществами, в результате которых вещества обмениваются своими составными частями.

Например, NaOH + H2SO4 = Na2SO4 + H2O.

Реакции нейтрализации

Реакция щелочи с кислотой называется реакцией нейтрализации и является частным случаем реакции обмена.

Для наглядности показали все типы химических реакций по этому признаку на схеме:

По изменению степени окисления

По этому признаку выделяют два вида реакций:

  • реакции, протекающие без изменения степени окисления;

  • окислительно-восстановительные реакции (ОВР) — реакции, протекающие с изменением степени окисления нескольких элементов.

В ОВР всегда участвуют вещество-окислитель и вещество-восстановитель. Другие исходные вещества, принимающие участие в реакции, выступают в качестве среды, в которой протекает эта реакция.

Окислитель — вещество, в состав которого входит ион или атом, который в процессе реакции принимает электроны, тем самым понижая свою степень окисления.

Восстановитель — вещество, в состав которого входит ион или атом, который в процессе реакции отдает электроны, тем самым повышая свою степень окисления.

Из определений можно сделать вывод, что в ходе реакции протекает два процесса: принятие электронов (восстановление) и отдача электронов (окисление). Протекают они одновременно.

По тепловому эффекту

По тепловому эффекту реакции делятся на эндотермические и экзотермические.

Эндотермические реакции протекают с поглощением теплоты (−Q). Буквой Q обозначается количество теплоты.

К таким реакциям относятся практически все реакции разложения. Пример:

CaCO3 = CaO + CO2 − Q.

Экзотермические реакции протекают с выделением теплоты (+Q).

К таким реакциям относятся практически все реакции соединения. Пример:

2H2 + O2 = 2H2O + Q.

По агрегатному состоянию исходных веществ

По этому признаку все реакции разделяют на гомогенные и гетерогенные.

Гомогенные реакции протекают в одной фазе.

К гомогенным реакциям относятся те, исходные вещества которых находятся либо в жидком агрегатном состоянии, либо в газообразном. Например, взаимодействие двух газообразных веществ — водорода и хлора:

H2(г) + Cl2(г) = 2HCl.

Агрегатное состояние указывается в правом нижнем углу: «г» — газообразное, «ж» — жидкое, «тв» — твердое.

Гетерогенные реакции протекают на границе раздела фаз.

Как правило, такие реакции протекают между веществами, которые находятся в разных агрегатных состояниях:

2Na(тв) + 2H2O(ж) = 2NaOH + H2.

Также к гетерогенным относятся реакции между двумя несмешивающимися жидкостями. Собрали несколько примеров гетерогенных реакций:

Между какими веществами протекает

Обозначение

Примеры реагентов

Между жидкостью и твердым веществом

ж—тв

Сода и уксусная кислота

Между твердым веществом и газом

г—тв

Натрий с водой

Между жидким веществом и газом

г—ж

Между двумя несмешивающимися жидкостями

ж—ж

Вода и масло

По наличию или отсутствию катализатора

По этому признаку выделяют реакции каталитические и некаталитические.

Каталитические реакции — реакции, протекающие с участием катализатора

Катализатор — вещество, которое ускоряет реакцию, участвует в ней, но остается неизменным после окончания этой реакции.

Наличие катализатора указывается над знаком равенства как kat или формула конкретного вещества, выступающего в роли катализатора.

Например:

Некаталитические реакции — реакции, протекающие без участия катализатора.

По обратимости

Различают обратимые и необратимые реакции.

Обратимые реакции — реакции, протекающие в двух противоположных направлениях.

При составлении уравнений обратимых реакций вместо знака равенства используют знак «⇄».

К обратимым реакциям относят реакции ионного обмена, диссоциации электролитов и многие другие:

H2 + I2 ⇄ 2HI.

Необратимые реакции — реакции, которые протекают только в одном направлении.

Чтобы научиться составлять уравнения химических реакций, нужно только одно — практика. Много практики школьники получают на онлайн-курсах по химии в Skysmart. Интересные задания на интерактивной платформе, примеры из жизни и опытные преподаватели обязательно приведут к желаемому результату — и просто помогут полюбить химию.

Татьяна Сосновцева

К предыдущей статье

Способы получения алкенов

К следующей статье

Тепловой эффект

Получите план обучения, который поможет понять и полюбить химию

На вводном уроке с методистом

  1. Выявим пробелы в знаниях и дадим советы по обучению

  2. Расскажем, как проходят занятия

  3. Подберём курс

Химические реакции — Общая информация

Химические реакции – общая информация

Химические уравнения — это сокращенный метод представления химических реакций. В химической реакции реагенты (то, что вы начинаете) превращаются в продукты (то, чем вы заканчиваете). Реагенты, показанные в левой части уравнения, и продукты, показанные в правой, разделены стрелкой. Уравнение ниже представляет собой реакцию углерода с газообразным кислородом с образованием двуокиси углерода.

C + O 2 CO 2

Эту реакцию также можно представить графически:

Обратите внимание, что количество атомов углерода одинаково по обеим сторонам стрелки. На стороне реагента находится один атом углерода, а на стороне продукта — один атом углерода. То же верно и для кислорода, за исключением того, что с каждой стороны находится по два атома кислорода (помните, что индекс «два» в молекуле кислорода означает, что два атома кислорода связаны вместе). Когда количество атомов в каждой части уравнения одинаково, говорят, что уравнение уравновешено. Сбалансированное уравнение согласуется с законом сохранения материи, который гласит, что материя не создается и не разрушается во время химической реакции.

При взаимодействии метана (CH 4 ) с кислородом образуется двуокись углерода и вода.

CH 4 + O 2 H 2 O + CO 2

Здесь мы замечаем, что реакция не уравновешена, так как количество атомов водорода с каждой стороны разное. То же самое верно и для количества атомов кислорода. Чтобы сбалансировать это уравнение, мы должны добавить коэффициенты перед кислородом и водой. Коэффициенты используются, когда мы хотим представить более одного конкретного атома или молекулы.

CH 4 + 2 O 2 2 H 2 O + CO 2

При добавлении коэффициентов реакция уравновешивается. Одна молекула метана взаимодействует с двумя молекулами кислорода, образуя одну молекулу углекислого газа и две молекулы воды. Наглядно это может быть представлено:

Часто физическое состояние реагентов или продуктов также включается в уравнение: (s) используется для твердого тела, (l) для жидкости и (g) для газа. Если вещество растворено в воде, используется (aq), что означает водный. Добавляя эти символы, уравнение принимает вид:
CH 4 (г) + 2 O 2 (г) 2 H 2 O(л) + CO 2 (г)

Ряд других символов иногда используется при написании химических уравнений.

Продолжайте читать о том, как классифицировать реакции.

введение химические реакции признаки реакции эксперимент дополнительная практика

бытовая химия — С+О2 равно С+О, как такое возможно

Давайте сначала взглянем на аллотропы кислорода и более подробно рассмотрим молекулярный кислород.

  • Атомарный кислород ($\ce{O1}$, свободный радикал)
  • Синглетный кислород ($\ce{O2}$), одно из двух метастабильных состояний молекулярного кислорода
  • Тетракислород ($\ce{O4}$), еще одна метастабильная форма

От НАСА, http://www. nasa.gov/topics/technology/features/atomic_oxygen.html, относительно атомарного кислорода:

Атомарный кислород недолго существует в природе на поверхности Земли, так как он очень реакционноспособен. Но в космосе, где много ультрафиолетового излучения, молекулы $\ce{O2}$ легче распадаются на атомарный кислород. Атмосфера на низкой околоземной орбите состоит примерно из 96% атомарного кислорода. В первые дни космических полетов НАСА присутствие атомарного кислорода вызывало проблемы.

Дикислород, или триплетный кислород, является наиболее широко известным аллотропом кислорода. Он имеет молекулярную формулу $\ce{O2}$. У кислорода 8 электронов: 2 на 1s, 2 на 2s, 4 на 3p-орбиталях. Альтернативно, есть 6 валентных электронов. Если есть другие молекулы кислорода, кислород будет спариваться, образуя двойную связь с порядком связи два. Короче говоря, потенциальная энергия дикислорода намного меньше, чем у атомарного кислорода. 91\Sigma \text{g+}$ возбужденное состояние.

Это определение, взятое из Университета Пердью, хорошо резюмирует правило максимальной простоты Хунда: каждая орбиталь в подоболочке занята одним электроном до того, как любая орбиталь будет занята дважды, и все электроны на однократно занятых орбиталях имеют одинаковый спин.

Две первые диаграммы нарушают 1.) правило выбора спина: перевороты спина запрещены и 2.) правило выбора Лапорта: переходы между орбиталями одной и той же четности запрещены, где четность означает симметрию относительно инверсия. Существует немецкое обозначение gerade, которое относится к симметричному по отношению к инверсии, и ungerade, которое относится к антисимметричному по отношению к инверсии.

Существует множество способов получения озона. https://en.wikipedia.org/wiki/Озон#Производство Озон представляет собой трехатомную молекулу с 3 атомами кислорода. Он гораздо менее стабилен, чем молекулярный кислород, и часто распадается на молекулярный кислород.

Возможно, вас смутило неправильное «правило наибольшей простоты» Дальтона.

Примеры решения задач по физике 8 класс: 8 КЛАСС — УЧИМСЯ РЕШАТЬ ЗАДАЧИ ПО ФИЗИКЕ — Каталог статей

Задачи по физике с ответами 8 класс

Задачи по физике с ответами для оценки уровня подготовки.

Задача 1 :
На сколько изменяется внутренняя энергия Царь-пушки массой 40 т при максимальном зарегистрированном в Москве перепаде температуры от + 36 °С до — 42,2 °С? Удельная теплоемкость металла 0,45 кДж/(кг • К).
Ответ: на 1420 МДж.

Задача 2 :
До какой температуры раскаляется почва в Узбеки­стане, если внутренняя энергия каждого кубометра изменя­ется при этом на 93,744 МДж? Начальная температура по­чвы 17 °С, плотность грунта 1800 кг/м3, его удельная теп­лоемкость 0,84 кДж/(кг • К).
Ответ: 79 °С.

Задача 3 :
Самая высокая температура почвы в Туркмении до­стигает 77 °С. Какова начальная температура куриного яйца-гиганта массой 420 г, зарегистрированного в 1977 г. в Киргизии, если оно получило при засыпании горячим песком 40 кДж энергии? Удельная теплоемкость содержи­мого яйца 2 кДж/(кг • К).
Ответ: 27 °С.

Задача 4 :
В 1879 г. на Урале нашли монолит малахита мас­сой 1054 кг. На сколько изменилась его внутренняя энер­гия, если при перевозке температура возросла на 20 °С?
Ответ: на 25,3 МДж.

Задача 5 :
В Калининградском музее янтаря хранится уникаль­ная находка массой 2480 г. На сколько изменилась внут­ренняя энергия этого куска при переносе его в музей, если температура воды в Балтийском море 10 °С, а в музее 20 °С? Удельная теплоемкость янтаря 2 кДж/(кг • К).
Ответ: на 85,6 кДж.

Задача 6 :
Какова масса куска янтаря, хранящегося в Палан­ге, если при изменении температуры от 5 до 18 °С его энер­гия увеличилась на 93,6 кДж?
Ответ: 3600 г.

Задача 7 :
Самый крупный топаз массой 117 кг был найден на Украине в 1965 г. Как изменится его внутренняя энер­гия при зимней транспортировке из Москвы в Париж, если средние температуры в этих городах составляют соответ­ственно -10 °С и +3,5 °С? Удельная теплоемкость камня 0,84 кДж/(кг • К).
Ответ: увеличится на 1,33 МДж.

Задача 8 :
Какова температура воды в самом горячем озере на Камчатке, если для приготовления ванны объемом 200 л температурой 40 °С в нее влили 40 л воды при 10 °С?
Ответ: 50 °С.

Задача 9 :
Какова летняя температура воды в самом холодном Восточно-Сибирском море, если для получения 10 м3 воды при температуре 20 °С в нее надо добавить 2 л кипят­ка?
Ответ: 0 °С.

Задача 10 :
В 1968 г. в Благовещенске выпал крупный град, при­чем при температуре 0 °С масса одной градины составляла 400-600 г. Сколько спирта надо сжечь, чтобы получить из нею воду при 20 °С? Потерями пренебречь. Удельная тепло­та сгорания спирта 27 МДж/кг.
Ответ: 6,1-9,2 г.

Задача 11 :
В 1965 г. в Кисловодске выпал град, который по­крыл почву слоем толщиной 75 см. На сколько измени­лась внутренняя энергия каждого квадратного метра при его таянии? Насыпная плотность вещества 800 кг/м3.
Ответ: 198 МДж.

Задача 12 :
В 1843 г. на Урале был найден самородок платины массой 9636 г.
Какова температура плавления платины, если для его переплавки израсходовали 3466 кДж тепла? Удельная теплоемкость платины 140 Дж/(кг* К), удельная теплота плавления 113 кДж/кг, начальная температура 10 °С
1770 °С.

Задача 13 :
Русский мастер Чохов в XVII в. отлил колокол мас­сой 35 т. Какое количество теплоты потребовалось для при­готовления расплава, если начальная температура металла была 20 °С? Удельная теплоемкость сплава 0,4 кДж/(кг • К), температура плавления 1100 °С, удель­ная теплота плавления 213 Дж/г.
Ответ: 2260 МДж.

Задача 14 :
В Алмазном фонде Кремля хранится золотой самородок «Лошадиная голова». Какова масса самородка, если для его полного расплавления потребовалось бы 938 кДж тепла?
Ответ: 14 кг.

Задача 15 :
Золотой самородок «Верблюд» имеет массу 9,3 кг и температуру 15 °С. Какова температура плавления золо­та, если для переплавки потребовалось бы 1892 кДж теп­ла?
Ответ: 1064 °С.

Задача 16 :
При раскопках в Алуште в 1990 г. нашли 17 слит­ков серебра общей массой 3,5 кг при температуре 5 °С. Какова удельная теплота плавления серебра, если для пе­реплавки потребовалось 254 г газа удельной теплотой сго­рания 45 МДж/кг? Потерями пренебречь.
Ответ: 87 кДж/кг.

Задача 17 :
Какова самая низкая температура, зарегистриро­ванная на арктической станции «Восток», если 200 мл воды температурой 15 °С, вынесенные из помещения и оставлен­ные на ночь, выделили 105 714 Дж энергии?
Ответ: -89,2 °С.

Задача 18 :
Какая самая низкая температура воздуха в районе реки Индигирки была зарегистрирована, если для получе­ния воды при 18 °С из куска льда объемом 0,5 м3 потребо­валось сжечь 6 кг дизельного топлива, удельная теплота сгорания которого 42,7 МДж/кг?
Ответ: -78 °С.

Задача 19 :
Самовар, изготовленный в Туле в 1922 г., имел ем­кость 250 л. За сколько времени он закипал при ежеминут­ном сгорании 600 г дров? Начальная температура воды 10 °С, КПД 40 %, удельная теплота сгорания дров 10 МДж/кг.
Ответ: за 40 мин.

Задача 20 :
Какую емкость имел новый тульский самовар-ре­кордсмен, если при КПД 50 % он закипал за 20 мин и по­треблял ежеминутно 460 г древесного угля, удельная тепло­та сгорания которого 35 МДж/кг? Начальная температура воды 15 °С.
Ответ: 450 л.

Задача 21 :
Сколько древесного угля нужно сжечь, чтобы вскипятить воду в 50 литровом Суксунском самоваре, если начальная температура воды равна 20°С? Удельная теплота сгорания древесного угля 35 МДж/кг?
Ответ: 0,48кг

Задача 22 :
Самый экономичный тепловой двигатель 1840 г. потреблял 0,77 кг угля при мощности 735 Вт. Каков КПД установки? Удельная теплота сгорания угля 29 Мдж/кг.
Ответ: 12 %.

Задача 23 :
Самый большой американский бойлер при мощ­ности 1330 МВт дает 4 232 000 кг пара в час. Каков КПД установки, если туда поступает вода при 20 °С?
Ответ: 50%.

Задача 24 :
Самый мощный дизельный двигатель в Швейца­рии имеет мощность 41 920 кВт. Сколько топлива в час он потребляет при работе, если его КПД 35 %? Удельная теп­лота сгорания топлива 42 МДж/кг.
Ответ: 10,3 т.

Задача 25 :
Самая крупная нефтеналивная цистерна имеет ем­кость 1,5 млн баррелей (1 баррель = 158,988 л). Сколько тепла выделяется при полном сгорании нефти? Удельная теплота сгорания нефти 43 МДж/кг, плотность 0,8 т/м3.
Ответ: 1015 Дж.

Задача 26 :
Крупнейшее месторождение в Уренгое дает 261,6 млрд кубометров газа в год. Какое количество тепло­ты ежедневно можно получать при его сжигании? Плот­ность газа 1,2 кг/м3, удельная теплота сгорания газа 50 МДж/кг.
Ответ: 35,6 • 1018 Дж.

Задача 27 :
Самый крупный ледник Западного Памира имеет объем 144 км3 и среднюю температуру -10 °С. Сколько теп­ла потребовалось бы для его плавления?
Ответ: 3 • 1020 Дж.

Задача 28 :
Россия, год 1842, 8 октября. На прииске Царево-Александровский близ города Миасс, что на Южном Урале, найден самородок золота весом 36 кг 16 г. Ныне «Большой треугольник» — так назвали уникальный экземпляр — можно увидеть в Алмазном фонде Московского Кремля. Он считается самым крупным, из сохранившихся в мире. На сколько градусов он нагреется, если по­лучит 18 720 Дж тепла? Удельная теплоемкость золота 0,13 кДж/(кг • К).
Ответ: на 4 °С.

Задача 29 :
Самородок «Заячьи Уши» имеет массу 3 344,3 г. Каков объем данного самородка?

Задача 30 :
Самый большой в мире самородок золота был найден в Австралии в 1872г на руднике Хилл-Энд. Самородок имел форму плитки длиной 144 см, шириной — 66 см и толщиной 10 см. Самородок был назван «Плита Холтермана». Чему равна масса самородка?

     Задачи по физике с ответами 8 класс          Тест по физике 8 класс

Решение задач на удельную теплоемкость | 8 класс

Содержание

    Вы познакомились с понятиями количества теплоты и удельной теплоемкости. В уроке «Расчет количества теплоты, необходимого для нагревании тела или выделяемого им при охлаждении» вы познакомились с основной формулой, которую мы будем использовать и в этом уроке:

    $Q = cm(t_2 — t_1)$

    В данном уроке мы рассмотрим задачи на нахождение различных величин, связанных с нагреванием и охлаждением тел. При их решении вам может понадобиться таблица значений удельной теплоемкости различных веществ из прошлого урока.

    Задача №1 на расчет количества теплоты

    Рассчитайте количество теплоты, необходимое для нагрева $15 \space кг$ меди на $80 \degree C$.

    Дано:
    $m = 15 \space кг$
    $c = 400 \frac{Дж}{кг \cdot \degree C}$
    $\Delta t = 80 \degree C$

    $Q — ?$

    Показать решение и ответ

    Скрыть

    Решение:

    Для решения этой задачи мы будем использовать формулу для расчета количества теплоты, необходимого для нагревания тела:
    $Q = cm(t_2 — t_1)$.

    В данном случае нам не известны начальная и конечная температуры тела ($t_2$ и $t_1$). Нам известно изменение этой температуры: $\Delta t = t_2 — t_1$. Тогда формула для расчета количества теплоты примет вид:
    $Q = cm \Delta t$.

    Подставим значения всех величин и рассчитаем количество теплоты:
    $Q = 400 \frac{Дж}{кг \cdot \degree C} \cdot 15 \space кг \cdot 80 \degree C = 480 \space 000 \space Дж = 480 \space кДж$. 3}$.

    Задача №5 на расчет удельной теплоемкости

    В калориметр было налито $450 \space г$ воды, температура которой $20 \degree C$. Когда в эту воду погрузили $200 \space г$ железных опилок, нагретых до $100 \degree C$, температура воды стала равна $24 \degree C$. Определите удельную теплоемкость опилок.

    Записывая условия задачи, используем индекс “в” для обозначения величин, связанных с водой, и индекс “ж” для обозначения величин, связанных с железными опилками.

    Дано:
    $m_в = 450 \space г$
    $m_ж = 200 \space г$
    $t_{в1} = 20 \degree C$
    $t_{в2} = 24 \degree C$
    $c_в = 4200 \frac{Дж}{кг \cdot \degree C}$
    $t_{ж1} = 100 \degree C$

    СИ:
    $m_в = 0.45 \space кг$
    $m_ж = 0.2 \space кг$

    $с_ж — ?$

    Посмотреть решение и ответ

    Скрыть

    Решение:

    Формула для расчета количества теплоты, необходимого для нагревания тела и выделяемого при его охлаждении:
    $Q = cm(t_2 — t_1)$.

    Запишем эту формулу для воды:
    $Q_в = c_в m_в (t_{в2} — t_{в1})$.

    Запишем формулу количества теплоты для железных опилок:
    $Q_ж = c_ж m_ж (t_{ж2} — t_{ж1})$.

    Нагретые железные опилки помещают в воду для их охлаждения. Значит, вода будет нагреваться и поглотит некоторое количество теплоты, а опилки будут охлаждаться и выделят некоторое количество теплоты. Т.е., между этими телами будет происходить теплообмен, для которого действует уже известное вам правило:

    Если между телами происходит теплообмен, то внутренняя энергия всех нагревающихся тел увеличивается на столько, на сколько уменьшается внутренняя энергия остывающих тел.

    Это значит, что количество теплоты $Q_в$, полученное водой, будет равно количеству теплоту $Q_ж$, которое выделится при охлаждении железных опилок, но с обратным знаком: $Q_в = — Q_ж$.

    Подставим выражения, которые дает формула для расчета количества теплоты:
    $c_в m_в (t_{в2} — t_{в1}) = — c_ж m_ж (t_{ж2} — t_{ж1})$.

    После завершения теплообмена температура воды и температура железных опилок будут равны друг другу: $t_в2 = t_ж2 = t_2$.

    Подставим в наше равенство и выразим $c_ж$:
    $c_ж = — \frac{c_в m_в (t_2 — t_{в1})}{m_ж (t_2 — t_{ж1})}$.

    Рассчитаем удельную теплоемкость железных опилок:
    $c_ж = — \frac{4200 \frac{Дж}{кг \cdot \degree C} \cdot 0.45 \space кг \cdot (24 \degree C — 20 \degree C)}{0.2 \space кг \cdot (24 \degree C — 100 \degree C)} = — \frac{7560 \space Дж}{- 15.2 \space кг \cdot \degree C} \approx 497 \frac{Дж}{кг \cdot \degree C} \approx 0.5 \frac{кДж}{кг \cdot \degree C}$.

    Ответ: $c_ж \approx 0.5 \frac{кДж}{кг \cdot \degree C}$.

    Задача №6 на использование графика

    Используя график зависимости температуры керосина от сообщенного ему количества теплоты (рисунок 1), определите массу керосина.

    Рисунок 1. График зависимости температуры керосина от сообщаемого количества теплоты

    Для начала нам нужно записать условия задачи. 3$

    $t_{в2} — ?$

    Посмотреть решение и ответ

    Скрыть

    Решение:

    Когда нагретый резец опускают в холодную воду, между этими двумя телами происходит теплообмен. Резец остывает и выделяет энергию, а вода получает эту энергию и нагревается. Соответственно, количество теплоты, которое выделится при остывании стального резца, численно будет равно количеству теплоту, которое получит вода. 

    Когда теплообмен завершится,температуры стального резца и воды будут одинаковы: $t_{в2} = t_{р2} = t_2$.

    Запишем формулу для расчета количества теплоты, которое выделится при остывании резца:
    $Q_р = с_р m_р (t_2 — t_{р1})$.

    Запишем формулу для расчета количества теплоты, которое получила вода:
    $Q_в = с_в m_в (t_2 — t_{в1})$.

    Приравняем правые части этих уравнений, не забыв про знак “минус”, которые указывает на выделение энергии при охлаждении тела:
    $с_р m_р (t_2 — t_{р1}) = — с_в m_в (t_2 — t_{в1})$.

    Раскроем скобки:
    $с_р m_р t_2 — с_р m_р t_{р1} = — с_в m_в t_2 + с_в m_в t_{в1}$. 3 \frac{Дж}{\degree C}} \approx 22.3 \degree C$.

    Ответ: $t_2 \approx 22.3 \degree C$.

    Задача №8 на смешивание трех жидкостей

    Какой температуры получится вода, если смешать $0.02 \space кг$ воды при $15 \degree C$, $0.03 \space кг$ воды при $25 \degree C$ и $0.01 \space кг$ воды при $60 \degree C$?

    Дано:
    $m_1 = 0.02 \space кг$
    $t_1 = 15 \degree C$
    $m_2 = 0.03 \space кг$
    $t_2 = 25 \degree C$
    $m_3 = 0.01 \space кг$
    $t_3 = 60 \degree C$

    $t — ?$

    Посмотреть решение и ответ

    Скрыть

    Решение:

    При смешивании жидкостей разных температур, мы знаем, что внутренняя энергия всех нагревающихся тел увеличивается на столько, на сколько уменьшается внутренняя энергия остывающих тел.

    Для смешивания двух жидкостей мы можем записать, что $Q_1 = — Q_2$ или $Q_1 + Q_2 = 0$.

    Сначала рассмотрим смешивание первых двух порций воды. Первая порция с температурой $15 \degree C$ будет нагреваться (получать энергию), а вторая порция с температурой $25 degree C$ будет охлаждаться (выделять энергию). Эти энергии будут численно равны друг другу, но противоположны по знаку:
    $cm_1(t_{1+2} — t_1) = — cm_2(t_{1+2} — t_2)$.

    Найдем конечную температуру этой смеси:
    $m_1(t_{1+2} — t_1) = — m_2 (t_{1+2} — t_2)$,
    $m_1 t_{1+2} — m_1 t_1 = -m_2 t_{1+2} + m_2 t_2$,
    $t_{1+2} (m_1 + m_2) = m_1 t_1 + m_2 t_2$,

    $t_{1+2} = \frac{m_1 t_1 + m_2 t_2}{m_1 + m_2} = \frac{0.02 \space кг \cdot 15 \degree C + 0.03 \space кг \cdot 25 \degree }{0.02 \space кг + 0.03 \space кг} = \frac{0.3 \space кг \cdot \degree + 0.75 \space кг \cdot \degree C}{0.05 \space кг} = 21 \degree C$.

    Так мы получили смесь первой и второй порций воды массой $m_{1+2} = 0.05 \space кг$ и температурой $t_{1+2} = 21 \degree C$.

    Теперь добавим третью порцию воды  в полученную смесь. Смесь будет нагреваться (получать энергию), а третья порция воды будет охлаждаться (выделять энергию):
    $Q_{1+2} = — Q_3$.
    $cm_{1+2} (t — t_{1+2}) = — cm_3 (t — t_3)$,
    $m_{1+2} (t — t_{1+2}) = — m_3 (t — t_3)$.

    Выразим отсюда конечную температуру смеси из трех порций воды $t$:
    $m_{1+2} t — m_{1+2} t_{1+2} = -m_3 t + m_3 t_3$,
    $t (m_{1+2} + m_3) = m_{1+2} t_{1+2} + m_3 t_3$,
    $t = \frac{m_{1+2} t_{1+2} + m_3 t_3}{m_{1+2} + m_3}$.

    Рассчитаем ее:
    $t = \frac{0.05 \space кг \cdot 21 \degree C + 0.01 \space кг \cdot 60 \degree}{0.05 \space кг + 0.01 \space кг} = \frac{1.05 \space кг \cdot \degree C + 0.6 \space кг \cdot degree C}{0.06 \space кг} = 27.5 \degree C$.

    Ответ: $t = 27.5 \degree C$.

    Задача №9 на расчет количества теплоты, рассеиваемого в окружающую среду

    Электрочайник с водой нагревается от температуры $70 \degree C$ до температуры $80 \degree C$ за $3 \space мин$, а остывает от температуры $80 \degree C$ до температуры $70 \degree C$ за $9 \space мин$. Какая часть количества теплоты, выделяемой  спиралью чайника при нагревании воды, рассеивается в окружающую среду? Тепловые потери считать постоянными.

    Внесем необходимые пояснения. Спираль чайника передает воде определенное количество теплоты $Q_2$. Часть ее ($Q_1$) рассеивается в окружающую среду. Т.е., количество теплоты $Q_2$, выделяемое спиралью, больше количества теплоты $Q$, необходимого для нагрева воды.

    Дано:
    $t_1 = 70 \degree C$
    $t_2 = 80 \degree C$
    $T_1 = 3 \space мин$
    $T_2 = 9 \space мин$

    $\frac{Q_1}{Q_2} — ?$

    Показать решение и ответ

    Скрыть 

    Решение:

    Сначала рассчитаем количество теплоты, которое необходимо сообщить воде в чайнике, чтобы ее температура увеличилась с $70 \degree C$ до $80 \degree C$:
    $Q = cm(t_2 — t_1)$.

    Масса воды в чайнике нам неизвестна, поэтому примем ее, равной $1 \space кг$. Тогда,
    $Q = 4200 \frac{Дж}{кг \cdot \degree C} \cdot 1 \space кг \cdot (80 \degree C — 70 \degree C) = 42 \space 000 \space Дж = 42 \space кДж$.

    Когда вода в чайнике остывает с температуры $80 \degree C$ до температуры $70 \degree C$, она выделяет в окружающую среду точно такое же количество энергии $Q$. Остывание происходит за $9 \space мин$. Значит, количество теплоты, которое выделяется в окружающую среду за $1 \space мин$ будет равно:
    $Q_0 = \frac{42 \space кДж}{9 \space мин} \approx 4.7 \frac{кДж}{мин}$.

    В условиях задачи сказано, что тепловые потери постоянны. Это означает, что вода массой $1 \space кг$ отдает $4.7 \space кДж$ каждую минуту, в том числе, и при ее нагревании.

    Нагревается вода за 3 минуты. За это время она отдает в окружающую среду следующее количество теплоты:
    $Q_1 = 4.7 \space кДж \cdot 3 = 14.1 \space кДж$.

    Тем не менее, чайник нагрел воду до нужной температуры. Значит, он сообщил воде количество энергии, равное $Q_2 = Q + Q_1$.
    $Q_2 = 42 \space кДж + 14.1 \space кДж = 56.1 \space кДж$.

    Теперь мы можем рассчитать отношение $\frac{Q_1}{Q_2}$, и узнать какая часть теплоты, выделяемая спиралью чайника, рассеивается в окружающую среду:
    $\frac{Q_1}{Q_2} = \frac{14.1 \space кДж}{56.1 \space кДж} \approx 0.25$.

    Т. е., в окружающую среду рассеивается $\frac{1}{4}$ часть энергии, сообщаемая воде в чайнике.

    Можно доказать, что это соотношение останется постоянным для воды любой массы в этой задаче. Чем больше будет масса воды, тем больше энергии ей будет нужно, чтобы нагреться до определенной температуры. Больше будут и тепловые потери. Искомое соотношение же останется неизменным.

    Ответ: $\frac{Q_1}{Q_2} \approx 0.25$.

    Кинематические уравнения: Примеры задач и решений

    Ранее в Уроке 6 были представлены и обсуждены четыре кинематических уравнения. Для использования с этими уравнениями была представлена ​​полезная стратегия решения проблем, и были приведены два примера, иллюстрирующие использование этой стратегии. Затем было обсуждено и проиллюстрировано применение кинематических уравнений и стратегии решения задач к свободному падению. В этой части Урока 6 будет представлено несколько примеров задач. Эти задачи позволяют любому изучающему физику проверить свое понимание использования четырех кинематических уравнений для решения задач, связанных с одномерным движением объектов. Вам предлагается прочитать каждую задачу и попрактиковаться в использовании стратегии при решении задачи. Затем нажмите кнопку, чтобы проверить ответ, или воспользуйтесь ссылкой для просмотра решения.

    Проверьте свое понимание

    1. Самолет разгоняется по взлетно-посадочной полосе со скоростью 3,20 м/с 2 в течение 32,8 с, пока, наконец, не оторвется от земли. Определить расстояние, пройденное до взлета.
    2. Автомобиль трогается с места и равномерно ускоряется за время 5,21 секунды на расстояние 110 м. Определить ускорение автомобиля.
    3. Аптон Чак едет на Гигантской капле в Большой Америке. Если Аптон будет свободно падать в течение 2,60 с, какова будет его конечная скорость и как далеко он упадет?
    4. Гоночный автомобиль равномерно ускоряется с 18,5 м/с до 46,1 м/с за 2,47 секунды. Определить ускорение автомобиля и пройденный путь.
    5. Перо падает на Луну с высоты 1,40 метра. Ускорение свободного падения на Луне равно 1,67 м/с 2 . Определите время падения пера на поверхность Луны.

      См. решение ниже.


    6. Сани с реактивным двигателем используются для проверки реакции человека на ускорение. Если сани с ракетным двигателем разгоняются до скорости 444 м/с за 1,83 секунды, то каково ускорение и какое расстояние проходят сани?
    7. Велосипед равномерно ускоряется из состояния покоя до скорости 7,10 м/с на пути 35,4 м. Определить ускорение велосипеда.
    8. Инженер проектирует взлетно-посадочную полосу для аэропорта. Из самолетов, которые будут использовать аэропорт, наименьшая скорость ускорения, вероятно, составит 3 м/с 2 . Взлетная скорость этого самолета составит 65 м/с. Предполагая это минимальное ускорение, какова минимальная допустимая длина взлетно-посадочной полосы?
    9. Автомобиль, движущийся со скоростью 22,4 м/с, останавливается за 2,55 с. Определить расстояние заноса автомобиля (считая ускорение равномерным).
    10. Кенгуру способен прыгать на высоту 2,62 м. Определить скорость взлета кенгуру.
    11. Если у Майкла Джордана вертикальный прыжок 1,29 м, то какова его скорость взлета и время зависания (общее время, чтобы подняться вверх до пика и затем вернуться на землю)?
    12. Пуля вылетает из винтовки с начальной скоростью 521 м/с. Разгоняясь через ствол винтовки, пуля проходит расстояние 0,840 м. Определить ускорение пули (ускорение считать равномерным).
    13. Бейсбольный мяч подбрасывается прямо в воздух и имеет время зависания 6,25 с. Определите высоту, на которую поднимается мяч, прежде чем он достигнет своей вершины. (Подсказка: время подъема на пик составляет половину общего времени зависания.)
    14. Смотровая площадка высотного небоскреба на высоте 370 м над ул. Определите время, за которое монета свободно упадет с палубы на улицу.

      См. решение ниже.


    15. Пуля, летящая со скоростью 367 м/с, застревает в комке влажной глины. Пуля проникает на расстояние 0,0621 м. Определить ускорение пули при движении в глину. (Предположим, что ускорение равномерное.)
    16. Камень брошен в глубокий колодец, и слышно, как он ударяется о воду через 3,41 с после падения. Определить глубину скважины.
    17. Однажды было зафиксировано, что Jaguar оставил следы заноса длиной 290 метров. Предполагая, что Ягуар занесло до остановки с постоянным ускорением -3,90 м/с 2 , определите скорость Ягуара до начала заноса.
    18. Самолет имеет скорость взлета 88,3 м/с, и для достижения этой скорости требуется 1365 м. Определить ускорение самолета и время, необходимое для достижения этой скорости.
    19. Драгстер разгоняется до скорости 112 м/с на расстоянии 398 м. Определить ускорение (предположим равномерным) драгстера.
    20. С какой скоростью в милях/ч (1 м/с = 2,23 мили/ч) должен быть брошен предмет, чтобы он достиг высоты 91,5 м (эквивалентно одному футбольному полю)? Примите пренебрежимо малое сопротивление воздуха.

    Решения вышеуказанных проблем

    1. Дано:

      а = +3,2 м/с 2

      т = 32,8 с

      v i = 0 м/с

      Найти:

      д = ??
      d = v i *t + 0,5*a*t 2

      d = (0 м/с)*(32,8 с)+ 0,5*(3,20 м/с 2 )*(32,8 с) 2

      д = 1720 м

      Вернуться к проблеме 1

       

    2. Дано:

      д = 110 м

      т = 5,21 с

      v i = 0 м/с

      Найти:

      а = ??
      d = v i *t + 0,5*a*t 2

      110 м = (0 м/с)*(5,21 с)+ 0,5*(а)*(5,21 с) 2

      110 м = (13,57 с 2 )*a

      а = (110 м)/(13,57 с 2 )

      а = 8,10 м/с 2

      Вернуться к задаче 2

       

    3. Дано:

      а = -9,8 м

      т = 2,6 с

      v i = 0 м/с

      Найти:

      д = ??

      v f = ??

      d = v i *t + 0,5*a*t 2

      d = (0 м/с)*(2,60 с)+ 0,5*(-9,8 м/с 2 )*(2,60 с) 2

      d = -33,1 м (- указывает направление)

      v f = v i + a*t

      v f = 0 + (-9,8 м/с 2 )*(2,60 с)

      v f = -25,5 м/с (- указывает направление)

      Вернуться к проблеме 3

       

    4. Дано:

      v i = 18,5 м/с

      v f = 46,1 м/с

      т = 2,47 с

      Найти:

      д = ??

      а = ??

      а = (Дельта v)/t

      а = (46,1 м/с — 18,5 м/с)/(2,47 с)

      а = 11,2 м/с 2

      д = v i *t + 0,5*a*t 2

      d = (18,5 м/с)*(2,47 с)+ 0,5*(11,2 м/с 2 )*(2,47 с) 2

      d = 45,7 м + 34,1 м

      д = 79,8 м

      (Примечание: d также можно рассчитать с помощью уравнения v f 2 = v i 2 + 2*a*d)

      Вернуться к задаче 4

       

    5. Дано:

      v i = 0 м/с

      d = -1,40 м

      а = -1,67 м/с 2

      Найти:

      т = ??
      d = v i *t + 0,5*a*t 2

      -1,40 м = (0 м/с)*(t)+ 0,5*(-1,67 м/с 2 )*(t) 2

      -1,40 м = 0+ (-0,835 м/с 2 )*(t) 2

      (-1,40 м)/(-0,835 м/с 2 ) = t 2

      1,68 с 2 = т 2

      т = 1,29 с

      Вернуться к проблеме 5

       

    6. Дано:

      v i = 0 м/с

      v f = 444 м/с

      т = 1,83 с

      Найти:

      а = ??

      д = ??

      а = (Дельта v)/t

      а = (444 м/с — 0 м/с)/(1,83 с)

      а = 243 м/с 2

      d = v i *t + 0,5*a*t 2

      d = (0 м/с)*(1,83 с)+ 0,5*(243 м/с 2 )*(1,83 с) 2

      d = 0 м + 406 м

      д = 406 м

      (Примечание: d также можно рассчитать с помощью уравнения v f 2 = v i 2 + 2*a*d)

      Вернуться к задаче 6

       


       

    7. Дано:

      v i = 0 м/с

      v f = 7,10 м/с

      д = 35,4 м

      Найти:

      а = ??
      v f 2 = v i 2 + 2*a*d

      (7,10 м/с) 2 = (0 м/с) 2 + 2*(а)*(35,4 м)

      50,4 м 2 2 = (0 м/с) 2 + (70,8 м)*a

      (50,4 м 2 2 )/(70,8 м) = а

      а = 0,712 м/с 2

      Вернуться к задаче 7

       

    8. Дано:

      v i = 0 м/с

      v f = 65 м/с

      а = 3 м/с 2

      Найти:

      д = ??
      v f 2 = v i 2 + 2*a*d

      (65 м/с) 2 = (0 м/с) 2 + 2*(3 м/с 2 )*d

      4225 м 2 2 = (0 м/с) 2 + (6 м/с 2 )*d

      (4225 м 2 2 )/(6 м/с 2 ) = d

      д = 704 м

      Вернуться к задаче 8

       

    9. Дано:

      v i = 22,4 м/с

      v f = 0 м/с

      т = 2,55 с

      Найти:

      д = ??
      d = (v i + v f )/2 *t

      d = (22,4 м/с + 0 м/с)/2 * 2,55 с

      d = (11,2 м/с)*2,55 с

      д = 28,6 м

      Вернуться к задаче 9

       

    10. Дано:

      а = -9,8 м/с 2

      v f = 0 м/с

      д = 2,62 м

      Найти:

      v i = ??
      v f 2 = v i 2 + 2*a*d

      (0 м/с) 2 = v i 2 + 2*(-9,8 м/с 2 )*(2,62 м)

      0 м 2 2 = v i 2 — 51,35 м 2 2

      51,35 м 2 2 = v i 2

      v i = 7,17 м/с

      Вернуться к проблеме 10

       

    11. Дано:

      а = -9,8 м/с 2

      v f = 0 м/с

      д = 1,29 м

      Найти:

      v i = ??

      т = ??

      v f 2 = v i 2 + 2*a*d

      (0 м/с) 2 = v i 2 + 2*(-9,8 м/с 2 )*(1,29 м)

      0 м 2 2 = v i 2 — 25,28 м 2 2

      25,28 м 2 2 = v i 2

      v i = 5,03 м/с

      Чтобы найти время зависания, найдите время до пика и удвойте его.

      v f = v i + a*t

      0 м/с = 5,03 м/с + (-9,8 м/с 2 )*t до

      -5,03 м/с = (-9,8 м/с 2 )*t до

      (-5,03 м/с)/(-9,8 м/с 2 ) = t up

      t до = 0,513 с

      время зависания = 1,03 с

      Вернуться к задаче 11

       

    12. Дано:

      v i = 0 м/с

      v f = 521 м/с

      d = 0,840 м

      Найти:

      а = ??
      против f 2 = v i 2 + 2*a*d

      (521 м/с) 2 = (0 м/с) 2 + 2*(а)*(0,840 м)

      271441 м 2 2 = (0 м/с) 2 + (1,68 м)*a

      (271441 м 2 2 )/(1,68 м) = a

      а = 1,62*10 5 м/с 2

      Вернуться к задаче 12

       

    13. Дано:

      а = -9,8 м/с 2

      v f = 0 м/с

      т = 3,13 с

      Найти:

      д = ??
      1. (ПРИМЕЧАНИЕ: время выхода на пик траектории составляет половину общего времени зависания — 3,125 с. )

      Первое использование: v f  = v i  + a*t

      0 м/с = v i  + (-9,8 м/с 2 )*(3,13 с)

      0 м/с = v i  — 30,7 м/с

      v i = 30,7 м/с (30,674 м/с)

      Теперь используйте: v f 2  = v i 2  + 2*a*d

      (0 м/с) 2  = (30,7 м/с) 2  + 2*(-9,8 м/с 2 )*(d)

      0 м 2 2 = (940 м 2 2 ) + (-19,6 м/с 2 )*d

      -940 м 2 2  = (-19,6 м/с 2 )*d

      (-940 м 2 2 )/(-19,6 м/с 2 ) = d

      д = 48,0 м

      Вернуться к задаче 13

       

    14. Дано:

      v i = 0 м/с

      d = -370 м

      а = -90,8 м/с 2

      Найти:

      т = ??
      d = v i *t + 0,5*a*t 2

      -370 м = (0 м/с)*(t)+ 0,5*(-9,8 м/с 2 )*(t) 2

      -370 м = 0+ (-4,9 м/с 2 )*(t) 2

      (-370 м)/(-4,9 м/с 2 ) = t 2

      75,5 с 2 = т 2

      т = 8,69 с

      Вернуться к задаче 14

       

       

    15. Дано:

      v i = 367 м/с

      против f = 0 м/с

      d = 0,0621 м

      Найти:

      а = ??
      v f 2 = v i 2 + 2*a*d

      (0 м/с) 2 = (367 м/с) 2 + 2*(а)*(0,0621 м)

      0 м 2 2 = (134689 м 2 2 ) + (0,1242 м)*a

      -134689 м 2 2 = (0,1242 м)*a

      (-134689 м 2 2 )/(0,1242 м) = а

      а = -1,08*10 6 м/с 2

      (Знак — означает, что пуля замедлилась. )

      Вернуться к задаче 15

       

    16. Дано:

      а = -9,8 м/с 2

      т = 3,41 с

      v i = 0 м/с

      Найти:

      д = ??
      d = v i *t + 0,5*a*t 2

      d = (0 м/с)*(3,41 с)+ 0,5*(-9,8 м/с 2 )*(3,41 с) 2

      d = 0 м+ 0,5*(-9,8 м/с 2 )*(11,63 с 2 )

      d = -57,0 м

      (ПРИМЕЧАНИЕ: знак — указывает направление)

      Вернуться к задаче 16

       

    17. Дано:

      а = -3,90 м/с 2

      v f = 0 м/с

      д = 290 м

      Найти:

      в и = ??
      v f 2 = v i 2 + 2*a*d

      (0 м/с) 2 = v i 2 + 2*(-3,90 м/с 2 )*(290 м)

      0 м 2 2 = v i 2 — 2262 м 2 2

      2262 м 2 2 = v i 2

      v i = 47,6 м/с

      Вернуться к задаче 17

       

    18. Дано:

      v i = 0 м/с

      v f = 88,3 м/с

      д = 1365 м

      Найти:

      а = ??

      т = ??

      v f 2 = v i 2 + 2*a*d

      (88,3 м/с) 2 = (0 м/с) 2 + 2*(а)*(1365 м)

      7797 м 2 2 = (0 м 2 2 ) + (2730 м)*a

      7797 м 2 2 = (2730 м)*a

      (7797 м 2 2 )/(2730 м) = а

      а = 2,86 м/с 2

      v f = v i + a*t

      88,3 м/с = 0 м/с + (2,86 м/с 2 )*t

      (88,3 м/с)/(2,86 м/с 2 ) = t

      t = 30,8 с

      Вернуться к задаче 18

       

    19. Дано:

      v i = 0 м/с

      v f = 112 м/с

      д = 398 м

      Найти:

      а = ??
      v f 2 = v i 2 + 2*a*d

      (112 м/с) 2 = (0 м/с) 2 + 2*(а)*(398 м)

      12544 м 2 2 = 0 м 2 2 + (796 м)*a

      12544 м 2 2 = (796 м)*a

      (12544 м 2 2 )/(796 м) = а

      а = 15,8 м/с 2

      Вернуться к проблеме 19

       

    20. Дано:

      а = -9,8 м/с 2

      v f = 0 м/с

      д = 91,5 м

      Найти:

      v i = ??

      т = ??

      Сначала найдите скорость в м/с:

      v f 2 = v и 2 + 2*а*д

      (0 м/с) 2 = v i 2 + 2*(-9,8 м/с 2 )*(91,5 м)

      0 м 2 2 = v i 2 — 1793 м 2 2

      1793 м 2 2 = v i 2

      v i = 42,3 м/с

      Теперь конвертируем из м/с в мили/ч:

      v i = 42,3 м/с * (2,23 мили/ч)/(1 м/с)

      v i = 94,4 миль/ч

      Вернуться к задаче 20

     

    Следующий раздел:

    Наборы задач на работу, энергию и мощность

    Калькулятор, версия 2

    Вы просматриваете устаревшую версию Калькулятора. Недавно мы переработали и улучшили Калькулятор.  Версия 2 теперь ЖИВАЯ!  Мы увеличили количество задач более чем в три раза, разбили каждую часть на несколько небольших однотематических наборов задач и использовали генератор случайных чисел для предоставления числовой информации по каждой задаче. Ответы учащихся оцениваются автоматически, а обратная связь осуществляется мгновенно. И мы сохранили такое же обязательство предоставлять помощь через ссылки на существующие ресурсы. В то время как БЕСПЛАТНАЯ версия делает все вышеперечисленное, учителя с подпиской на Task Tracker могут пойти еще дальше. Они могут модифицировать наши готовые наборы задач, писать свои собственные задачи с помощью нашего простого в использовании Конструктора задач и использовать планшет для разработки собственной программы, выражающей их акцент на использовании математики в физике.

    Вернитесь на главную страницу, чтобы перейти к Версии 2. Узнайте больше о Версии 2. Или посетите Магазин, чтобы совершить покупку в системе отслеживания задач.

    Работа, энергия и мощность: набор задач

    Задача 1:

    Ренатта Гасс гуляет со своими друзьями. Происходит несчастье, и Ренатта и ее друзья обнаруживают, что получают работу . Они прикладывают совокупную силу 1080 Н, чтобы толкнуть автомобиль на 218 м до ближайшей заправочной станции. Определить работу, совершенную автомобилем.

    • Решение со звуковым сопровождением
    Задача 2:

    Ганс Фулл тянет за веревку, чтобы тащить свой рюкзак в школу по льду. Он тянет вверх и вправо с силой 22,9 ньютона под углом 35 градусов над горизонталью, чтобы протащить свой рюкзак на горизонтальное расстояние 129 метров вправо. Определить работу (в джоулях), совершенную над рюкзаком.

    • Аудиогид
    Задача 3:

    Ламар Гант, звезда американского пауэрлифтинга, стал первым человеком, поднявшим становую тягу, в пять раз превышающую собственный вес, за 19 лет.85. Становая тяга предполагает подъем нагруженной штанги с пола в положение над головой на вытянутых руках. Определить работу, совершенную Ламаром при подъеме становой тяги 300 кг на высоту 0,90 м над землей.

    • Аудиогид
    Задача 4:

    Шейла только что прибыла в аэропорт и тащит свой чемодан к стойке регистрации багажа. Она тянет за лямку с силой 190 Н под углом 35° к горизонтали, чтобы сместить ее на 45 м к столу. Определите работу, проделанную Шейлой над чемоданом.

    • Аудиогид
    Задача 5:

    Во время подготовки к сезону размножения 380-граммовый самец белки делает 32 отжимания в минуту, смещая свой центр масс на расстояние 8,5 см при каждом отжимании. Определить общую работу, совершенную белкой при движении вверх (32 раза).

    • Аудиогид
    Задача 6:

    Во время лаборатории Powerhouse Джером бежит вверх по лестнице, поднимая свое 102-килограммовое тело на расстояние 2,29 м по вертикали.метров за 1,32 секунды с постоянной скоростью.

    а. Определите работу Джерома при подъеме по лестнице.
    б. Определите мощность, генерируемую Джеромом.

    • Аудиогид
    Задача 7:

    В новой конвейерной системе на местном заводе по упаковке будет использоваться механический рычаг с приводом от двигателя, который будет прилагать среднюю силу 890 Н для толкания больших ящиков на расстояние 12 метров за 22 секунды. Определить мощность, необходимую для такого двигателя.

    • Аудиогид
    Задача 8:

    Taipei 101 на Тайване — это 101-этажный небоскреб высотой 1667 футов. Небоскреб является домом для самого быстрого лифта в мире. Лифты доставляют посетителей с первого этажа на смотровую площадку на 89-м этаже со скоростью до 16,8 м/с. Определите мощность, развиваемую двигателем, чтобы поднять 10 пассажиров с этой скоростью. Суммарная масса пассажиров и салона составляет 1250 кг.

    • Аудиогид
    Задача 9:

    На лыжных трассах горы Блюберд сноубордисты и лыжники поднимаются на вершину холма с помощью буксирных тросов. Один из буксирных тросов приводится в действие двигателем мощностью 22 кВт, который тянет лыжников по обледенелому склону 14° с постоянной скоростью. Предположим, что 18 лыжников со средней массой 48 кг держатся за веревку и предположим, что мотор работает на полную мощность.

    а. Определите совокупный вес всех этих лыжников.
    б.  Определите силу, необходимую для того, чтобы подтянуть этот груз вверх под углом 14° с постоянной скоростью.
    г. Определите скорость, с которой лыжники будут подниматься в гору.

    • Аудиогид
    Задача 10:

    Первый открытый астероид — Церера. Это самый большой и самый массивный астероид в поясе астероидов нашей Солнечной системы, имеющий расчетную массу 3,0 x 10 21 кг и орбитальную скорость 17900 м/с. Определите количество кинетической энергии, которой обладает Церера.

    • Аудиогид
    Задача 11:

    Кинетическая энергия велосипеда равна 124 Дж. Какой кинетической энергией был бы велосипед, если бы он имел …

    a. … в два раза больше массы и двигался с той же скоростью?
    б. … той же массы, но двигался с удвоенной скоростью?
    г. … вдвое меньше массы и двигался с удвоенной скоростью?
    д. … такая же масса и двигалась с половинной скоростью?
    г. н.э. … в три раза больше массы и двигался с половиной скорости?

    • Аудиогид
    Задача 12:

    Парашютист массой 78 кг имеет скорость 62 м/с на высоте 870 м над землей.

    а. Определите кинетическую энергию парашютиста.
    б. Определите потенциальную энергию парашютиста.
    г. Определите полную механическую энергию парашютиста.

    • Аудиогид
    Задача 13:

    Ли Бен Фардест (уважаемый американский прыгун с трамплина) имеет массу 59,6 кг. Он движется со скоростью 23,4 м/с на высоте 44,6 метра над землей. Определить полную механическую энергию Ли Бен Фардеста.

    • Аудиогид
    Задача 14:

    Хлоя возглавляет университетскую команду по софтболу Саута по ударам. В игре против New Greer Academy в минувшие выходные Хлоя так сильно ударила по 181-граммовому софтболу, что тот перелетел забор и приземлился на Лейк-авеню. В какой-то момент своей траектории мяч находился на высоте 28,8 м над землей и двигался со скоростью 190,7 м/с. Определить полную механическую энергию мяча.

    • Аудиогид
    Задача 15:

    Олив Удади в парке со своим отцом. Оливка весом 26 кг качается на качелях, как показано на рисунке. Олива имеет скорость 0 м/с в точке А и находится на высоте 3,0 м над землей. В позиции B Олив находится на высоте 1,2 м над землей. В положении C (2,2 м над землей) Оливка высовывается из сиденья и летит как снаряд по показанной траектории. В точке F Олив всего лишь пикометров над землей. Примите пренебрежимо малое сопротивление воздуха во время движения. Используйте эту информацию для заполнения таблицы.

    Должность Высота (м) Полиэтилен (J) КЭ (Дж) TME (J) Скорость (м/с)
    А 3,0       0,0
    Б 1,2        
    С 2,2        
    Ф 0        
    • Аудиогид
    Задача 16:

    Сьюзи Лавтаски (м=56 кг) катается на лыжах на горе Блуберд. Она движется со скоростью 16 м/с по гребню лыжной горки, расположенной на высоте 34 м над уровнем земли в конце трассы.

    а. Определите кинетическую энергию Сьюзи.
    б. Определите потенциальную энергию Сьюзи относительно высоты земли в конце бега.
    г. Определите полную механическую энергию Сьюзи на вершине холма.
    д. Если между вершиной холма и ее первоначальным прибытием в конце забега энергия не теряется и не приобретается, то какова будет полная механическая энергия Сьюзи в конце забега?
    эл. Определите скорость Сьюзи в момент ее прибытия к концу пробега и перед торможением до полной остановки.

    • Аудиогид
    Задача 17:

    Николас находится в парке развлечений «Ноев ковчег» и готовится покататься на гоночной горке «Точка невозврата». В верхней части горки Николай (м=72,6 кг) находится на высоте 28,5 м над землей.

    а. Определите потенциальную энергию Николаса в верхней части слайда.
    б. Определите кинетическую энергию Николаса в верхней части горки.
    г. Предполагая пренебрежимо малые потери энергии между вершиной горки и его подходом к нижней части горки (h=0 м), определите полную механическую энергию Николаса, когда он достигает нижней части горки.
    д. Определите потенциальную энергию Николаса, когда он достигнет нижней части слайда.
    эл. Определите кинетическую энергию Николаса, когда он достигает нижней части горки.
    ф. Определите скорость Николаса, когда он достигнет нижней точки горки.

    • Аудиогид
    Задача 18:

    Има Скаарред (m=56,2 кг) движется со скоростью 12,8 м/с на вершине петли американских горок высотой 19,5 м.

    г. н.э. Определите кинетическую энергию Имы в верхней части петли.
    б. Определите потенциальную энергию Имы в верхней части петли.
    г. Предполагая незначительные потери энергии из-за трения и сопротивления воздуха, определите полную механическую энергию Имы в нижней части петли (h=0 м).
    д. Определите скорость Имы в конце петли.

    • Аудиогид
    Задача 19:

    Джастин Тайм едет по Лейк-авеню со скоростью 32,8 м/с на своем 1510-килограммовом автомобиле 19.92 Камаро. Он замечает полицейскую машину с радаром и быстро снижает скорость до разрешенной 20,1 м/с.

    а. Определите начальную кинетическую энергию Камаро.
    б. Определите кинетическую энергию Camaro после замедления.
    г. Определите объем работы, проделанной Camaro во время торможения.

    • Аудиогид
    Задача 20:

    Пит Зариа работает по выходным в пиццерии Барнаби. Его основная обязанность — выполнять заказы на напитки для клиентов. Он наполняет кувшин колой, ставит его на прилавок и толкает кувшин весом 2,6 кг вперед с усилием 8,8 Н на расстояние 48 см, чтобы отправить его покупателю в конце прилавка. Коэффициент трения между кувшином и столешницей равен 0,28.

    а. Определите работу Пита над кувшином при толчке на 48 см.
    б. Определите работу трения о кувшин.
    г. Определите полную работу, совершенную над кувшином.
    д. Определите кинетическую энергию кувшина, когда Пит толкает его.
    эл. Определите скорость кувшина, когда Пит толкает его.

    • Аудиогид

     

    Проблема 21:

    Стратакоастер Top Thrill Dragster в парке развлечений Сидар-Пойнт в Огайо использует гидравлическую систему запуска, чтобы разогнать райдеров от 0 до 53,6 м/с (120 миль/ч) за 3,8 секунды перед подъемом на полностью вертикальный 420-футовый холм.

    а. Джером (м=102 кг) посещает парк со своей церковной молодежной группой. Он садится в машину, пристегивается ремнями и готовится к волнениям дня. Какова кинетическая энергия Джерома до периода ускорения?
    б. 3,8-секундный период ускорения начинает разгонять Джерома по ровной трассе. Какова кинетическая энергия Джерома в конце этого периода ускорения?
    г. Когда запуск завершен, Джером начинает кричать на 420-футовом, полностью вертикальном участке трассы. Определить потенциальную энергию Джерома в верхней части вертикального сечения. ( ДАННО : 1,00 м = 3,28 фута)
    d. Определите кинетическую энергию Джерома в верхней части вертикального сечения.
    эл. Определите скорость Джерома в верхней части вертикального сечения.

    • Аудиогид
    Задача 22:

    Пейдж — самый высокий игрок в волейбольной команде Университета Юга. Она находится в пиковом положении, когда Джулия дает ей идеальный набор. Волейбольный мяч массой 0,226 кг находится на высоте 2,29 м над землей и имеет скорость 1,06 м/с. Пейдж бросает мяч, совершая над ним работу 9,89 Дж.

    а. Определить потенциальную энергию мяча до того, как Пейдж вонзит в него шип.
    б. Определить кинетическую энергию мяча до того, как Пейдж ударит его шипом.
    г. Определить полную механическую энергию мяча до того, как Пейдж вонзит в него шип.
    д. Определите полную механическую энергию мяча при ударе об пол на стороне соперника.
    эл. Определите скорость мяча при ударе об пол на стороне соперника.

    • Аудиогид
    Задача 23:

    Согласно шоу ABC Wide World of Sports, есть радость победы и агония поражения. 21 марта 1970 года Винко Богатай был югославским участником чемпионата мира, проходившего в бывшей Западной Германии. К его третьему и последнему прыжку дня сильный и стойкий снег создал опасные условия на склоне. В середине бега Богатай осознал опасность и попытался внести коррективы, чтобы прекратить свой прыжок. Вместо этого он потерял равновесие, кувыркнулся и кувыркнулся со склона в плотную толпу. В течение почти 30 лет после этого кадры этого события были включены во вступление к печально известному спортивному шоу ABC, и Винко стал известен как 9-й.1808 агония поражения икона.

    а. Определить скорость Винко массой 72 кг после того, как он спустился на лыжах с горы на высоту, находящуюся на 49 м ниже точки старта.
    б. Спустившись с высоты 49 м, Винко скатился с трассы и спустился еще на 15 м вниз по склону, прежде чем наконец остановился. Определить изменение потенциальной энергии Винко от вершины холма до точки, в которой он останавливается.
    г. Определите суммарную работу тела Винко, когда он останавливается.

    • Аудиогид
    Задача 24:

    У Нолана Райана, как сообщается, была самая быстрая подача в бейсболе, разогнал со скоростью 100,9 миль / ч (45,0 м / с). Если бы такая подача была направлена ​​​​вертикально вверх с той же скоростью, то на какой высоте оно путешествовало?

    • Аудиогид
    Задача 25:

    В лаборатории «Энергия наклона» партнеры Анна Литикал и Ноа Формула придают тележке весом 1,00 кг начальную скорость 2,35 м/с с высоты 0,125 м над лабораторным столом. Определить скорость тележки, когда она находится на высоте 0,340 м над лабораторным столом.

    • Аудиогид
    Задача 26:

    В апреле 1976 года отбивающий из «Чикаго Каб» Дэйв Кингман совершил хоум-ран, перепрыгнув забор «Ригли Филд» и поразив дом, расположенный в 530 футах (162 м) от домашней площадки. Предположим, что бейсбольный мяч весом 0,145 кг вылетел из биты Кингмана со скоростью 92,7 м/с и потерял 10 % своей первоначальной энергии при полете по воздуху. Определить скорость мяча, когда он пересек стену стадиона на высоте 25,6 м.

    • Аудиогид
    Задача 27:

    Диззи мчится со скоростью 22,8 м/с, приближаясь к ровному участку пути рядом с погрузочной площадкой американских горок Whizzer. Тормозная система резко разгоняет 328-килограммовый автомобиль (включая массу гонщика) до скорости 2,9 м/с на дистанции 5,55 метра. Определите тормозную силу, действующую на автомобиль Диззи.

    • Аудиогид
    Задача 28:

    Сани массой 6,8 кг толкают по замерзшему пруду так, что они приобретают скорость 1,9РС. Коэффициент трения между прудом и санями равен 0,13. Определите расстояние, которое санки скользят до остановки.

    • Аудиогид
    Задача 29:

    Коннор (масса тела 76,0 кг) участвует в чемпионате штата по прыжкам в воду. Он покидает трамплин с высоты 3,00 м над поверхностью воды со скоростью 5,94 м/с в направлении вверх.
    а.  Определить скорость Коннора в момент удара о воду.
    б.  Тело Коннора погружается на глубину 2,15 м ниже поверхности воды, прежде чем остановиться. Определите среднюю силу сопротивления воды, которую испытывает его тело.

    • Аудиогид
    Задача 30:

    Гвен присматривает за семьей Паркеров. Она берет 3-летнюю Эллисон в соседний парк и усаживает ее на детские качели. Гвен тянет 1,8-метровую цепь назад, образуя угол 26° с вертикалью, и отпускает 14-килограммовую Эллисон (включая поворотный груз). Предполагая, что трение и сопротивление воздуха пренебрежимо малы, определите скорость Эллисон в самой нижней точке траектории.

    • Аудиогид
    Задача 31:

    Шейла (масса тела 56,8 кг) в своих санях-тарелках движется со скоростью 12,6 м/с у подножия холма для катания на санках у озера Блюберд. Она подходит к длинной насыпи, наклоненной вверх под углом 16° над горизонтом. Когда она скользит вверх по насыпи, она сталкивается с коэффициентом трения 0,128. Определите высоту, на которую она поднимется перед тем, как остановиться.

    • Аудиогид
    Задача 32:

    Мэтью стартует с места на вершине холма для катания на санях высотой 8,45 м. Он скользит вниз по 32-градусному склону и пересекает плато у его основания. Коэффициент трения между санями и снегом равен 0,128 как для холма, так и для плато.

Как считать логарифмы lg: Калькулятор десятичный логарифм

Помогите решить / разобраться (М)

Сообщения без ответов | Активные темы | Избранное


Правила форума


Посмотреть правила форума


 
WinterPrimat 

 Запись логарифмического ответа в пособиях

26.02.2023, 13:37 

02/01/23
36

Читаю сейчас пособие Вентцель по теории вероятности и статистике.
К одному из примеров дан следующий ответ

Но можно же и так
И я такое видел уже сколько раз, по большей части в пособиях прошлого века. Отдается предпочтение именно записи при помощи десятичного логарифма.
Есть для такого конкретная причина?


   

                  

wrest 

 Re: Запись логарифмического ответа в пособиях

26.02.2023, 14:14 

05/09/16
10362

WinterPrimat в сообщении #1583373 писал(а):

Отдается предпочтение именно записи при помощи десятичного логарифма.

Отношение логарифмов не зависит от основания, в смысле

Поэтому десятичные там или ещё какие неважно.
Из практических соображений, на логарифмических линейках имеется шкала десятичных логарифмов, в этом смысле они удобнее. В таблицах (типа таблиц Брадиса) по десятичным логарифмам больше информации — есть десятичные логарифмы синусов/косинусов. По натуральным только сами логарифмы. Ну и для калькуляторов даже. Там тоже логарифм по произвольному основанию считать только через частное логарифмов.


   

                  

Gagarin1968 

 Re: Запись логарифмического ответа в пособиях

26. 02.2023, 14:18 

01/11/14
1261
Principality of Galilee

WinterPrimat в сообщении #1583373 писал(а):

Читаю сейчас пособие Вентцеля по теории вероятности и статистике

Правильно — пособие Вентцель.
Дело в том, что Елена Сергеевна — женщина, и не знать этого просто неприлично.

WinterPrimat в сообщении #1583373 писал(а):

Отдается предпочтение именно записи при помощи десятичного логарифма.
Есть для такого конкретная причина?

Есть, конечно — усовершенствование вычислительной техники к концу ХХ века. Оно и убило (почти) десятичные логарифмы.


   

                  

WinterPrimat 

 Re: Запись логарифмического ответа в пособиях

26.02.2023, 14:33 

02/01/23
36

Gagarin1968 в сообщении #1583384 писал(а):

Дело в том, что Елена Сергеевна — женщина, и не знать этого просто неприлично.

Спасибо. Я только неделю назад вообще узнал о таком пособии.


   

                  

Ende 

 Posted automatically

26.02.2023, 14:35 

Супермодератор

02/02/19
1119

 i Тема перемещена из форума «Математика (общие вопросы)» в форум «Помогите решить / разобраться (М)»
Причина переноса: темы, в которых нужно что-то объяснить или подсказать, создаются в этом разделе.


   

                  

Евгений Машеров 

 Re: Запись логарифмического ответа в пособиях

26.02.2023, 20:11 

Заслуженный участник

11/03/08
8673
Москва

Я бы начал с того, что Елена Сергеевна ещё и полковник, профессор военной академии. И обращён материал к её слушателям, практикам. скажем. офицерам-расчётчикам, а не математикам. восхищающимся изяществом формулы. По ней надо получить конкретное значение. Для тех лет, когда создавалось пособие — у расчётчика была логарифмическая линейка, а на ней шкала десятичных логарифмов. Сейчас есть калькулятор, где тоже есть десятичные логарифмы, есть натуральные. А по произвольному основанию нет, по нему надо считать по приведенной ею формуле, или по аналогичной, с заменой lg на ln.


   

                  

svv 

 Re: Запись логарифмического ответа в пособиях

27.02.2023, 23:52 

Заслуженный участник

23/07/08
9919
Crna Gora

А я ещё в старших классах прочитал её повесть «Кафедра», которую она написала под своим литературным псевдонимом И. Грекова. Лишь гораздо позже узнал, что это псевдоним, и что её настоящее имя — Елена Сергеевна Вентцель. (Более того, думал, что она Ирина Грекова; на самом деле псевдоним образован от «игрек».)


   

                  

Евгений Машеров 

 Re: Запись логарифмического ответа в пособиях

28.02.2023, 11:58 

Заслуженный участник

11/03/08
8673
Москва

Повесть хороша, но настоящий драматизм в

Цитата:

Вентцель Е. С. Элементарный курс теории вероятностей в применении к задачам стрельбы и бомбометания. — М.: Военная Воздушная Краснознамённая ордена Ленина академия им. Жуковского, 1945.


   

                  

Gagarin1968 

 Re: Запись логарифмического ответа в пособиях

28.02.2023, 13:20 

01/11/14
1261
Principality of Galilee

Евгений Машеров в сообщении #1583709 писал(а):

Повесть хороша, но настоящий драматизм в

Цитата:

Вентцель Е. С. Элементарный курс теории вероятностей в применении к задачам стрельбы и бомбометания. — М.: Военная Воздушная Краснознамённая ордена Ленина академия им. Жуковского, 1945.

Ну зачем же такие крайности? Вон у меня на полке стоит вполне приемлемый учебник, да и годом издания поновее. Там о поражении наземных целей совсем чуть-чуть.

Скорее всего, ТС имел в виду именно его?!


   

                  

Показать сообщения за: Все сообщения1 день7 дней2 недели1 месяц3 месяца6 месяцев1 год Поле сортировки АвторВремя размещенияЗаголовокпо возрастаниюпо убыванию 
  Страница 1 из 1
 [ Сообщений: 9 ] 

Модераторы: Модераторы Математики, Супермодераторы



Кто сейчас на конференции

Сейчас этот форум просматривают: нет зарегистрированных пользователей


Вы не можете начинать темы
Вы не можете отвечать на сообщения
Вы не можете редактировать свои сообщения
Вы не можете удалять свои сообщения
Вы не можете добавлять вложения

Найти:

Как пользоваться логарифмической линейкой?

Не стоит забывать, что именно с помощью логарифмической линейки человек впервые ступил на Луну.

Теги:

интересное

Unsplash

Логарифмическая линейка — это универсальный счетный прибор, который применялся для умножения, деления, возведения в квадрат и куб, вычисления квадратных и кубических корней, синусов, тангенсов и других значений. До появления калькуляторов, компьютеров и смартфонов инженеры носили логарифмические линейки на поясе, а линейка «Pickett» даже полетела на Луну вместе с космонавтами.

Содержание статьи

Уильям Отред — изобретатель логарифмической линейки

Уильям Отред, выпускник Итонской школы и Кембриджского королевского колледжа, пастор церкви в Олсбери в графстве Суррей, был страстным математиком и с удовольствием преподавал любимый предмет многочисленным ученикам, с которых не брал никакой платы. «Маленького роста, черноволосый и черноглазый, с проницательным взглядом, он постоянно что-то обдумывал, чертил какие-то линии и диаграммы в пыли, — так описывал Отреда один из биографов.  — Когда ему попадалась особенно интересная математическая задача, бывало, что он не спал и не ел, пока не находил ее решения». Он является первым изобретателем логарифмической линейки.

youtube

Нажми и смотри

История изобретения

В 1631 году Отред опубликовал главный труд своей жизни — учебник Clavis Mathematicae («Ключ математики»), выдержавший несколько переизданий на протяжении почти двух веков. Однажды, обсуждая «механические вычисления» с помощью линейки Гюнтера со своим учеником Уильямом Форстером, Отред отметил несовершенство этого метода. Между делом учитель продемонстрировал свое изобретение — несколько концентрических колец с нанесенными на них логарифмическими шкалами и двумя стрелками.

Форстер был восхищен и позднее писал: «Это превосходило любой из инструментов, которые были мне известны. Я удивлялся, почему он скрывал это полезнейшее изобретение многие годы…» Сам Отред говорил, что он «просто изогнул и свернул шкалу Гюнтера в кольцо», и к тому же был уверен, что «настоящее искусство [математики] не нуждается в инструментах. ..», их использование он считал допустимым только после овладения этим искусством. Однако ученик настоял на публикации, и в 1632 году Отред написал (на латыни), а Форстер перевел на английский брошюру «Круги пропорций и горизонтальный инструмент», где была описана логарифмическая линейка.

Споры об авторстве

Авторство этого изобретения оспаривал другой его ученик — Ричард Деламэйн, опубликовавший в 1630  году книгу «Граммелогия, или Математическое кольцо». Некоторые утверждают, что он просто украл изобретение счетной линейки у учителя, но возможно, он пришел к похожему решению независимо. Еще один претендент на авторство  - лондонский математик Эдмунд Уингейт, предложивший в 1626 году использовать две линейки Гюнтера, скользящие друг относительно друга. До современного состояния инструмент довели Роберт Биссакер, сделавший линейку прямой (1654), Джон Робертсон, снабдивший ее бегунком (1775), и Амеде Маннгейм, оптимизировавший расположение шкал и бегунка.

РЕКЛАМА – ПРОДОЛЖЕНИЕ НИЖЕ

Логарифмическая линейка значительно облегчила сложные вычисления для инженеров и ученых. В XX веке до появления калькуляторов и компьютеров логарифмическая линейка была таким же символом инженерных специальностей, каким для врачей является фонендоскоп.

Как пользоваться логарифмической линейкой

Рассмотрим, как проводить базовые математические операции с помощью логарифмической линейки. Принцип ее действия основан на том, что умножение и деление чисел заменяется соответственно сложением и вычитанием их логарифмов.

Сложение

Представим, что нам нужно найти сумму двух и четырех. На одной линейке (нижней) откладываем два деления (на рисунке отрезок а), вторую линейку (верхнюю) сдвигаем вправо на эти же два деления, после чего откладываем на ней еще четыре деления (отрезок b на рисунке). Смотрим на нижней линейке, над каким числом находится точка, в которую мы пришли — это шесть.

Умножение

Для начала введем переменные: a ∙ b = с при a = 2, b = 3. Затем возведем в логарифм обе части равенства и получим Lg(a) + lg(b )= lg(с). Взяв две линейки с логарифмическими шкалами, увидим, что сложение значений lg2 и lg3 дает в результате lg6, то есть произведение 2 на 3.

На основной шкале корпуса линейки (вторая снизу) выбираем первый сомножитель и на него устанавливаем начало основной, нижней, шкалы  движка (она на лицевой стороне последнего и точно такая же, как основная шкала корпуса).

Затем на основной шкале движка волосок бегунка устанавливается на втором сомножителе. На основной шкале корпуса линейки под волоском смотрим ответ. Если при этом волосок выходит за пределы шкалы, то на первый сомножитель устанавливают не начало, а конец движка (с числом 10).

Деление

Пусть a/b = с при a = 8, b = 4. Возведем в логарифм обе части равенства и тогда получим: Lg(a) – lg(b) = lg(с). Разность логарифмов делимого и делителя дает логарифм частного, в нашем случае — 2. 

На основной шкале корпуса линейки выбирается делимое, на которое устанавливается волосок бегунка. Под волосок подводится делитель, найденный на основной шкале движка. Результат определяется на основной шкале корпуса напротив начала или конца движка.

Читайте также:

Зачем нужен безель в наручных часах?

Невероятные механические головоломки от инженера-энтузиаста: мир шестеренок

Счет, аппроксимация, временная сложность и большое арифметическое

CSC300: счет, аппроксимация, временная сложность и большое арифметическое и возведение в степень [2/9] Счет и логарифмирование [3/9] Забавные факты [4/9] Обычные числа [5/9]/Обычные числа [ 9] Смена базы [7/9] приближение [8/9] Порядок роста [9/9]

(нажмите здесь на один слайд на страницу)


[1/9]

В двух частях

Открытый список воспроизведения

Открытый плейлист

. Счет и эксплуатация [2/910 = 1024
Счет и логарифм [3/9]

Все мы привыкли считать, прибавляя единицу (арифметический ряд):

0 1 2 3 4 5 6 7 ... Н
 

Также можно считать умножением на два (геометрический ряд):

1 2 4 8 16 32 64 128 ... Н
 

Существует обратная операция, называемая логарифмом . Люди часто пишут lg по основанию 2 логарифма.

     lg(1) = 0 lg(тысячи) ~ 10
     lg(2) = 1 lg(миллион) ~ 20
     lg(4) = 2 lg(миллиард) ~ 30
     lg(8) = 3 lg(триллион) ~ 40
    lg(16) = 4 lg(квадриллион) ~ 50
    lg(32) = 5 lg(квинтиллион) ~ 60
    lg(64) = 6
   lg(128) = 7
   lg(256) = 8
   lg(512) = 9
  lg(1024) = 10
 
Интересные факты [4/9]

Они эквивалентны: 960 ~ 16 квинтиллионов (на самом деле 18_446_744_073_709_551_616)

Общие номера [6/9]

С

  lg(N*M) = lg(N) + lg(M)
 

у нас есть:

  lg(4 миллиарда) ~ lg(4) + lg(миллиард) = 2 + 30
  lg(16 квинтиллионов) ~ lg(16) + lg(квинтиллионов) = 4 + 60
 
Замена основания [7/9]

Вообще логи могут иметь любую базу. Как правило, мы пишем log_b , где b — база.

  log_b(N) = log_a(N) / log_a(b)
 

Например, где lg(n) = log_2(n) :

  log_1024(N) = lg(N)/10 --- так как lg(1024) = 10
 

Другой пример, где log(n) = log_10(n) :

  log(N) ~ lg(N)/3 --- поскольку lg(10) = 3,32192809489
 

Кому-то может быть полезно это видео:

Открыть список воспроизведения

Вот расслабляющее введение в логарифмы, которое может вам понравиться:

Открыть список воспроизведения

Приблизительное значение [8/9]

Часто детали не важны

Мы пишем f(n) ~ g(n) , если f и г более или менее то же самое

  • См. Раздел 1.4 Алгоритмов для формального определения.
  • Грубо говоря, они соответствуют одной кривой

Мы говорим, что f(n) равно O(g(n)) , если f(n) ~ a * g(n) , для некоторой константы a

  • Звонил порядок роста
  • Вызывается сложность времени
  • Вызывается Большой О
Порядок возрастания [9/9]
Имя Функция Интуиция
Константа О(1) Независимость от данных
Логарифмический О (лг (н)) Итеративно разрезается пополам
Линейный О(н) Повторить один раз для каждого элемента
Линейно-арифмический О(п * лг(п)) Вложенная итерация: один раз линейная, один раз логарифмическая
Квадратичный О(n^2) 9п) Комбинации
Факториал О(н!) Перестановки

Изображение из 8 временных сложностей, которые должен решить каждый программист знать Адриан Мехиа


Пересмотрено: 17. 03.2008 13:01

Экспоненты и логарифмы

Экспоненты и логарифмы
 
Надеюсь, вы оцените это модуль называется «научным», хотя на самом деле его следует называть «базовый», если не «элементарный». Ну, никто не идеален, включая меня. Я ничего не знаю об организованных видах спорта или знаменитостях вне науки, например. Итак, я объясню экспоненты и логарифмы вам, как вы бы объяснили мне основы бейсбола.
Я перейду от чрезвычайно элементарная математика к более тяжелым вещам. Испытайте себя немного и посмотрите, как далеко вы получать.
 
  Полномочия Десять
Если вам нужно иметь дело с большими такие цифры, как американский дефицит (около 14 трлн долларов или 14 000 000 000 000 $), количество раз, когда ваши дети спрашивали «почему» (3,5 миллиарда), или количество атомов в мече (около 10 000 000 000 000 000 000 000 000), получается утомительно следить за всеми нулями. Не говоря уже о том, что вы склонны к делать ошибки.
Так человечество впервые изобрело экспоненциальных нотация просто как простой способ сокращения больших чисел. Только запишите количество нулей как «показатель степени» числа десять, поставив это позади 10 более мелким шрифтом. 100 = 10 2 , 100.000 = 10 5 .
Точно так же вы можете интерпретировать показатель как как сколько раз вы умножаете на основание 10 сам с собой. 10 3 = 10 · 10 · 10 = 1000 . Это просто. Цифры выше, например, преобразуются в:
  • Американский дефицит: около 14 триллионов долларов или 14 · 10 12 = 1,4 · 10 13 .
  • Количество раз, когда ваши дети спрашивали «почему»: 3,5 gazillion = 10 © .
  • Количество атомов в мече: около 10 25 .
Пока так легко. Мы придумали простой способ сокращения написания большие числа.
Сюрприз! Оказывается, экспоненциальное представление намного мощнее, чем вы могли себе представить в самых смелых мечтах. Давайте посмотрите на первое и пока довольно простое свойство экспоненциального обозначения:
1. Это также отличный способ выразить очень маленький числа. Маленькое число SN всегда можно записать как 1 разделить на большое число LN , SN = 1/LN . SN 0,0000001 банка можно записать как 0,0000001 = 1/10.000,000 = 1/10 7 = 10 7 .
Правильно: напишите экспоненту со знаком минус , и теперь вы снова считаете нули, но в знаменателе дроби. Конечно, вы теперь теряете простодушие интерпретация значения показателя степени, ну и что.
2. Это на самом деле отличный способ выразить любой номер . Все, что нам нужно понять, это то, что у нас есть 1 = 10 0 . Любое число теперь можно записать как a · 10 x Рассмотрим примеры:
  • 3,14 = 3,14 · 10 0 .
  • 31,400 = 3,14 · 10 4 .
  • 0,000314 = 3,14 · 10 4 .
3. Далее мы обнаруживаем, что у нас есть хороших (но еще не великий ) способ умножить любые два числа, например а · 10 х умножить на б · 10 у . Продукт просто аб · 10 х + у .
Это было здорово! Сложный и трудоемкий умножения теперь заменены одним простым умножением и простым добавление.
В настоящее время компьютеры делают все это за вас — и замена умножения на дополнение на самом деле является первым шагом к тому, как они это делают на самом деле.
Прежде чем я перейду к отличному способу делать умножения, я только упоминаю, что мы также рассмотрели деление сейчас. Если вы хотите разделить A на B вы Точно так же можно умножить A на 1/B . То, что вы получаете, это a · 10 x /(b · 10 y ) = (a/b) · 10 x у .
Теперь давайте перейдем к более сложному вещи. Мотивация в том, что мы не любим умножать или делить a и b . Ну, мы можем не нравится, но компьютера даже этого не умеют. Все, что они могут сделать, это сложить два числа, если они равны 0 или 1.
Итак, давайте расширим понятие экспоненты и разрешить любое число как экспонента, а не просто целых числа .
Другими словами: мы рассматриваем выражения типа 10 1,5 , 10 , 10 р , и, в частности, 10 i , где i является единицей мнимых чисел , определенных как i 2 = 1 .
Это немного сбивает с толку, потому что рецепт «подсчета нулей» теперь полностью выходит из строя. Преимущество этого несколько странного обобщения состоит в том, что что теперь мы можем выразить любым (положительным) воображаемое число и — большое, малое, целое, дробное, иррациональное, трансцендентный, воображаемый, сложный, какой угодно — на у = 10 х с x — число из списка выше.
Если нам нужно отрицательное число, мы просто пишем г = 10 x .
Хоппла, вдруг у нас функциональная связь с x как переменная: y( x ) = 10 x .
Нет пота. Если у нас есть известных x , мы можем вычислить значение г( х ) . Например, у нас есть y ( x = 2) = 10 2 = 100 , y( x = 2) = 10 2 = 1/10 2 = 1/100 = 0,01 .
Хорошо, но как насчет x = 1,3 ? Не волнуйся — будь счастлив. Eсть четко определенный алгоритм для вычисления y для все экспоненты . Просто посмотрите на график функции y( x ) = 10 x ниже, чтобы найти ответ или доверьтесь своему карманному калькулятору, который сделает это за вас
     
Функция y = 10 x построена из
     
Теперь предположим, что у вас есть число вроде 3,14 , и вы хотели бы знать, что x даст это число если принять за показатель степени 10 ?
Другими словами, у вас есть y( x ) = 10 x = 3,14 и вы хотите решить это уравнение для x .
Ну нельзя. Вам нужно изобрести логарифмов первый. Тогда решение становится x = lg 3,74 , а « lg » означает «логарифм к база 10».
Вы, наверное, не знаете, как это вычислить? Не волнуйтесь, вы, вероятно, не знать, как вычислить x = sin 3,74 или x = (3,74) 2,3 . В настоящее время вам не нужно знать, как это сделать. математики, потому что даже самый маленький и глупый карманный калькулятор с некоторыми математические функции будут знать все это. Вот почему мы изобрели их в первое место. Они просто сильно облегчают жизнь. Даже если вы до знаю, как делать работу выше, это скучная и утомительная работа.
     
  Полномочия Что угодно
Если вы обратили внимание, вы заметили что в примере выше я уже прокрался в очередное обобщение. Вы делаете а не нужно брать 10 как основание для ввода показателя степени. Ты можешь взять любой номер .
Если ваш показатель степени является целым числом, он напрямую понятно что делать. Показатель степени говорит вам, как часто умножать основание на сам. 3,74 3 = 3,74 · 3,74 · 3,74 = … . Если показатель степени , а не целое число, у вас просто есть ситуация, о которой я говорил выше — просто немного изменена..
Теперь должно немного помутнеть. У нас есть открыл бесконечность возможностей выразить любое заданное число y в степени: y = x a Основой нашего экспоненциального описания может быть любое число x , о котором вы хотите подумать; все, что вам нужно сделать, это найти правильный показатель степени a , который дает y , которые вы ищете.
Бесконечно много возможностей не как бы круто это ни звучало. Всего несколько близких друзей (девушек) лучше, чем бесконечно много на фейсбуке, например. Мы должны спросить себя, есть ли основания лучше других? Получаются два очевидных случая и один не столь очевидный случай. ум:
  1. Основание x = 10 . Это возвращает нас к тому, с чего мы начали. Нам нравится эта база, потому что мы считаем в системе с 10 цифрами. Собственно, мы выразились числа как суммы потенций 10 все время — даже если у вас может не быть знал об этом. Число 1,234 , например, просто означает 1 · 10 3 + 2 · 10 2 + 3 · 10 1 + 4 · 10 0 или 19 = 1 · 10 1 + 9 · 10 0 . Для любого числа y у вас есть y = 10 x и x = lg y для обратной зависимости.
    В «10 системе» нет ничего особенного, кроме что вы изобретете это почти автоматически, если вы считаете виды с 10 пальцами.
  2. Если вы такой вид, как компьютеры, которые могут считать только до двух (или компьютер учёные которые вообще без железа считать не умеют) вы используете только два цифры для счета: 0 и 1. У вас есть 2 в качестве основы для вашего счета система. 19 = 1 · 2 4 + 0 · 2 3 + 1 · 2 1 + 1 · 2 0 или 1011 . У нас есть у = 2 x и x = ld y для обратной зависимости; «лд» это аббревиатура для « двойной логарифм «или «логарифм по основанию 2».
  3. Если вы ученый или умный человек, вы используете базу x = e , иногда известное как число Эйлера , где e » 2,718281828… .
    У нас есть y = e x и x = ln y для обратной зависимости. Аббревиатура «ln» означает « натуральный логарифм », намекая на то, что основание e является наиболее естественным основанием для построения экспонент.
Почему e так заметно основание для экспонент и всего остального в математической науке? Если ты этого не знаешь, мне тебя жаль. Вы многое упустили в своем жизнь. Я не могу восполнить этот неудачный поворот вашей биографии всего за несколько линии. Таким образом, я дам вам только крошечное удовольствие от вкуса.
1. Почти все основные уравнения, описывающие вселенную и то, что происходит на вашей планете, так называемые дифференциальные уравнения. Как правило, вы хотите определить неизвестное функция y(x) , но все, что вы знаете, это то, как эта функция и ее производные относятся друг к другу. Например, ваше дифференциальное уравнение может быть d 2 у/дх 2 = А · у . На словах: вторая производная определяемой функции y равна сама функция, умноженная на некоторую константу А . Вы можете понять, что это было бы полезно знать функцию y(x) , которая совпадает со своей производной, или dy(x)/dx = y(x) .
Есть одна такая функция: y(x) = e x (также записывается как у(х) = ехр(х) )
Теперь вы знаете, почему «экспоненциалы» играют чрезвычайно заметную роль в наука. Они появляются более или менее автоматически, как только вы беретесь за дело. проблемы; вот пример с участием пиво.
2. Много чего есть происходящее в мире в целом содержит волны или колеблющиеся вещи. Если вы активировать эту ссылку, вы можете узнать, что каким-то образом все включает в себя волны или колеблющиеся вещи. Затем вам нужно описать что-то вроде этот в уравнения.
Хорошо — вы знаете, что y = sin x сделает эту работу. Правда — но с тригонометрическими функциями трудно иметь дело. Вот почему мы превращаем их в экспоненты, используя одно из самых замечательных уравнений всех математика:
     
e ix    =  cos x + i · sin х Уравнение Эйлера
         
e ip + 1  =    0     Тождество Эйлера
     
Эйлер Уравнение говорит вам, как перейти от функций синуса и косинуса к функциям гораздо более простые экспоненты. Приз, который нужно заплатить, заключается в том, что теперь вы должны работать с комплексные числа с вида с = с’ + ic» с i = воображаемая единица, обычно, хотя и несколько ошибочно, называемая «квадратом». корень минус -1″. Правильно это: i 2 = 1 . И нет, это не то же самое.
Однако это не является реальной проблемой, поскольку не только расчеты с комплексные числа часто намного проще, чем с прямыми числами, много проблем (включая почти всю квантовую теорию) просто не разрешимы без комплексные числа.
Эйлер тождество получается, если взять x = p . Это замечательное соотношение, потому что оно связывает пять самых важных чисел. по математике, из которого можно вывести все остальные: единица ( 1 ) и ноль ( 0 ), основание для всех натуральных чисел; i , основа для воображаемого числа; и иррациональные и выходят за пределы чисел стр и e , которые «каким-то образом» кодируют более глубокая работа Вселенной и, таким образом, всплывает все время.
     
  Графики и Уравнения
Вот графики наиболее важных экспоненциальные функции. Не используйте их для получения числовых значений, поскольку они немного качественный относительно подробности.
Если вы хотите на кривых, просто поменяйте на и на .
     
Графики 4 важнейших экспоненциальных функции
     
Вот несколько наиболее важных уравнения
     
е х  =  1
е x
  х ) у  =  е х · у   пер (х · у)  =  пер. х + пер. у
                 
е х · е у  =  е х + у   (e x ) 1/год  =  е х/у по х
г
 =  л х л н у
                     
е х
е у
 =  е х у   е длина х  =  х Д х Д  =  г · пер х
     
Вот два самых удивительных отношений, доказывая, что слово « натуральный » по отношению к числу e и его логарифм меток:
     
у(х)  =  e x     dy(x)
дх
 =  e x  
             
у(х)  =  л х   dy(x)
дх
 =  1
х
     
Если вам интересно, как на самом деле вычисляет числа, вот отношения. Если вы хотите рассчитать числовое значение e , просто возьмите x = 1 .
Если это ваша первая встреча с бесконечные ряды, вы упустили много чудес математики.
     
е х  =  1  +  х
1!
 +   х 2
2!
 +  x 3
3!
 + .

пер. (1 + х)  =  х   х 2
2
 +  x 3
3
    x 4
4
 + ..    

П (1 х)  =      æ
è
х + х 2
2
 +  x 3
3
 +   x 4
4
 + .

Как зная диаметр найти радиус: Онлайн калькулятор радиуса круга. Как узнать радиус круга, окружности.

как найти, зная один из параметров, формула с примерами

Связь между длиной окружности и площадью круга 

Определение 1

Окружность — это замкнутая плоская кривая, все точки которой удалены на одинаковое расстояние от заданной точки, которая называется центром окружности.

Источник: rusinfo.info

Определение 2

Центр окружности — точка, которая равноудалена от каждой точки окружности. Ее обозначают заглавной буквой О.

Определение 3

Круг — это часть плоскости, которая ограничена окружностью.

Источник: boeffblog.ru

Определение 4

Радиус окружности — это отрезок, соединяющий центр окружности с любой точкой, расположенной на линии окружности. Обозначается r или R. В окружности можно провести столько радиусов, сколько имеет точек сама окружность, то есть бесконечное множество. При этом все радиусы будут равны по величине.

Определение 5

Диаметр — это отрезок прямой, который проходит через центр окружности и соединяет две любые точки на линии окружности. Обозначается d или D. По величине диаметр равен двум радиусам:

D=2R, а следовательно радиус равен:

R=D:2

Определение 6

Длина окружности — это длина замкнутой плоской кривой, ограничивающей круг. Обозначается C. Длина окружности прямо пропорционально зависит от длины диаметра. Чем больше диаметр, тем больше длина окружности. Отсюда следует вывод, что отношение длины окружности к длине диаметра является постоянной величиной.

Эта величина, которая также называется коэффициентом пропорциональности, обозначается π. π — это иррациональное число, выражающееся бесконечной непериодической дробью. Для решения задач используется приближенное значение π ≈ 3,14.

Таким образом, можно вывести формулу длины окружности:

Зная, что  C:D=π, то отсюда C=πD.

А так как D=2R, то C=2πR.

Круг — это часть плоскости, а значит, одной из его характеристик является площадь. Площадь обозначается S.

S=πR²

А так как R=D:2,то S=π(D:2)²=(πD²):4

Методы решения задач, зная один из параметров

Для того чтобы решить задачу на нахождение длины окружности, площади круга, величины радиуса или диаметра, достаточно знать хотя бы один из параметров.

  1. Нахождение диаметра, зная радиус: D=2R
  2. Нахождение радиуса, зная диаметр: R=D:2
  3. Длина окружности, зная диаметр: C=πD
  4. Длина окружности, зная радиус: C=2πR
  5. Нахождение диметра, зная длину окружности: D=C:π
  6. Нахождение радиуса, зная длину окружности: R=C:(2π)
  7. Площадь, зная радиус: S=πR²
  8. Площадь, зная диаметр: S=(πD²):4
  9. Нахождение радиуса, зная площадь: R=√(S:π)
  10. Нахождение диаметра, зная площадь: D=√(4S:π)

Примеры решения задач

Задача 1

Овца привязана цепью длиной 9,6 м. Рассчитать доступную ей площадь.

Объяснение: если принять кол, к которому привязана овца за центр воображаемой окружности, то R этой окружности будет равен 9,6 м. Значит, необходимо найти площадь, зная радиус.

S=πR²=3,14*(9,6)² ≈289 м²

Задача 2

Вычислить длину окружности, если ее диаметр равен 5 м.

Правило нахождения длины окружности при известном радиусе C=2πR. Подставляем значения C=2*3,14*5=31,4 м

Как найти окружность зная диаметр.

Как найти длину окружности: через диаметр и радиус

Таким образом, длину окружности (C ) можно вычислить, умножив константу π на диаметр (D ), или умножив π на удвоенный радиус, так как диаметр равен двум радиусам. Следовательно, формула длины окружности будет выглядеть так:

C = πD = 2πR

где C — длина окружности, π — константа, D — диаметр окружности , R — радиус окружности.

Так как окружность является границей круга , то длину окружности можно также назвать длиной круга или периметром круга.

Задачи на длину окружности

Задача 1. Найти длину окружности, если её диаметр равен 5 см.

Так как длина окружности равна π умноженное на диаметр, то длина окружности с диаметром 5 см будет равна:

C ≈ 3,14 · 5 = 15,7 (см)

Задача 2. Найти длину окружности, радиус которой равен 3,5 м.

Сначала найдём диаметр окружности, умножив длину радиуса на 2:

D = 3,5 · 2 = 7 (м)

теперь найдём длину окружности, умножив π на диаметр:

C ≈ 3,14 · 7 = 21,98 (м)

Задача 3. Найти радиус окружности, длина которой равна 7,85 м.

Чтобы найти радиус окружности по её длине, надо длину окружности разделить на 2π

Площадь круга

Площадь круга равна произведению числа π на квадрат радиуса. Формула нахождения площади круга :

S = πr 2

где S — площадь круга, а r — радиус круга.

Так как диаметр круга равен удвоенному радиусу, то радиус равен диаметру, разделённому на 2:

Задачи на площадь круга

Задача 1. Найти площадь круга, если его радиус равен 2 см.

Так как площадь круга равна π умноженное на радиус в квадрате, то площадь круга с радиусом 2 см будет равна:

S ≈ 3,14 · 2 2 = 3,14 · 4 = 12,56 (см 2)

Задача 2. Найти площадь круга, если его диаметр равен 7 см.

Сначала найдём радиус круга, разделив его диаметр на 2:

7: 2 = 3,5 (см)

теперь вычислим площадь круга по формуле:

S = πr 2 ≈ 3,14 · 3,5 2 = 3,14 · 12,25 = 38,465 (см 2)

Данную задачу можно решить и другим способом. Вместо того чтобы сначала находить радиус, можно воспользоваться формулой нахождения площади круга через диаметр:

S = π D 2 ≈ 3,147 2 = 3,1449 =153,86 = 38,465 (см 2)
4444

Задача 3. Найти радиус круга, если его площадь равна 12,56 м 2 .

Чтобы найти радиус круга по его площади, надо площадь круга разделить π , а затем из полученного результата извлечь квадратный корень:

r = √S : π

следовательно радиус будет равен:

r ≈ √12,56: 3,14 = √4 = 2 (м)

Число

π

Длину окружности предметов, окружающих нас, можно измерить с помощью сантиметровой ленты или верёвки (нитки), длину которой потом можно померить отдельно. Но в некоторых случаях померить длину окружности трудно или практически невозможно, например, внутреннюю окружность бутылки или просто длину окружности начерченной на бумаге. В таких случаях можно вычислить длину окружности, если известна длина её диаметра или радиуса.

Чтобы понять, как это можно сделать, возьмём несколько круглых предметов, у которых можно измерить и длину окружности и диаметр. Вычислим отношение длины к диаметру, в итоге получим следующий ряд чисел:

Из этого можно сделать вывод, что отношение длины окружности к её диаметру это постоянная величина для каждой отдельной окружности и для всех окружностей в целом. Это отношение и обозначается буквой π .

Используя эти знания, можно по радиусу или диаметру окружности находить её длину. Например, для вычисления длины окружности с радиусом 3 см нужно умножить радиус на 2 (так мы получим диаметр), а полученный диаметр умножить на π . В итоге, с помощью числа π мы узнали, что длина окружности с радиусом 3 см равна 18,84 см.

Инструкция

В случае, если известен только диаметр, то формула будет выглядеть как «R = D/2».

Если длина окружности неизвестна, но есть данные о длине определенного , то формула будет иметь вид «R = (h^2*4 + L^2)/8*h», где h – высота сегмента (является расстоянием от середины хорды до самой выступающей части указанной дуги), а L – длина сегмента (которая не является длиной хорды). 2»).

Зная лишь длину диаметра окружности, можно вычислить не только площадь круга, но и площади некоторых других геометрических фигур. Это вытекает из того, что диаметры вписанных или описанных вокруг таких фигур окружностей совпадают с длинами их сторон либо диагоналей.

Инструкция

Если надо найти площадь (S) по известной длине его диаметра (D), умножайте число пи (π) на возведенную в длину диаметра , а результат делите на четыре: S=π ²*D²/4. Например, круга равен двадцати сантиметрам, то его площадь можно вычислить так: 3,14² * 20² / 4 = 9,86 * 400 / 4 = 986 сантиметров.

Если надо найти площадь квадрата (S) по диаметру вокруг него окружности (D), возводите длину диаметра в квадрат, а результат разделите пополам: S=D²/2. Например, если диаметр описанной окружности равен двадцати сантиметрам, то площадь квадрата можно вычислить так: 20² / 2 = 400 / 2 = 200 квадратных сантиметров.

Если площадь квадрата (S) нужно найти по диаметру вписанной в него окружности (D), достаточно возвести длину диаметра в квадрат: S=D². 2 / 4 + 20*10/2», а нажать клавишу Enter.

Источники:

  • как найти площадь окружности по диаметру

Круг — это плоская геометрическая фигура, все точки которой находятся на одинаковом и отличном от нуля удалении от выбранной точки, которую называют центром окружности. Прямую, соединяющую любые две точки круга и проходящую через центр, называют его диаметром . Суммарная длина всех границ двухмерной фигуры, которую обычно называют периметром, у круга чаще обозначается как «длина окружности». Зная длину окружности можно вычислить и ее диаметр.

Инструкция

Используйте для нахождения диаметра одно из основных свойств окружности, которое заключается в том, что соотношение длины ее периметра к диаметру одинаково для абсолютно всех окружностей. Конечно, постоянство не осталось не отмеченным математиками, и эта пропорция давно уже получила собственное — это число Пи (π — первая греческих слов «окружность » и «периметр»). Числовое этой определяется длиной окружности, у которой диаметр равен единице.

Делите известную длину окружности на число Пи, чтобы вычислить ее диаметр. Так как это число является « », то не имеет конечного значения — это дробь. Округляйте число Пи в соответствии с точностью результата, которую вам необходимо получить.

Используйте какой-либо , чтобы рассчитать длину диаметра, если сделать это в уме не получается. Например, можно воспользоваться тем, который встроен в поисковую систему Nigma или Google — он математические операции, вводимые на «человеческом» . Например, если известная длина окружности составляет четыре метра, то для нахождения диаметра можно «по-человечески» попросить поисковик: «4 метра разделить на пи». Но если вы введете в поле поискового запроса, например, «4/пи», то поисковик поймет и такую постановку задачи. В любом случае ответом будет «1.27323954 метра».

Вопрос о диаметре земного шара не так прост, как может показаться на первый взгляд, ведь само понятие «земной шар» весьма условно. У настоящего шара диаметр всегда будет одинаковым, в каком бы месте ни был проведен отрезок, соединяющий две точки на поверхности сферы и проходящий через центр.

Применительно к Земле не представляется возможным, поскольку ее шарообразность далеко не идеальна (в природе вообще не бывает идеальных геометрических фигур и тел, они представляют собой абстрактные геометрические понятия). Для точного обозначения Земли ученым даже пришлось ввести специальное понятие – «геоид».

Официальный диаметр Земли

Величина диаметра Земли определяется тем, в каком месте его будут измерять. Для удобства за официально признанный диаметр принимаются два показателя: диаметр Земли по экватору и расстояние между Северным и Южным полюсами. Первый показатель равен 12 756,274 км, а второй – 12 714, разница между ними составляет немногим менее 43 км.

Данные числа не производят особого впечатления, они уступают даже расстоянию между Москвой и Краснодаром – двумя городами, расположенными на территории одной страны. Тем не менее, вычислить их было непросто.

Вычисление диаметра Земли

Диаметр планеты высчитывается по такой же геометрической формуле, как и любой другой диаметр.

Чтобы найти периметр окружности, необходимо умножить ее диаметр на число πи. Следовательно, для нахождения диаметра Земли нужно измерить ее окружность в соответствующем сечении (по экватору или в плоскости полюсов) и разделить ее на число πи.

Первым человеком, попытавшимся измерить окружность Земли, был древнегреческий ученый Эратосфен Киренский. Он обратил внимание, что в Сиене (ныне – Асуан) в день летнего солнцестояния Солнце находится в зените, освещая дно глубокого колодца. В Александрии же в этот день оно отстояло от зенита на 1/50 окружности. Из этого ученый сделал вывод, что расстояние от Александрии до Сиена составляет 1/50 окружности Земли. Расстояние между этими городами равно 5 000 греческим стадиям (приблизительно 787,5 км), следовательно, окружность Земли равна 250 000 стадий (примерно 39 375 км).

В распоряжении современных ученых имеются более совершенные средства измерения, но их теоретическая основа соответствует идее Эратосфена. В двух точках, расположенных в нескольких сотнях километров друг от друга, фиксируют положение Солнца или определенных звезд на небосводе и вычисляют разницу между результатами двух измерений в градусах. Зная расстояние в километрах, несложно вычислить длину одного градуса, а затем умножить ее на 360.

Для уточнения размеров Земли используется и лазерная дальнометрия, и спутниковые системы наблюдения.

На сегодняшний день считается, что окружность Земли по экватору составляет 40 075,017 км, а по – 40 007,86. Эратосфен лишь немного ошибся.

Величина и окружности, и диаметра Земли увеличивается из-за метеоритного вещества, постоянно выпадающего на Землю, но процесс этот идет очень медленно.

Источники:

  • Как измерили Землю в 2019

Окружностью называется ряд равноудалённых точек от одной точки, которая, в свою очередь, является центром этой окружности. Окружность имеет также свой радиус, равный расстоянию этих точек от центра.

Отношение длины, какой либо окружности к её диаметру, для всех окружностей одинаково. Это отношение есть число, являющееся математической константой, которое обозначается греческой буквой π .

Определение длины окружности

Произвести расчёт окружности можно по следующей формуле:

L = π D = 2 π r

r — радиус окружности

D — диаметр окружности

L — длина окружности

π — 3. 14

Задача:

Вычислить длину окружности , имеющей радиус 10 сантиметров.

Решение:

Формула для вычисления дины окружности имеет вид:

L = π D = 2 π r

где L – длина окружности, π – 3,14 , r – радиус окружности, D – диаметр окружности.

Таким образом, длина окружности, имеющей радиус 10 сантиметров равна:

L = 2 × 3,14 × 10 = 62,8 сантиметра

Окружность представляет собой геометрическую фигуру, являющуюся совокупностью всех точек на плоскости, удаленных от заданной точки, которая называется ее центром, на некоторое расстояние, не равное нулю и именуемое радиусом. Определять ее длину с различной степенью точности ученые умели уже в глубокой древности: историки науки считают, что первая формула для вычисления длины окружности была составлена примерно в 1900 году до нашей эры в древнем Вавилоне.

С такими геометрическими фигурами, как окружности, мы сталкиваемся ежедневно и повсеместно. Именно ее форму имеет внешняя поверхность колес, которыми оснащаются различные транспортные средства. Эта деталь, несмотря на свою внешнюю простоту и незатейливость, считаются одним из величайших изобретений человечества, причем интересно, что аборигены Австралии и американские индейцы вплоть до прихода европейцев совершенно не имели понятия о том, что это такое.

По всей вероятности, самые первые колеса представляли собой отрезки бревен, которые насаживались на ось. Постепенно конструкция колеса совершенствовалась, их конструкция становилась все более и более сложной, а для их изготовления требовалось использовать массу различных инструментов. Сначала появились колеса, состоящие из деревянного обода и спиц, а затем, для того, чтобы уменьшить износ их внешней поверхности, ее стали обивать металлическими полосами. Для того чтобы определить длины этих элементов, и требуется использовать формулу расчета длины окружности (хотя на практике, вероятнее всего, мастера это делали «на глаз» или просто опоясывая колесо полосой и отрезая требуемый ее участок).

Следует заметить, что колесо используется отнюдь не только в транспортных средствах. Например, его форму имеет гончарный круг, а также элементы шестеренок зубчатых передач, широко применяемых в технике. Издавна колеса использовались в конструкциях водяных мельниц (самые древние из известных ученым сооружений такого рода строились в Месопотамии), а также прялок, применявшихся для изготовления нитей из шерсти животных и растительных волокон.

Окружности нередко можно встретить и в строительстве. Их форму имеют достаточно широко распространенные круглые окна, очень характерные для романского архитектурного стиля. Изготовление этих конструкций – дело весьма непростое и требует высокого мастерства, а также наличия специального инструмента. Одной из разновидностей круглых окон являются иллюминаторы, устанавливаемые в морских и воздушных судах.

Таким образом, решать задачу определения длины окружности часто приходится инженерам-конструкторам, разрабатывающим различные машины, механизмы и агрегаты, а также архитекторам и проектировщикам. Поскольку число π , необходимое для этого, является бесконечным, то с абсолютной точностью определить этот параметр не представляется возможным, и поэтому при вычислениях учитывается та ее степень, которая в том или ином конкретном случае является необходимой и достаточной.

Как найти длину радиуса

Все ресурсы по базовой геометрии

9 Диагностические тесты 164 практических теста Вопрос дня Карточки Learn by Concept

← Предыдущая 1 2 3 4 Следующая →

Справка по основам геометрии » Плоская геометрия » Круги » Радиус » Как найти длину радиуса

Площадь круга равна одному квадратному ярду. Дайте его радиус в дюймов, с точностью до десятых долей дюйма.

Возможные ответы:

Правильный ответ:

Объяснение:

Площадь круга

Замените 1 на:

Это радиус в ярдах. Радиус в дюймах в 36 раз больше этого.

20,3 дюйма — это радиус.

Сообщить об ошибке

Круг имеет площадь 36π дюймов. Каков радиус окружности в дюймах?

 

Возможные ответы:

6

9

36

18

Правильный ответ: 9 0018

6

Пояснение:

Мы знаем, что формула площади круга: π r 2 . Следовательно, мы должны установить 36π равным этой формуле, чтобы найти радиус круга.

36π = π r 2

36 = r 2

6 = r

Сообщить об ошибке

Круг X разделен на 3 части: A, B и C. Эти 3 части равны по площади. Если площадь сечения C равна 12π, каков радиус окружности?

         Обведите X

 

 

Возможные ответы:

4

√12

7

6

Правильный ответ:

6

Объяснение:

Найдите общую площадь круга, затем используйте формулу площади, чтобы найти радиус.

Площадь сечения A = сечение B = сечение C

Площадь круга X = A + B + C = 12π+ 12π + 12π = 36π

Площадь круга =  где r — радиус круга

36π = πr 2

36 = r 2

√36 = r

6 = r

 

Сообщить об ошибке 900 05

Согласно спецификации официального баскетбольного мяча NBA, он должен быть 290,5 дюйма в окружности и весом 22 унции. Каков примерный радиус баскетбольного мяча?

 

Возможные ответы:

14,75 дюйма

3,06 дюйма

9,39 дюйма

5,43 дюйма

4,70 дюйма

Правильный ответ:

4,70 дюйма

Пояснение:

Чтобы найти ответ, воспользуемся формулой: C=2πr. Нам дано, что C = 29,5. Таким образом, мы можем подключиться, чтобы получить [29.5]=2πr, а затем умножить 2π, чтобы получить 29,5=(6,28)r. Наконец, мы делим обе части на 6,28, чтобы получить 4,70 = r. (Информация о 22 унциях бесполезна)

 

Сообщить об ошибке

Как называется сегмент, выделенный красным?

Возможные ответы:

Луч

Хорда

Диагональ

Радиус

Диаметр

Правильный ответ: 90 018

Радиус

Объяснение:

Радиус — это расстояние от центра круга до любой точки его периметра.

Сообщить об ошибке

Как называется сегмент, выделенный коричневым цветом?

Возможные ответы:

Хорда

Радиус

Диаметр

Диагональ

Луч

Правильный ответ: 90 018

Аккорд

Объяснение:

Хорда — это отрезок, соединяющий две точки на кривой. Хорда не проходит через центр окружности.

Сообщить об ошибке

Диаметр круга 16 сантиметров. Каков радиус окружности в сантиметрах?

Возможные ответы:

Правильный ответ:

Объяснение:

Радиус равен половине диаметра. Чтобы найти радиус, просто разделите диаметр на 2.

Сообщить об ошибке

Окружность с центром (8, 5) касается оси y в стандартной (x,y) координатной плоскости . Каков радиус этой окружности?

Возможные ответы:

Правильный ответ:

8

Объяснение:

Чтобы окружность касалась оси Y, ее внешний край должен располагаться на оси. Центр находится в 8 единицах от края.

Сообщить об ошибке

Круг, показанный ниже, имеет площадь, равную . Какова длина радиуса этой окружности?

Возможные ответы:

Невозможно определить.

Правильный ответ:

Объяснение:

Формула площади круга . Мы можем заполнить то, что мы знаем, площадь, а затем найти радиус .

Разделите каждую часть уравнения на:

Извлеките квадратный корень из каждой стороны:

Сообщите об ошибке

Круг имеет площадь . Каков радиус окружности в дюймах?

Возможные ответы:

7 дюймов

49 дюймов

16 дюймов

14 дюймов

24,5 дюйма

9 0017 Правильный ответ:

7 дюймов

Пояснение:

Мы знаем, что формула площади круга: πr 2 . Следовательно, мы должны установить 49 π равным этой формуле, чтобы найти радиус круга.

49 π = πr 2

49 = r 2

7 = r

Сообщить об ошибке 9000 5

← Назад 1 2 3 4 Далее →

Уведомление об авторских правах

Все ресурсы по базовой геометрии

9 Диагностические тесты 164 практических теста Вопрос дня Карточки Learn by Concept

Радиус окружности — определение, формула, примеры решений и часто задаваемые вопросы

Радиус круга определяется как расстояние между центром и любой точкой на окружности круга. Это всегда половина диаметра круга. Радиус используется в круге для нахождения его площади и длины окружности, а также используется в других формулах. Поскольку мы знаем, что геометрическое место точки, расстояние до которой всегда фиксировано от фиксированного, является окружностью уравнения, а фиксированное расстояние является радиусом окружности.

Давайте узнаем о радиусе, его формуле и многом другом в этой статье.

Радиус Определение

Радиус — это отрезок, соединяющий центр круга или сферы с их периферией или границами. Это компонент кругов и сфер, который обычно обозначается аббревиатурой «r». Множественное число слова «радиус» — «радиусы», которое используется при одновременном обсуждении более чем одного радиуса. Диаметр круга или сферы — это самый длинный отрезок, соединяющий все точки на противоположной стороне от центра, а радиус равен половине длины диаметра.

 

Диаметр окружности

Линия, соединяющая две точки окружности и проходящая через центр окружности, называется диаметром окружности. Обозначается символом «d» или «D». Диаметр круга вдвое больше его радиуса.

  • Диаметр = 2 × Радиус
  • Радиус = Диаметр/2

Диаметр – это хорда окружности, которая является самой длинной в природе.

Формулы окружности (если указан диаметр)

Пусть d представляет диаметр окружности, тогда

  • Длина окружности = π(d)
  • Площадь окружности = π/4(d) 2

Радиус, диаметр и хорда

Любой Линии, проходящие через окружность, можно разделить на три категории:

  • Пересекающиеся прямые
  • Касательные к окружности
  • Непересекающиеся прямые

Пересекающиеся прямые

Если линия касается окружности ровно два раза, то она называется пересекающейся линией. Ее также называют секущей окружности.

Касательная к окружности

Если прямая касается окружности ровно один раз, то она называется касательной к окружности.

Непересекающиеся линии

Если линия не касается окружности, то она называется непересекающейся линией.

На изображении ниже показаны секущая к окружности, касательная к окружности и непересекающиеся прямые к окружности.

 

  • Любой отрезок, соединяющий центр круга с его окружностью, называется его радиусом.
  • Отрезок, соединяющий две точки на окружности окружности, называется хордой окружности.
  • Хорда, проходящая через центр окружности, называется диаметром окружности, которая является наибольшей хордой окружности.

 

Формула радиуса

Радиус окружности рассчитывается по некоторым специальным формулам, которые приведены ниже в таблице

Рад ius в терминах диаметра d ⁄ 2
Радиус по окружности  C ⁄ 2π
Радиус по площади √(A ⁄ π)

где,
d диаметр,
C длина окружности,
A площадь.

Формула радиуса из диаметра

Радиус равен половине диаметра. то есть Диаметр = 2 × радиус . Диаметр — это наибольшая хорда любого круга.

Радиус = диаметр / 2 

Формула радиуса из длины окружности

С = 2πr

где,
C длина окружности
r радиус
π константа со значением 3,14

Радиус окружности также определяется как отношение длины окружности к 2π. Формула радиуса с использованием длины окружности:

Радиус = длина окружности / 2π

Формула радиуса из площади

Пространство, занимаемое кругом, называется его площадью. Формула площади круга = πr 2 квадратных единиц определяет отношение радиуса любого круга к его площади. Здесь r представляет радиус, а константа π равна 3,14159.. Формула радиуса с использованием длины окружности выглядит следующим образом:

Радиус = √(Площадь / π) единиц

Как рассчитать радиус окружности?

Радиус окружности можно найти с помощью трех основных формул радиуса в соответствии с различными условиями, когда дан диаметр, когда дана площадь, и когда известна длина окружности. Воспользуемся этими формулами, чтобы найти радиус окружности.

  • Если Диаметр известен. Радиус = диаметр / 2
  • Если длина окружности известна. Радиус = Окружность / 2π
  • Если площадь известна. Радиус = √(Площадь круга/π)

Например,

  • Если диаметр равен 28 см, то радиус R = 28/2 = 14 см
  • Когда длина окружности круг равен 66 см, тогда радиус равен R = 66/2π = 10,5 см
  • Когда площадь круга равна 154 см 2 , тогда радиус равен R = √(154/π) = 7 см

Радиус сферы

Сфера представляет собой твердое трехмерное тело. Радиус сферы — это расстояние между ее центром и любой точкой на ее поверхности. Радиус сферы легко вычислить, если задан объем сферы или площадь поверхности сферы.

Радиус сферы при заданном объеме

R = 3 √{(3V) / 4π} единиц

где,
V представляет собой объем
π является константой со значением примерно 3,14

Радиус сферы при заданной площади поверхности

R = √(A / 4π) единиц

где
A представляет площадь поверхности 9 0511 π является константой со значением приблизительно 3.14

Уравнение радиуса окружности

Уравнение окружности на декартовой плоскости с центром (h, k) записывается как (x − h) 2 + (y − k) 2 = r 2 . Где (x, y) — геометрическое место любой точки на окружности круга, а «r» — радиус круга. Если начало координат (0,0) становится центром окружности, то ее уравнение задается как x 2 + у 2 = г 2 . Для нахождения радиуса окружности используется следующая формула:

(Радиус) r = √( x 2 + y 2 )

Теоремы окружности

Теорема 1: Перпендикулярная линия проведенный из центра окружности к хорде, делит хорду пополам.

 

Дано: 

Хорда AB и отрезок ОС перпендикулярны AB

Доказательство:

AC = BC

Конструкция:

Радиус соединения OA и OB

Доказательство:

В ΔOAC и ΔOBC

∠OCA = ∠OCB (OC перпендикулярен AB)

OA = OB       (Радиусы одной окружности)

OC = OC     (Общая сторона)

Итак, по критерию правосторонней конгруэнтности ΔOAC ≅ ΔOBC

Таким образом, AC = CB (по CPCT)

Обратное выше теорема тоже верна.

Теорема 2: Прямая, проведенная через центр окружности и делящая хорду пополам, перпендикулярна хорде.

(Ссылку на изображение см. на изображении выше.)

Дано:

C — середина хорды AB окружности с центром окружности в точке O

Чтобы доказать: 90 005

OC перпендикулярен AB

Конструкция: 

Соедините радиусы OA и OB, также соедините OC

Доказательство:

In ∆OAC и ∆OBC

AC = BC (Данные)

OA = OB (Радиусы одной окружности)

OC = OC (Общие)

По SSS критерий соответствия ∆OAC ≅ ∆ OBC 

∠1 = ∠2 (по CPCT)…(1)

∠1 + ∠2 = 180° (парные линейные углы)…(2)

Решение уравнения (1) и (2)

∠1 = ∠2 = 90° 

Таким образом, OC перпендикулярна AB

Также отметьте

  • Квадрат
  • Треугольник
  • Трапеция

Решенный пример по формулам радиуса

Пример 1: Найдите радиус круга, диаметр которого равен 18 см.

Решение:  

Дано, Диаметр круга = d = 18 см

Радиус круга с использованием диаметра,

Радиус = (диаметр ⁄ 2)
          = 18 ⁄ 2 см 
           = 9 см

Следовательно, радиус окружности равен 9 см.

Пример 2: Найдите радиус круга с длиной окружности 14 см.

Решение:

Радиус окружности с длиной окружности 14 см можно рассчитать по формуле

Радиус = Длина окружности / 2π

r = C / 2π

г = 14/2π {значение π = 22/7}

r = (14 × 7) / (2 × 22)

r = 98 / 44

r = 2,22 см

Следовательно, радиус данной окружности равен 2,22 см

Пример 3: Найдите площадь и длину окружности, радиус которой равен 12 см. (Взять значение π = 3,14)

Решение:  

Дано: радиус = 12 см.

Площадь круга = π r 2
                           = 3,14  × (12) 2

A = 452,6 см 2

Окружность круга,

C = 2πr
C = 2 × 3,14 × 12

Окружность = 75,36 см

Следовательно, площадь круга  452,6 см 2 и длина окружности 75,36 см

Пример 4: Каков радиус круга, если площадь равна 42 м 2 ?

Решение:  

Дано: Площадь круга, A = 42 м 2 .

Радиус круга с использованием площади можно рассчитать по формуле: радиус = √(A/π). Так, радиус длина окружности 13,36 м

Пример 5. Найдите диаметр окружности, зная, что площадь окружности численно равна удвоенной длине ее окружности.

Дано,

Площадь круга = 2 × длина окружности

Мы знаем, 

Площадь круга = π r 2
Длина окружности = 2πr

90 004 Следовательно,

π r 2 = 2 ×2×π×r

r = 4

Следовательно,

диаметр = 2 × радиус

диаметр = 2 × 4 Радиус круга

Q1: Какой радиус круга ?

Ответ:

Линия, соединяющая центр круга с любой точкой его окружности, называется радиусом круга. Обозначается буквой «r» или «R»

В2: Сколько радиусов можно провести в окружности?

Ответ:

Круг может иметь бесконечные радиусы внутри него.

© 2015 - 2019 Муниципальное казённое общеобразовательное учреждение «Таловская средняя школа»

Карта сайта